You are on page 1of 303



Question bank
PAST-PAPERS
‫‪‬إ‪‬اء‪‬‬
‫‪ ‬آﯾﺔ ﻣﻦ ﻛﺘﺎب ﷲ ﺣﻔﻈﻨﺎھﺎ وﻟﻜﻞ ﻣﻌﻠﻮﻣﺔ ﻗﺮأﻧﺎھﺎ وﻧﺴﯿﻨﺎھﺎ ﻟﻌﻞ ﷲ ﯾﺤﻤﯿﻨﺎ ﻣﻦ اﻟﻨﺴﯿﺎن وﯾﺮدھﺎ إﻟﯿﻨﺎ‬
‫‪ ‬ﺷﺨﺺ دﻋﻮﻧﺎ ﻟﮫ ﻓﻲ ظﮭﺮاﻟﻐﯿﺐ ﻟﻌﻞ ﷲ أن ﯾﺴﺘﺠﯿﺐ دﻋﺎﺋﻨﺎ‬
‫‪ ‬ﻣﺮﯾﺾ ﻗﺪ آﻟﻤﮫ ﻣﺎ أﻟﻤﮫ ﻟﻌﻞ ﷲ ﯾﺸﻔﯿﮫ وﯾﻌﯿﺪ إﻟﯿﮫ ﺻﺤﺘﮫ وﻋﺎﻓﯿﺘﮫ‬
‫‪ ‬ﻟﺤﻈﺔ ﺗﺂﻟﻢ ﺑﮭﺎ أﺧﻲ وﺟﺪي وأﺣﺒﺘﻲ أﺛﻨﺎء ﻋﻼﺟﮭﻢ ﻟﻌﻞ ﷲ ﻻ ﯾﺮﯾﮭﻢ اﻵﻟﻢ وﻻ ﯾﻀﯿﻤﮭﻢ ﻣﺮة أﺧﺮى‬
‫‪ ‬ﻣﻮﻗﻒ اﺣﺘﺠﺖ ﺑﮫ ﻋﯿﻨﺎي ﻓﻠﻢ ﺗﻠﺒﯿﺎﻧﻲ ﻟﻌﻞ ﷲ ﻻ ﯾﻀﺮﻧﻲ ﺑﮭﻤﺎ ﻓﻲ ﺣﯿﺎﺗﻲ‬
‫‪ ‬ﺣﺎﻟﻢ ﺻﺎﺣﺐ أھﺪاف وﻣﺒﺎدئ ﻟﻌﻞ ﷲ ﯾﺤﻘﻖ ﻟﮫ ﻣﻘﺎﺻﺪه ﺣﺘﻰ ﯾﺮﺿﯿﮫ وﯾﺮﺿﻰ ﻋﻨﮫ‬
‫‪ ‬ﻣﺠﺘﮭﺪ ﻟﻌﻞ ﷲ ﯾﺠﻌﻞ ﻟﮫ ﻣﻦ اﻟﺘﻮﻓﯿﻖ أوﻓﺮ ﺣﻆ و ﻧﺼﯿﺐ وﯾﮭﻮن ﻟﮫ اﻷﺳﺒﺎب ﻛﻠﮭﺎ ﻣﻦ ﺣﯿﺚ ﻻ‬
‫ﯾﺪري ﻣﻦ ﻏﯿﺮ ﺿﺮاء ﻣﻀﺮة وﻻ ﻓﺘﻨﺔ ﻣﻀﻠﮫ‬
‫‪ ‬ﻣﻦ ﻓﻘﺪ ﺷﯿﺌﺎ ﻓﻲ اﻟﺤﯿﺎة ﻟﻌﻞ ﷲ ﯾﻌﯿﺪه إﻟﯿﮫ أو ﯾﻌﻮﺿﮫ ﺑﻤﺎ ﯾﺴﻌﺪه‬
‫‪ ‬ﺷﺨﺺ اﺑﺘﻌﺪﻧﺎ ﻋﻨﮫ ﻟﻌﻞ ﷲ ﯾﺠﻤﻌﮫ ﺑﻤﻦ ھﻢ أﻓﻀﻞ ﻣﻨﺎ‬
‫وأ‪‬ا وﻟﯿﺲ آﺧﺮا ﻷھﻠﻨﺎ ﻟﻌﻞ ﷲ ﯾﺠﻌﻠﮭﺎ ﻟﮭﻢ ﺻﺪﻗﺔ ﺟﺎرﯾﮫ ﺗﺮد إﻟﯿﮭﻢ ﻗﻠﯿﻼ ﻣﻦ ﻓﻀﻠﮭﻢ ﻋﻠﯿﻨﺎ وﺗﻜﻮن ﺳﺒﺒﺎ‬
‫ﯾﻠﻢ ﺷﻤﻠﻨﺎ ﺗﺤﺖ ﺳﻘﻒ واﺣﺪ ﺑﻌﺪ أن ھ ﱠﺠ َﺮﺗﻨﺎ اﻟﺤﺮوب واﻟﻤﺼﺎﺋﺐ‬

‫‪PT Part I 2019 index‬‬


‫‪Branch‬‬ ‫‪No. of Q‬‬
‫‪Cardiology‬‬ ‫)‪(386‬‬
‫‪Clinical haematology/oncology‬‬ ‫)‪(392‬‬
‫‪Clinical‬‬
‫)‪(513‬‬
‫‪pharmacology/therapeutics/toxicology‬‬
‫‪Dermatology‬‬ ‫)‪(138‬‬
‫‪Endocrinology‬‬ ‫)‪(414‬‬
‫‪Gastroenterology‬‬ ‫)‪(403‬‬
‫‪General Revision‬‬ ‫)‪(122‬‬
‫‪Infectious diseases/sexually‬‬
‫)‪(412‬‬
‫‪transmitted diseases‬‬
‫‪Nephrology‬‬ ‫)‪(335‬‬
‫‪Neurology‬‬ ‫)‪(413‬‬
‫‪Ophthalmology‬‬ ‫)‪(81‬‬
‫‪Psychiatry‬‬ ‫)‪(268‬‬
‫‪Respiratory Medicine‬‬ ‫)‪(457‬‬
‫‪Rheumatology‬‬ ‫)‪(380‬‬
Clinical Sciences
Cell/molecular and membrane
(72)
biology
Clinical anatomy (67)
Clinical biochemistry and metabolism (118)
Clinical physiology (92)
Genetics (107)
Immunology (118)
Statistics (158)

Past-Papers index
Passing mark range between 63-66%
Year No. of Q
MRCP 1 May 19 100
MRCP 1 Jan 19 100
MRCP 1 Sept 18 100
MRCP 1 May 18 100
MRCP 1 Jan 18 100
MRCP 1 Sept 17 100
MRCP 1 May 17 100
MRCP 1 Jan 17 100
MRCP 1 Sept 16 100
MRCP 1 May 16 100
MRCP 1 Jan 2016 100
MRCP 1 Sept 2015 100
MRCP 1 May 2015 100
MRCP 1 Jan 2015 100

A. H. Murad
‫ﻻ ﺗﻨﺴﻮﻧﺎ ﻣﻦ ﺻﺎﻟﺢ دﻋﺎﺋﻜﻢ‬
‫ﺗﻢ ﺑﺤﻤﺪ ﷲ وﺗﻮﻓﯿﻘﮫ وﻣﻨﮫ‬
‫ﺟﻌﻞ ﷲ ﻋﻤﻠﻨﺎ ﻣﺘﻘﺒﻼ ﺧﺎﻟﺼﺎ ﻟﻮﺟﮭﮫ اﻟﻜﺮﯾﻢ‬
0:02:24/03:00:00

Haemodialysis machines utilise osmosis in the process of removing waste products from
blood.
Which of the following best describes the process of osmosis?

A Active transport of large molecules across a semi-permeable membrane

B Movement of molecules from a high concentration to a low concentration in a


solvent pool

C Oxidation of charged particles in plasma

D Passage of solvent from a low to a high concentration across a semi-permeable


membrane

E Passage of solutes from a high to a low concentration across a semi-permeable


membrane

Explanation 

D Passage of solvent from a low to a high concentration across a semi-permeable


membrane

The process of osmosis is utilised in medical procedures such as haemodialysis and


peritoneal dialysis, and is involved in disease processes such as central pontine myelinolysis.

A Active transport of large molecules across a semi-permeable membrane

Active transport of large molecules across a semi-permeable membrane is incorrect. This


implies a process which utilises energy. Examples of active transporters of charged ions
include those found in the kidney.

B Movement of molecules from a high concentration to a low concentration in a


solvent pool
Movement of molecules from a high concentration to a low concentration in a solvent pool is
incorrect. This is the process of diffusion, which occurs when a concentrated solution is
dissolved in water.

C Oxidation of charged particles in plasma

Oxidation of charged particles in plasma is incorrect. Oxidation is not linked to osmosis.


Examples of where oxidation is involved in disease processes include methaemoglobinaemia,
where oxidation of Fe 2+ to Fe 3+ results in severe tissue hypoxia.

E Passage of solutes from a high to a low concentration across a semi-permeable


membrane

Passage of solutes from a high to a low concentration across a semi-permeable membrane is


incorrect. This is the process of reverse osmosis, which is utilised in techniques to purify
water.
51398

Rate this question:

Next Question

Previous Question Tag Question

Feedback End Review

Difficulty: Average

Peer Responses %

Q. Answered Flagged

Q1

Q2
Q. Answered Flagged

Q3

Q4

Q5

Q6

Q7

Q8 
0:02:24/03:00:00

A 74-year-old woman is admitted to the Emergency Department with symptoms of anaemia.


She has started fludarabine for chronic lymphocytic leukaemia (CLL) a few weeks earlier. She
looks pale, her blood pressure is 105/70 mmHg, pulse is 95 bpm and regular. She is short of
breath at rest and there is obvious splenomegaly on abdominal palpation.
Investigations:

Hb 70 g/l (MCV 92 fl)

Coombs positive

WCC 47.1 ×10 9/l

PLT 149 × 10 9/l

Na + 142 mmol/l

K+ 5.2 mmol/l

Cr 129 µmol/l

Which of the following is the most appropriate intervention?

A Alemtuzumab

B Chlorambucil

C Cyclophosphamide

D Prednisolone

E Rituximab

Explanation 

D Prednisolone

This patient has autoimmune haemolytic anaemia (AHA) as a result of CLL. Fludarabine, a
commonly used therapy for CLL, increases the risk of an imbalance in T-regulatory
lymphocyte function and AHA. Prednisolone is the initial intervention of choice. In patients
who fail to respond, rituximab is an appropriate next step.
A Alemtuzumab

Alemtuzumab is incorrect. Alemtuzumab is a monoclonal antibody directed at CD52 that is


approved for use in CLL. It is currently licensed for use in patients who relapse on fludarabine
therapy or don’t respond to it. Response rates of around 50% are seen.

B Chlorambucil

Chlorambucil is incorrect. Chlorambucil is an alkylating agent used as an alternative first-line


agent for treating CLL to fludarabine. Bendamustine may also be used as an initial therapy
and endorsed by The National Institute of Health and Care Excellence (NICE) for Binet stage
B or C CLL, where fludarabine combination therapy isn’t appropriate.

C Cyclophosphamide

Cyclophosphamide is incorrect. Cyclophosphamide is used in combination with fludarabine


for the treatment of CLL but it isn’t a first-line treatment option for haemolytic anaemia
associated with the disease.

E Rituximab

Rituximab is incorrect. Rituximab is an anti-CD20 monoclonal antibody which is used second


line for the treatment of AHA in CLL for patients who fail to respond to corticosteroid
therapy alone. Haemolytic anaemia which is refractory to therapy may eventually require
blood transfusion as frequently as every 3–4 weeks.
51379

Rate this question:

Next Question

Previous Question Tag Question

Feedback End Review

Difficulty: Average

Peer Responses %
Q. Answered Flagged

Q1

Q2

Q3

Q4

Q5

Q6

Q7

Q8

Q9

0:02:24/03:00:00

A 71-year-old man who is treated with warfarin for chronic atrial fibrillation comes to the
Emergency Department with a large upper gastrointestinal haemorrhage. He is thought to
have vomited at least 250 ml of fresh haematemesis. A bedside international normalised ratio
(INR) test is 9.1. On examination, he is exquisitely tender in the epigastrium, his blood
pressure is 90/60 mmHg, pulse is 110 bpm (atrial fibrillation). He is given intravenous (IV)
vitamin K. Hb is 84 g/l, platelet count is 95 × 10 9/l.
Which of the following is the most appropriate next intervention?

A Fresh frozen plasma (FFP)

B Platelet transfusion

C Protamine sulphate

D Prothrombin complex concentrate (PCC)

E 3-unit packed cells transfusion

Explanation 

D Prothrombin complex concentrate (PCC)

PCC contains factors II, VII, IX and X and is a targeted agent to reverse warfarin-related
coagulopathy. It is given at a dose of 25–50 U/kg together with IV vitamin K when patients
present with INR > 8 and significant haemorrhage. Blood transfusion may of course also be
required but this is secondary to warfarin reversal.

(see External Links)

A Fresh frozen plasma (FFP)

FFP is incorrect. FFP is less effective than PCC in reversing warfarin therapy. It is given at a
dose of 15 mg/kg, but should only be used in the event that PCC is not available.

B Platelet transfusion
Platelet transfusion is incorrect. Platelet transfusion is usually instigated in cases of active
bleeding when the platelet count is below 20 × 10 9/l.

C Protamine sulphate

Protamine sulphate is incorrect. Protamine is utilised for reversal of heparin treatment. When
dalteparin has been given less than 8 h before reversal is required, a dose of 1 mg of
protamine for every 100 units of dalteparin is required.

E 3-unit packed cells transfusion

3-unit packed cells transfusion is incorrect. A transfusion will be required, although it is


secondary to reversal of warfarin therapy with PCC first.
51380

Rate this question:

Next Question

Previous Question Tag Question

Feedback End Review

Difficulty: Average

Peer Responses %

Q. Answered Flagged

Q1

Q2

Q3
Q. Answered Flagged

Q4

Q5

Q6

Q7

Q8

 External Links

octaplex 500 IU
medicines.org.uk/emc/product/6566/smpc
(https://www.medicines.org.uk/emc/product/6566/smpc)
0:02:24/03:00:00

A 56-year-old woman is referred to the Lipid Clinic for review. She has been noted by her GP
to have both hypercholesterolaemia and hypertriglyceridaemia and also has hypertension,
currently managed with ramipril 10 mg daily. Her blood pressure is 152/82 mmHg, pulse is 52
bpm and regular. You note that she has thinning hair and is obese with a body mass index of
32 kg/m 2.
Investigations:

Hb 125 g/l (MCV 102 fl)

WCC 7.1 × 10 9/l

PLT 231 × 10 9/l

Na + 131 mmol/l

K+ 4.5 mmol/l

Bicarbonate 24 mmol/l

Cr 122 µmol/l

Glucose 5.6 mmol/l

HDL 0.9 mmol/l

LDL 3.8 mmol/l

Triglycerides 2.4 mmol/l

Which of the following is the most likely cause of her dyslipidaemia?

A Cushing syndrome

B Familial combined hyperlipidaemia

C Familial hypercholesterolaemia

D Hypothyroidism

E Polycystic ovarian syndrome (PCOS)

Explanation

D Hypothyroidism

Clues here to hypothyroidism include the clinical picture of relative bradycardia, thinning hair,
increased BMI, as well as biochemical profile of mixed dyslipidaemia, low serum sodium and
raised mean corpuscular volume (MCV). Taken together, these factors should prompt thyroid
function testing as the most appropriate next step. In the event hypothyroidism is confirmed,
thyroid hormone replacement is likely to result in an improvement in the lipid profile.

A Cushing syndrome

Cushing syndrome is incorrect. Although Cushing syndrome could account for the picture of
dyslipidaemia seen here, it is less likely given the low serum sodium and normal serum
bicarbonate.

B Familial combined hyperlipidaemia

Familial combined hyperlipidaemia is incorrect. Familial combined hyperlipidaemia would be


considered in the event this picture of dyslipidaemia was still present despite correction of
thyroid hormone status. A greater elevation in triglycerides and low-density lipoprotein (LDL)
might also be expected.

C Familial hypercholesterolaemia

Familial hypercholesterolaemia is incorrect. Familial hypercholesterolaemia is an autosomal


dominant condition which results in an isolated rise in LDL cholesterol, much greater than
that seen here. It’s also not associated with an increase in MCV nor with hyponatraemia.

E Polycystic ovarian syndrome (PCOS)

PCOS is incorrect. PCOS presents at an earlier age with features of hyperandrogenism and
insulin resistance. It isn’t consistent with the picture seen here.
51362

Rate this question:

Next Question

Previous Question Tag Question

Feedback End Review

Difficulty: Average
Peer Responses %

Q. Answered Flagged

Q1

Q2

Q3

Q4

Q5

Q6

Q7

Q8

Q9

0:02:24/03:00:00

A 25-year-old motorcyclist comes to the clinic for review a few weeks after injury to his right
arm from an accident. He complains of a patch of numbness on the inside of his right arm,
and difficulty holding things in his right hand. Examination reveals numbness over the medial
aspect of the upper arm and weakness of the intrinsic muscles of the hand.
Where is the most likely site of the lesion?

A C5

B C6

C C7

D C8

E T1

Explanation 

E T1

T1 lesions present with weakness and wasting of the intrinsic muscles of the hand and the
interossei, and areas of sensory loss corresponding to the T1 dermatome over the medial
aspect of the upper arm.

A C5

C5 is incorrect. C5 innervates the deltoids and the biceps and supplies sensation to the area
over the deltoids (known as the regimental badge area).

B C6

C6 is incorrect. C6 partially supplies the biceps and the wrist extensors (extensor carpi
radialis longus and brevis). It supplies sensation to the lateral forearm.

C C7
C7 is incorrect. C7 supplies motor function to the triceps, wrist flexors and finger extensors. It
supplies sensation to the area over the middle finger.

D C8

C8 is incorrect. C8 is responsible for motor supply to the finger flexors, it supplies sensation
to the medial forearm and hand.
51394

Rate this question:

Next Question

Previous Question Tag Question

Feedback End Review

Difficulty: Average

Peer Responses %

Q. Answered Flagged

Q1

Q2

Q3

Q4

Q5

Q6

Q7
Q. Answered Flagged

Q8
0:02:24/03:00:00

You are asked to review a 72-year-old patient who is suffering from orthostatic hypotension.
He has suffered three syncopal episodes over the past four months.
When a patient without orthostatic hypotension stands up for one minute, which of the
following occurs?

A Central venous pressure (CVP) increases

B Heart rate decreases

C Stroke volume increases

D Systemic vascular resistance increases

E Venous compliance increases

Explanation 

D Systemic vascular resistance increases

Sympathetic activity increases systemic vascular tone on standing to combat the effects of
venous pooling in the lower limbs in reducing blood pressure.

A Central venous pressure (CVP) increases

CVP increases is incorrect. CVP falls on standing because of venous pooling in the lower
limbs, and a reduction in thoracic venous pressure.

B Heart rate decreases

Heart rate decreases is incorrect. Heart rate increases in patients on standing to compensate
for reduced stroke volume and to maintain mean arterial pressure.

C Stroke volume increases

Stroke volume increases is incorrect. Stroke volume decreases on standing because of


decreased pre-load as a result of pooling of blood in the lower limbs.
E Venous compliance increases

Venous compliance increases is incorrect. Venous compliance decreases on standing because


of sympathetic nervous system action on large capacitance vessels in an attempt to maintain
arterial blood pressure.
51399

Rate this question:

Next Question

Previous Question Tag Question

Feedback End Review

Difficulty: Average

Peer Responses %

Q. Answered Flagged

Q1

Q2

Q3

Q4

Q5

Q6

Q7

Q8



Q Answered Flagged
0:02:24/03:00:00

A 23-year-old man presents to the Emergency Department with a narrow complex


tachycardia and pulse rate up to 180 bpm. He tells you that the palpitations began one hour
earlier after returning home from the local gym. He suffered a similar episode of paroxysmal
supraventricular tachycardia (SVT) around two weeks earlier. Examination reveals a blood
pressure of 115/75 mmHg, pulse is confirmed at 180 bpm. Chest is clear and there is no ankle
swelling.
Which of the following is the most appropriate intervention?

A Adenosine

B Direct current (DC) cardioversion

C Ice swallowing

D Postural modified Valsalva manoeuvre

E Verapamil

Explanation 

D Postural modified Valsalva manoeuvre

The postural modification to the standard Valsalva manoeuvre for emergency treatment of
supraventricular tachycardias (REVERT) trial suggested that the head down Valsalva
manoeuvre leads to reversion to sinus rhythm for SVT in approximately 40% of patients
versus less than 20% for the standard Valsalva approach. This has led to it being the
intervention of choice for patients presenting with paroxysmal SVT without circulatory
compromise.
(see External Links)

A Adenosine

Adenosine is incorrect. Adenosine causes a brief pause in cardiac conduction, which is


effective in reverting SVT. It is, however, associated with feelings of impending doom which
can be very unpleasant for patients and is contraindicated in patients with a history of
asthma.
B Direct current (DC) cardioversion

DC cardioversion is incorrect. DC cardioversion is indicated for patients with circulatory


compromise or those who fail to respond to intervention with Valsalva manoeuvre or
adenosine.

C Ice swallowing

Ice swallowing is incorrect. Ice swallowing has a similar success rate in achieving sinus rhythm
to the Valsalva manoeuvre and is therefore unlikely to be successful here.

E Verapamil

Verapamil is incorrect. Verapamil is effective in slowing ventricular rate for patients


presenting with SVT but would be considered a third-line intervention here after modified
Valsalva and adenosine. Other options may, for example, include carotid sinus massage, but
this is less effective in stimulating the vagus nerve than the REVERT procedure.
51366

Rate this question:

Next Question

Previous Question Tag Question

Feedback End Review

Difficulty: Average

Peer Responses %

Q. Answered Flagged

Q1
Q. Answered Flagged

Q2

Q3

Q4

Q5

Q6

Q7

Q8 

 External Links

Postural modification to the standard Valsalva manoeuvre for emergency treatment of supraventricular ta…
thelancet.com/journals/lancet/article/PIIS0140-6736(15)61485-4/abstract
(http://www.thelancet.com/journals/lancet/article/PIIS0140-6736(15)61485-
4/abstract)
0:02:24/03:00:00

A 29-year-old woman is brought to the Emergency Department with aggressive behaviour,


having hit another person in a bar. She tells you that she can do what she likes because she is
a millionaire, and when you talk to her you note clear evidence of pressure of speech, with
racing thoughts and switching of the conversation every 1–2 min. You understand from a
colleague who is with her that she has a very successful job in a bank and no previous
medical or psychiatric illnesses. Physical examination reveals a blood pressure of 164/100
mmHg, her pulse is 100 bpm and regular.
Which of the following is the most likely diagnosis?

A Amphetamine psychosis

B Bipolar disease type 1

C Marijuana abuse

D Schizoaffective disorder

E Schizophrenia

Explanation 

A Amphetamine psychosis

The features of an acute manic episode coupled with marked sympathetic activation with
hypertension and tachycardia drive us towards amphetamine psychosis rather than bipolar
disease as the likely diagnosis. In the acute situation benzodiazepines to reduce agitation
would be the intervention of choice.

B Bipolar disease type 1

Bipolar disease type 1 is incorrect. Bipolar disease type 1 presents with florid manic episodes
resulting in frequent hospital admissions, interspersed with major depressive episodes. There
is no evidence that there is underlying bipolar disease given she is a high-functioning
individual with no previous psychiatric history.

C Marijuana abuse
Marijuana abuse is incorrect. Marijuana abuse is more likely to be associated with depressive
symptoms. It is associated with increased risk of hallucinations and delusions in keeping with
a diagnosis of schizophrenia, rather than the manic picture seen here.

D Schizoaffective disorder

Schizoaffective disorder is incorrect. Schizoaffective disorder presents with bipolar or


depressive symptoms, the presentation here with acute agitation, coupled with evidence of
sympathetic activation, fits better with amphetamine abuse.

E Schizophrenia

Schizophrenia is incorrect. Evidence of a more prolonged deterioration, coupled with more


pronounced hallucinations and other features of schizophrenia, including thought insertion,
thought broadcasting and delusions of reference, would be expected to make this diagnosis.
51443

Rate this question:

Next Question

Previous Question Tag Question

Feedback End Review

Difficulty: Average

Peer Responses %

Q. Answered Flagged

Q1

Q2
Q. Answered Flagged

Q3

Q4

Q5

Q6

Q7

Q8
0:02:24/03:00:00

A 65-year-old man with type 2 diabetes controlled with metformin 1 g twice a day, comes to
the Cardiology Clinic for review. He has noted over the last six months that he is having to
use one puff of glyceryl trinitrate at the end of his dog walk as he approaches a small hill and
before he mows his lawn. An angiogram from nine months earlier shows distal disease which
is not amenable to percutaneous intervention. Other medication includes ramipril 10 mg,
indapamide 2.5 mg, bisoprolol 10 mg, and atorvastatin 40 mg. Pulse is 60 bpm and regular,
and blood pressure is 127/82 mmHg.
Which of the following is the most appropriate next intervention?

A Amlodipine

B Doxazosin

C Isosorbide mononitrate (ISMN)

D Ivabradine

E Nebivolol

Explanation 

C Isosorbide mononitrate (ISMN)

For patients with chronic stable angina who are uncontrolled on maximal beta-blockade,
ISDN is the logical next step. Tolerance is seen in some patients to ISDN, where further
therapy may be required.

A Amlodipine

Amlodipine is incorrect. Amlodipine may lead to an increase in heart rate and is therefore
usually considered only after beta-blockade and nitrate therapy in combination. There is
thought to be little value in going above 5 mg daily dose since 10 mg usually only leads to a
worsening of dependent oedema in the lower limbs.

B Doxazosin
Doxazosin is incorrect. Doxazosin is an alpha-blocker. These are less effective than other
options in controlling angina and may exacerbate/increase the risk of cardiac failure; as such,
as a class, they are best avoided here.

D Ivabradine

Ivabradine is incorrect. Ivabradine is an I f channel inhibitor and an effective treatment for


both angina and heart failure but as the heart rate in this patient is below 75 bpm, there is no
evidence it will improve symptoms here.

E Nebivolol

Nebivolol is incorrect. Nebivolol is a beta-blocker with some nitric-oxide-mediated


vasodilating properties. It is thought to be less likely to cause tolerance than isosorbide
mono- or dinitrate preparations. Since this patient is already treated with bisoprolol, nebivolol
is therefore not required here.
51367

Rate this question:

Next Question

Previous Question Tag Question

Feedback End Review

Difficulty: Average

Peer Responses %

Q. Answered Flagged

Q1

Q2
Q. Answered Flagged

Q3

Q4

Q5

Q6

Q7

Q8 
0:02:24/03:00:00

A 63-year-old man comes to the Neurology Clinic for review. He has recently retired early
from his job as a solicitor after allegations of inappropriate touching of female staff were
made against him and an episode of bizarre binge eating behaviour at a staff function. His
wife tells you that he has begun to hoard and check paperwork and mail obsessively and that
there are piles of papers all over the house. Anterior temporal and medial frontal atrophy is
noted on magnetic resonance imaging (MRI) scan, and there is frontal hypoperfusion on
positron emission tomography (PET) imaging.
Which of the following is the most appropriate pharmacological intervention?

A Carbamazepine

B Chlorpromazine

C Donepezil

D Lorazepam

E Paroxetine

Explanation 

E Paroxetine

There is evidence that behavioural changes seen in fronto-temporal dementia result from
alterations in serotoninergic neurotransmission. Trials of selective serotonin reuptake
inhibitors (SSRIs) including paroxetine and fluoxetine have shown a reduction in disinhibited
and inappropriate behaviour. Paroxetine is, therefore, an appropriate intervention here.

A Carbamazepine

Carbamazepine is incorrect. Anti-seizure drugs are used to modulate behaviour in fronto-


temporal dementia, although there is little conclusive evidence to support their use. A small
trial suggested that topiramate may modulate compulsive alcohol abuse in association with
fronto-temporal dementia.

B Chlorpromazine
Chlorpromazine is incorrect. Conventional anti-psychotics are not recommended in the
treatment of fronto-temporal dementia. These patients are particularly prone to the extra-
pyramidal side-effects. Atypical anti-psychotics may be of value at low dose in treating
delusions.

C Donepezil

Donepezil is incorrect. There is little evidence that the cholinergic nervous system is
disordered in frontotemporal dementia. These drugs may actually make disinhibited
behaviour worse and should therefore be avoided.

D Lorazepam

Lorazepam is incorrect. Benzodiazepines lead to paradoxical agitation in patients who have a


diagnosis of fronto-temporal dementia. They should therefore be avoided.
51434

Rate this question:

Next Question

Previous Question Tag Question

Feedback End Review

Difficulty: Average

Peer Responses %

Q. Answered Flagged

Q1

Q2
Q. Answered Flagged

Q3

Q4

Q5

Q6

Q7

Q8
0:02:24/03:00:00

A 34-year-old woman is admitted to the Emergency Department with a sudden onset, severe,
occipital headache, which has been followed by marked neck pain and stiffness. The pain
began in the morning and by the time she has presented to the hospital some 5 h has passed.
She smokes 20 cigarettes per day and tells you that she uses cocaine on two to three
occasions per month. Examination reveals a blood pressure of 178/105 mmHg, pulse is 85
bpm and regular. There is no evidence of papilloedema. She has obvious neck stiffness.
Computed tomography (CT) brain shows no evidence of acute haemorrhage. Analgesia is
prescribed.

Which of the following is the most appropriate next step?

A Amlodipine

B Lumbar puncture

C Magnetic resonance angiogram (MRA)

D Observation

E Ramipril

Explanation 

B Lumbar puncture

The suspicion here is of subarachnoid haemorrhage, given the history and likely paroxysms of
hypertension related to smoking and cocaine abuse. A non-contrast CT actually rules out
98% of subarachnoid haemorrhages (SAHs), although given the potentially catastrophic
effects of missing a small herald bleed, the majority of patients with this history progress to
lumbar puncture (LP). Blood cells present in serial bottles can be measured if the LP is
conducted within 12 h; after 12 h, xanthochromia can be measured.

A Amlodipine

Amlodipine is incorrect. In the event this is a subarachnoid haemorrhage, acute control of


blood pressure may actually worsen neurological deficit; as such, amlodipine isn’t required
here. Studies suggest that in aneurysmal SAH acute lowering of blood pressure may reduce
the risk of a further bleed but can increase the risk of a watershed territory cerebral infarction
occurring. Nimodipine which is selective for the cerebral vasculature, may be used to reduce
cerebral artery vasospasm.

C Magnetic resonance angiogram (MRA)

MRA is incorrect. MRA is considered post CT and lumbar puncture to further investigate for
potential sites of bleeding/vascular malformations.

D Observation

Observation is incorrect. Observation alone isn’t appropriate here because approximately 2%


of SAHs may be missed if a CT scan alone is performed.

E Ramipril

Ramipril is incorrect. Like amlodipine, ramipril isn’t indicated here because there is no
indication that control of blood pressure from a baseline systolic of 178 mmHg has any
positive impact and in some cases, may actually increase the risk of neurological deficit.
51436

Rate this question:

Next Question

Previous Question Tag Question

Feedback End Review

Difficulty: Average

Peer Responses %

Q. Answered Flagged
Q. Answered Flagged

Q1

Q2

Q3

Q4

Q5

Q6

Q7

Q8

0:02:24/03:00:00

A 26-year-old woman presents to the Rheumatology Clinic with a new diagnosis of


rheumatoid arthritis (RA). Her sister, mother and aunt also suffer from RA.
Which of the following human leukocyte antigen (HLA) types is most likely to be found in
this patient?

A B1502

B B27

C B5701

D DR2

E DR4

Explanation 

E DR4

DR4 is strongly associated with rheumatoid arthritis which runs in family groups and is
therefore the most appropriate answer here. MHC class II antigens like DR4 are involved in
the presentation of antigen to T cells, and different DR subtypes are associated with different
rheumatological diseases. DR4 is also associated with pemphigus, drug-induced lupus
erythematosus, immunoglobulin A (IgA) nephropathy and polymyalgia rheumatica.

A B1502

B1502 is incorrect. The presence of B1502 is associated with the development of Stevens-
Johnson syndrome in response to carbamazepine use in the Han Chinese population. This has
led to a warning and precautions addition to the datasheet.

B B27

B27 is incorrect. B27 has a well-known association with seronegative arthritis, ankylosing
spondylitis, anterior uveitis and inflammatory bowel disease.
C B5701

B5701 is incorrect. The presence of B5701 is associated with the development of


hypersensitivity to abacavir, used in the treatment of human immunodeficiency virus (HIV)
and has prompted genetic screening at baseline in patients prescribed the agent.

D DR2

DR2 is incorrect. DR2 is associated with anti-glomerular basement membrane (GBM) disease,
systemic lupus erythematosus (SLE), multiple sclerosis, primary biliary cirrhosis and
autoimmune hepatitis.
51400

Rate this question:

Next Question

Previous Question Tag Question

Feedback End Review

Difficulty: Average

Peer Responses %

Q. Answered Flagged

Q1

Q2

Q3

Q4

Q5
Q. Answered Flagged

Q6

Q7

Q8

0:02:24/03:00:00

A 22-year-old man comes to the Rheumatology Clinic for review with symptoms of lumbar
spine pain and stiffness for up to 2 h each morning and plantar fasciitis. The symptoms are
only partially relieved by regular ibuprofen, and he is finding it difficult to hold down his job
as an electrician. Examination confirms limited lateral and forward flexion of the spine. X-ray
of the hips and pelvis reveals early sacroiliitis.
Which of the following is the most appropriate next step?

A Azathioprine

B Golimumab

C Methotrexate

D Naproxen

E Prednisolone

Explanation 

D Naproxen

The most likely diagnosis is ankylosing spondylitis (AS). In AS, the recommendation is to trial
a more potent non-steroidal anti-inflammatory drug (NSAID) in patients where symptoms are
not controlled by an agent such as ibuprofen. After this point, the next step is to move to a
tumour necrosis factor (TNF)-modulating biological. Treatment should, of course, be
supplemented with intensive physiotherapy.

A Azathioprine

Azathioprine is incorrect. Although azathioprine is effective in the treatment of patients with


severe AS as adjunctive therapy, anti-TNF-based therapies are thought to be potentially more
effective. This drives azathioprine as a choice for patients where disease is not controlled by
an anti-TNF based intervention, or where anti-TNF therapy is not tolerated.

B Golimumab
Golimumab is incorrect. Golimumab is an effective anti-TNF therapy for AS, although it
should be introduced after a trial of at least two NSAIDs given at appropriate doses.
Infliximab isn’t recommended by National Institute for Clinical Excellence (NICE) guidelines,
although other anti-TNF-based therapies are.

C Methotrexate

Methotrexate is incorrect. Methotrexate is the intervention of choice for psoriatic and


rheumatoid arthritis; the response to anti-TNF-based therapy of AS drives this class as the
preferred option.

E Prednisolone

Prednisolone is incorrect. Corticosteroids may improve short-term flares of AS but shouldn’t


be used as a long-term therapy because of the benefit-risk profile they carry with respect to
glucose tolerance, weight gain and bone mineral density over the longer term.
51455

Rate this question:

Next Question

Previous Question Tag Question

Feedback End Review

Difficulty: Average

Peer Responses %

Q. Answered Flagged

Q1

Q2
Q. Answered Flagged

Q3

Q4

Q5

Q6

Q7

Q8 
0:02:24/03:00:00

A 74-year-old woman is diagnosed with type 2 diabetes having suffered from nocturia and
lethargy over the past few months. She has a history of obesity and hypertension and has
been managed by the nurse at her GP practice for weight loss advice. Medication includes
lisinopril and indapamide. Examination reveals a blood pressure of 149/80 mmHg, pulse is 64
bpm and regular. She is obese with a body mass index of 34 kg/m 2.
Investigations:

Hb 129 g/l

WCC 7.0 × 10 9/l

PLT 204 × 10 9/l

Na + 139 mmol/l

K+ 4.3 mmol/l

Cr 138 µmol/l (estimated glomerular filtration rate (eGFR) 55 ml/min)

Glucose 9.8 mmol/l

HbA1c 60 mmol/mol

Which of the following is the initial therapy of choice?

A Gliclazide

B Insulin glargine

C Linagliptin

D Metformin

E Pioglitazone

Explanation 

D Metformin
This patient is elderly and at risk of hypoglycaemia; she is also obese. Taking these factors
into account, metformin is the most appropriate intervention to control blood sugar. It
reduces hepatic glucose output and therefore does not lead to hypoglycaemia when used as
monotherapy. It promotes modest weight loss and is associated with cardiovascular
protection. It can be used in patients with a glomerular filtration rate (GFR) as low as 45
ml/min with dosing unchanged; between 30–45ml/min, the dose should be reduced. It should
not be used in patients with an eGFR < 30 ml/min.

(see External Links)

A Gliclazide

Gliclazide is incorrect. Gliclazide is a sulphonylurea; it is associated with weight gain and


increased risk of hypoglycaemia, two aspects of intervention which are best avoided in this
elderly patient.

B Insulin glargine

Insulin glargine is incorrect. Insulin promotes weight gain and hypoglycaemia and in this
patient, is best avoided until alternative oral therapy options have been exhausted.

C Linagliptin

Linagliptin is incorrect. Linagliptin is a dipeptidyl peptidase-4 (DPP-IV) inhibitor. It leads to an


increase in endogenous incretin levels (glucagon-like peptide 1 and gastric inhibitory
polypeptide). It may be used in patients with renal impairment and reduced where there are
lower GFRs than that seen here and is an option if metformin alone is ineffective or if renal
disease progresses.

E Pioglitazone

Pioglitazone is incorrect. Pioglitazone is a glitazone. As a class, glitazones are effective at


achieving glucose control without increasing the risk of hypoglycaemia, although they
promote weight gain and loss of bone mineral density, both of which should be avoided here.
51353

Rate this question:

Next Question

Previous Question Tag Question

Feedback End Review


Difficulty: Average

Peer Responses %

Q. Answered Flagged

Q1

Q2

Q3

Q4

Q5

Q6

Q7

Q8

Q9

 External Links

Algorithm for blood glucose lowering therapy in adults with type 2 diabetes
nice.org.uk/guidance/ng28/resources/algorithm-for-blood-glucose-lowering-therapy-in-adults-with-type-2-diabetes-…
(https://www.nice.org.uk/guidance/ng28/resources/algorithm-for-blood-glucose-
lowering-therapy-in-adults-with-type-2-diabetes-pdf-2185604173)
0:02:24/03:00:00

A 52-year-old man comes to the clinic for review. He has suffered an inferior myocardial
infarction for which he received two stents some three months earlier. He has type 2 diabetes
controlled with metformin 1 g twice a day and hypertension. He takes a range of medications
for cardiovascular risk including lisinopril, atenolol, atorvastatin, eplerenone, aspirin and
ticagrelor. His main complaint is of erectile dysfunction since his admission to hospital. His
blood pressure is 122/84 mmHg and his pulse is 60 bpm and regular at rest.
Which of the following is the most likely cause of his erectile dysfunction?

A Atenolol

B Atorvastatin

C Eplerenone

D Lisinopril

E Ticagrelor

Explanation 

A Atenolol

Atenolol, like others in the beta-blocker class is associated with erectile dysfunction. Rather
than stopping the beta-blocker, which is associated with significant outcome benefits in this
patient population, a phosphodiesterase-5 inhibitor (e.g. sildenafil) could be instigated
assuming that symptoms of cardiac ischaemia are stable.

(see External Links)

B Atorvastatin

Atorvastatin is incorrect. A Cochrane meta-analysis of statin trials has recorded no negative


impact of statin therapy on erectile dysfunction. Across the meta-analysis a trend towards
improvement in erectile function scores was seen for statin vs no statin subjects.

C Eplerenone
Eplerenone is incorrect. Spironolactone, not eplerenone, is recognised to interfere with
androgen metabolism. It is a well-known cause of both gynaecomastia and erectile
dysfunction. Eplerenone is much more specific for the aldosterone receptor.

D Lisinopril

Lisinopril is incorrect. Angiotensin-converting enzyme inhibitors are not associated with


disordered androgen metabolism, nor with vascular effects which impact on erectile function.

E Ticagrelor

Ticagrelor is incorrect. Ticagrelor is a P2Y12-adenosine diphosphate receptor inhibitor used in


high-risk individuals as an antiplatelet agent post-myocardial infarction. It does not cause
erectile dysfunction.
51368

Rate this question:

Next Question

Previous Question Tag Question

Feedback End Review

Difficulty: Average

Peer Responses %

Q. Answered Flagged

Q1

Q2

Q3
Q. Answered Flagged

Q4

Q5

Q6

Q7

Q8 

 External Links

Effect of different beta blockers on penile vascular velocities in hypertensive male


sciencedirect.com/science/article/pii/S0378603X1500073X
(https://www.sciencedirect.com/science/article/pii/S0378603X1500073X)
0:02:24/03:00:00

A 26-year-old woman who is 24 weeks’ pregnant with her second child comes to the
Emergency Department complaining of left-sided pleuritic chest pain of sudden onset 24 h
earlier. She has had a mild viral illness over the past few days which has caused her to rest
and she has developed a swollen left lower limb. Examination reveals a blood pressure of
105/80 mmHg, pulse is 85 bpm and regular. O 2 saturation is 92% on air. Auscultation of the
chest is clear. Abdominal examination is consistent with a 24-week pregnancy. The left calf is
larger than the right, with prominent varicosities and minor pitting oedema. A chest X-ray is
normal. Electrocardiogram (ECG) reveals a right ventricular strain pattern.

Which of the following is the optimal next investigation?

A Arterial-blood gases (ABGs)

B CT pulmonary angiogram (CTPA)

C Compression duplex ultrasound of the left leg

D Echocardiogram

E Ventilation-perfusion (VQ) scan

Explanation 

C Compression duplex ultrasound of the left leg

This patient’s presentation is suspicious of a pulmonary embolism, related to her period of


inactivity and pregnancy. In this situation, where there are obvious signs of a lower limb deep
vein thrombosis (DVT), the Royal College of Obstetricians and Gynaecologists green-top
guidance recommends ultrasound of the leg, with no further investigation necessary in the
event that a DVT is confirmed.

(see External Links)

A Arterial-blood gases (ABGs)

ABGs is incorrect. Although an ABG may reveal an increased A–a gradient, raising suspicion
of a pulmonary embolism, it isn’t useful in confirming the diagnosis of thromboembolic
disease.
B CT pulmonary angiogram (CTPA)

CTPA is incorrect. CTPA could be considered according to the guidelines in the event that
there is no reason to suspect or investigate for lower limb DVT. There are pros and cons of
CTPA versus VQ scanning, with CTPA delivering a larger radiation dose to breast tissue than
a VQ scan.

D Echocardiogram

Echocardiogram is incorrect. Echocardiogram may be useful in the acute situation, where it


can demonstrate right ventricular dilatation in the event of massive pulmonary embolism.
Here, with a relatively stable patient, ultrasound of the lower limb is more appropriate.

E Ventilation-perfusion (VQ) scan

VQ scan is incorrect. A VQ scan may be indicated, like CTPA, if there is no evidence of lower
limb thrombosis. In the event that VQ is offered, it is associated with a larger potential
radiation dose to the fetus than CTPA and risks and benefits should be discussed with the
mother if possible.
51355

Rate this question:

Next Question

Previous Question Tag Question

Feedback End Review

Difficulty: Average

Peer Responses %

Q. Answered Flagged
Q. Answered Flagged

Q1

Q2

Q3

Q4

Q5

Q6

Q7

Q8 

 External Links

Thromboembolic Disease in Pregnancy and the Puerperium: Acute Management


rcog.org.uk/globalassets/documents/guidelines/gtg-37b.pdf
(https://www.rcog.org.uk/globalassets/documents/guidelines/gtg-37b.pdf)
0:02:24/03:00:00

A 45-year-old woman presents to the Psychiatry Clinic for review. She has suffered from
multiple medical problems over the past three years including irritable bowel syndrome,
proximal arm and leg pains, chronic backache and reflux oesophagitis. Blood work up,
endoscopy, colonoscopy and magnetic resonance imaging (MRI) lumbar spine have all
proved unremarkable. She is unable to work because of the symptoms and currently claims
multiple sickness benefits. Examination reveals a body mass index of 31 kg/m 2 and she has
gained 6 kg over the past six months.
Which of the following is the most appropriate next step?

A Cognitive behavioural therapy (CBT)

B Diazepam

C Repeat endoscopy, colonoscopy and MRI scan

D Risperidone

E Sertraline

Explanation 

A Cognitive behavioural therapy (CBT)

This patient has somatic symptom disorder, characterised by one or more somatic symptoms
that are accompanied by excessive thoughts, feelings and/or behaviours related to the
symptoms. The weight gain and inability to work are consistent with the diagnosis. CBT is the
obvious initial intervention of choice.

B Diazepam

Diazepam is incorrect. Diazepam is sedating and addictive, although it may impact over the
short term on anxiety; over the longer term, it will only have a negative impact on outcomes.

C Repeat endoscopy, colonoscopy and MRI scan


Repeat endoscopy, colonoscopy and MRI scan is incorrect. Further investigations should be
avoided if possible, as they can reinforce illness behaviour and increase the impetus to seek
further diagnostic testing.

D Risperidone

Risperidone is incorrect. Risperidone may be of value in patients with somatic symptom


disorder who have symptoms which are resistant both to CBT and to conventional
antidepressants.

E Sertraline

Sertraline is incorrect. Selective serotonin reuptake inhibitors (SSRIs) are effective in treating
somatic symptom disorder when used in combination with CBT. It is CBT however that forms
the cornerstone of initial therapy, rather than reaching for pharmacotherapy in the first
instance.
51444

Rate this question:

Next Question

Previous Question Tag Question

Feedback End Review

Difficulty: Average

Peer Responses %

Q. Answered Flagged

Q1

Q2
Q. Answered Flagged

Q3

Q4

Q5

Q6

Q7

Q8 
0:02:24/03:00:00

A 70-year-old man comes to the Gastroenterology Clinic for review. He has suffered from
intermittent dysphagia to both solids and occasionally liquids, for up to a year or more, with
occasional regurgitation of food. He takes amlodipine for hypertension and a salbutamol
inhaler. His weight has remained stable over the past 12 months. His abdomen is soft and
non-tender with no masses; his body mass index is 29.5 kg/m 2.
Investigations:

Hb 135 g/l

WCC 6.2 × 10 9/l

PLT 199 × 10 9/l

Na + 142 mmol/l

K+ 4.9 mmol/l

Cr 102 µmol/l

Barium Swallow 'bird's beak' appearance of the distal oesophagus

Which of the following is the most likely diagnosis?

A Achalasia

B Candida oesophagitis

C Gastro-oesophageal reflux disease (GORD)

D Gastric cancer

E Oesophageal carcinoma

Explanation 

A Achalasia

The striking features here are weight maintenance and lack of iron deficiency anaemia, both
of which count against a diagnosis of malignancy. The chronic history here, with intermittent
dysphagia, is much more in keeping with achalasia, linked to a failure of the lower
oesophageal sphincter to relax, and diagnosed approximately 90% of the time by manometry
On barium swallow it is classically described as a “bird beak” appearance. Systemic calcium
channel blockers or nitrates may be effective in relieving symptoms in a limited number of
patients, although local therapies including dilatation and botulinum toxin have a higher
success rate.

B Candida oesophagitis

Candida oesophagitis is incorrect. Candida oesophagitis is associated with


immunosuppression and with inappropriate use of inhaled corticosteroids, neither of which
are obvious causes for this patient’s symptoms.

C Gastro-oesophageal reflux disease (GORD)

GORD is incorrect. GORD is associated with retrosternal chest pain, epigastric pain and water
brash, particularly at night. It is not associated with the barium swallow features seen here.

D Gastric cancer

Gastric cancer is incorrect. Although obstruction of the lower oesophageal sphincter can
occur with gastric cancer, the lack of weight loss or anaemia both count against a cancer
diagnosis as the likely cause of this patient’s symptoms.

E Oesophageal carcinoma

Oesophageal carcinoma is incorrect. The lack of weight loss or anaemia with a prolonged
period of symptoms counts against oesophageal carcinoma. Symptoms of dysphagia are also
usually progressive, rather than the intermittent picture seen here.
51354

Rate this question:

Next Question

Previous Question Tag Question

Feedback End Review

Difficulty: Average

Peer Responses %
Q. Answered Flagged

Q1

Q2

Q3

Q4

Q5

Q6

Q7

Q8

Q9

0:02:24/03:00:00

A 49-year-old woman who suffers from type 1 diabetes comes to the Emergency Department
because of double vision when looking to the left. Examination in the Emergency Department
reveals a left lateral rectus palsy.
Which of the following is the most likely cause of her symptoms?

A Facial nerve palsy

B Oculomotor nerve palsy

C Optic nerve palsy

D Trigeminal nerve palsy

E Abducens nerve palsy

Explanation 

E Abducens nerve palsy

The lateral rectus muscle is supplied by the sixth cranial nerve (abducens). Mononeuritis is
relatively common in patients with type 1 diabetes and is the likely cause here. Nerve function
may improve over the period of a few weeks.

A Facial nerve palsy

Facial nerve palsy is incorrect. The seventh nerve doesn’t supply the ocular muscles; it is
responsible for motor supply to facial muscles and can also lead to decreased taste
sensation, hyperacusis and decreased salivation.

B Oculomotor nerve palsy

Oculomotor nerve palsy is incorrect. The oculomotor nerve supplies all of the eye muscles
apart from the superior oblique and the lateral rectus. Oculomotor nerve palsy therefore
results in unopposed action of the lateral rectus and superior oblique, which leads to the eye
looking ‘down and out’.
C Optic nerve palsy

Optic nerve palsy is incorrect. Optic nerve palsy leads to visual field loss and loss of colour
vision or acuity rather than to paralysis of ocular muscles.

D Trigeminal nerve palsy

Trigeminal nerve palsy is incorrect. Trigeminal nerve palsy results in sensory loss over the
face rather than paralysis of any of the muscles controlling eye movements.
51395

Rate this question:

Next Question

Previous Question Tag Question

Feedback End Review

Difficulty: Average

Peer Responses %

Q. Answered Flagged

Q1

Q2

Q3

Q4

Q5

Q6
Q. Answered Flagged

Q7

Q8

0:02:24/03:00:00

A 45-year-old woman presents to the Emergency Department with sudden deterioration in


the vision in her right eye from waking that morning. She has type 1 diabetes mellitus and
suffered a deep vein thrombosis some 10 years ago. She takes no regular medication apart
from insulin and ramipril. She is anxious, with a pulse of 90 bpm and regular, and a blood
pressure of 155/90 mmHg. Fundoscopy of the right eye reveals multiple flame haemorrhages
over the whole fundus; she is able to perceive light and dark only with the right eye, and
vision in the left eye is 6/12.
Which of the following is the most likely diagnosis?

A Central retinal artery occlusion

B Central retinal vein occlusion

C Macular oedema

D Retinal detachment

E Vitreous haemorrhage

Explanation 

B Central retinal vein occlusion

Central retinal vein occlusion is seen in patients with thrombophilia (in keeping with the
previous deep vein thrombosis) and in patients with diabetes mellitus. It fits with the
generalised flame haemorrhages seen here and the marked deterioration in visual acuity.
Sectoral visual field loss due to branch retinal vein occlusion is also seen. No treatments have
proven benefit currently; regular ophthalmological follow up is designed to look for
complications of neovascularisation is essential.

A Central retinal artery occlusion

Central retinal artery occlusion is incorrect. Central retinal artery occlusion is associated with
sudden visual field loss, but is associated with a pale retina and a ‘cherry red’ spot due to
continued arterial perfusion of a small area via the ciliary artery.
C Macular oedema

Macular oedema is incorrect. Macular oedema is associated with a deterioration in visual


acuity, although the appearance is different, with oedema over the area of the macular, and
changes consistent with retinal ischaemia such as hard exudates and dot/blot haemorrhages
close to the macula.

D Retinal detachment

Retinal detachment is incorrect. Retinal detachment is associated with sudden, marked visual
field loss, although clear detachment of the retina, accompanied by a confluent area of
haemorrhage is usually seen.

E Vitreous haemorrhage

Vitreous haemorrhage is incorrect. Vitreous haemorrhage is associated with decreased visual


acuity, although obscuration of the retina occurs because of the haemorrhage within the
vitreous, rather than the features seen here on fundoscopy.
51406

Rate this question:

Next Question

Previous Question Tag Question

Feedback End Review

Difficulty: Average

Peer Responses %

Q. Answered Flagged
Q. Answered Flagged

Q1

Q2

Q3

Q4

Q5

Q6

Q7

Q8
0:02:24/03:00:00

A 45-year-old woman comes to the Emergency Department with a severe sore throat and
fevers which have developed over the past 48 h. She knew to attend urgently because she is
taking carbimazole for the treatment of Graves’ disease. On examination, she is pyrexial at
38.8 °C, blood pressure is 105/85 mmHg, pulse is 94 bpm and regular. You note a severe
pharyngitis with pus over the tonsils. A throat swab was sent by her GP some two days
earlier.
Investigations:

Hb 125 g/l

WCC 4.9 × 10 9/l (Neutrophils 0.1 x 10 9/l)

PLT 191 × 10 9/l

Na + 140 mmol/l

K+ 4.4 mmol/l

Cr 132 µmol/l

Throat swab positive for Streptococcus pyogenes.

She is commenced on antibiotics and the carbimazole is stopped.


Which of the following is the most appropriate next step?

A Deferiprone

B Switching to propylthiouracil

C Granulocyte-colony stimulating factor (G-CSF)

D Observation

E Prednisolone

Explanation 

C Granulocyte-colony stimulating factor (G-CSF)


In patients with significant bacterial sepsis and neutropaenia (in this case, related to
carbimazole), G-CSF carries a positive benefit-risk profile. A dose of 4–10 µg/kg per day is
usually instigated. Use of G-CSF is associated with shorter antibiotic use and shorter length
of hospital stay versus current or historical controls.

(see External Links)

A Deferiprone

Deferiprone is incorrect. Deferiprone is an iron-chelating agent rather than a treatment for


neutropaenia. It is actually a cause of drug-induced neutropaenia.

B Switching to propylthiouracil

Switching to propylthiouracil is incorrect. Propylthiouracil also causes agranulocytosis, hence


switching from one anti-thyroid agent to the other is not appropriate here.

D Observation

Observation is incorrect. In this case, with significant infection, observation is likely to be


inferior to use of G-CSF, leading to a greater morbidity and a longer hospital stay.

E Prednisolone

Prednisolone is incorrect. Corticosteroids do not have a role in the treatment of drug-induced


neutropaenia. In this case, use of steroids may also result in more severe consequences of
streptococcal infection.
51391

Rate this question:

Next Question

Previous Question Tag Question

Feedback End Review

Difficulty: Average

Peer Responses %
Q. Answered Flagged

Q1

Q2

Q3

Q4

Q5

Q6

Q7

Q8

Q9

 External Links

Neupogen Singleject 30 MU (0.6 mg/ml)


medicines.org.uk/emc/product/608/smpc
(https://www.medicines.org.uk/emc/product/608/smpc)
0:02:24/03:00:00

A 45-year-old patient comes to the Oncology Clinic for follow up post-chemotherapy. She
has been treated with a range of chemotherapeutic agents for colon cancer. Her main
complaint is of numbness, and pins and needles affecting both feet. You confirm peripheral
sensory neuropathy on neurological examination of both lower limbs.
Which of the following agents is the most likely cause of her peripheral neuropathy?

A Bleomycin

B Cisplatin

C Cyclophosphamide

D Pembrolizumab

E Trastuzumab

Explanation 

B Cisplatin

Peripheral neuropathy is caused by the range of platinum-based chemotherapies, including


cisplatin, oxaliplatin and carboplatin. Both short-term effects are seen at the time of infusion,
and long-term peripheral nerve damage is recognised. This is in contrast to pembrolizumab,
where reports of peripheral neuropathy are much rarer, and frequency is listed as uncommon.
(see External Links)

A Bleomycin

Bleomycin is incorrect. Bleomycin-induced pneumonitis is seen in approximately 10% of


patients who use the drug. It can be life-threatening and unrecognised can progress to
pulmonary fibrosis. Early recognition of symptoms of pneumonitis is crucial in managing the
condition. Bleomycin is a more usual component of treatments for haematological
malignancy, rather than sold tumours.

C Cyclophosphamide
Cyclophosphamide is incorrect. Cyclophosphamide is a cause of haemorrhagic cystitis and is
associated with increased risk of bladder neoplasm. For this reason, it is given with MESNA to
protect against cystitis occurring.

D Pembrolizumab

Pembrolizumab is incorrect. Pembrolizumab binds the programmed cell-death 1 (PD-1)


receptor, and is an immune-checkpoint inhibitor; this leads to an increased risk of
autoimmune disease, with patients prescribed pembrolizumab reporting increased rates of
type 1 diabetes and autoimmune thyroid disease. Peripheral neuropathy is reported in the
prescribing information as uncommon in association with pembrolizumab therapy.

(see External Links)

E Trastuzumab

Trastuzumab is incorrect. Trastuzumab is a human epidermal growth factor receptor 2


(HER2) antagonist. It is associated with the development of dilative cardiomyopathy rather
than peripheral neuropathy, and regular monitoring of left ventricular (LV) function is advised
in patients for whom it has been prescribed. Peripheral neuropathy has been reported in
patients taking trastuzumab, although it’s frequency is not known, (ie it is rare).
51392

Rate this question:

Next Question

Previous Question Tag Question

Feedback End Review

Difficulty: Average

Peer Responses %
Q. Answered Flagged

Q1

Q2

Q3

Q4

Q5

Q6

Q7

Q8

Q9

 External Links

Cisplatin 1 mg/ml Sterile Concentrate


medicines.org.uk/emc/product/3788/smpc
(https://www.medicines.org.uk/emc/product/3788/smpc)

KEYTRUDA® undesirable effects


medicines.org.uk/emc/product/6947/smpc#UNDESIRABLE_EFFECTS
(https://www.medicines.org.uk/emc/product/6947/smpc#UNDESIRABLE_EFFECTS)
0:02:24/03:00:00

A 25-year-old woman who is treated with an insulin pump for type 1 diabetes comes to the
clinic for review. She is treated with ramipril 5 mg daily for microalbuminuria and announces
that she is 8 weeks’ pregnant. Her blood pressure is 100/60 mmHg. Urine is 1+ for protein.
Which of the following is the most appropriate next step?

A Continue ramipril

B Stop ramipril and start diltiazem

C Stop ramipril and start amlodipine

D Stop ramipril and start methyldopa

E Stop ramipril and observe blood pressure

Explanation 

E Stop ramipril and observe blood pressure

Ramipril is associated with birth defects such as renal tract abnormalities in the offspring
when prescribed for pregnant women. As such, it should not be continued. Ideally, switching
to a more appropriate agent such as labetalol or methyldopa is preferred for the treatment of
hypertension at the point of pre-pregnancy counselling. In this case, with normal blood
pressure, an alternative anti-hypertensive isn’t required.

A Continue ramipril

Continue ramipril is incorrect. Continuing the ramipril puts this patient’s child at significant
risk of renal tract abnormalities; it should be stopped as soon as the pregnancy is confirmed.

B Stop ramipril and start diltiazem

Stop ramipril and start diltiazem is incorrect. Diltiazem has limited evidence supporting its
use in reducing albuminuria. However, here, there is no urgency to start a further therapy,
particularly as it may be poorly tolerated.
C Stop ramipril and start amlodipine

Stop ramipril and start amlodipine is incorrect. Dihydropyridine calcium antagonists are well
tolerated in pregnancy, although there is no evidence in this case that an alternative anti-
hypertensive is required.

D Stop ramipril and start methyldopa

Stop ramipril and start methyldopa is incorrect. Methyldopa is the second-line therapy of
choice for patients with hypertension in pregnancy, although the normal blood pressure
means that it isn’t required here.
51356

Rate this question:

Next Question

Previous Question Tag Question

Feedback End Review

Difficulty: Average

Peer Responses %

Q. Answered Flagged

Q1

Q2

Q3

Q4

Q5
Q. Answered Flagged

Q6

Q7

Q8

0:02:24/03:00:00

A 43-year-old woman presents to the clinic with Raynaud’s phenomenon and has skin
thickening with areas of calcinosis affecting her hands and feet, with areas of digital
ulceration affecting her toes. You also notice some telangiectasias around her lips, although
microstomia appears not to be present. Blood workup is unremarkable with normal renal
function. Pulmonary function tests are in the normal range for her age.
Which of the following auto-antibodies is most likely to be present?

A Anti-centromere

B Anti-fibrillarin

C Anti-RNA polymerase 3

D Anti-topoisomerase 1

E Anti-U11 RNP

Explanation 

A Anti-centromere

This patient has limited cutaneous systemic sclerosis as indicated by the lack of skin disease
above the knees or elbows. This means that anti-centromere antibodies are more likely to be
found than the other options because they are all associated with visceral organ involvement.
Patients with limited cutaneous systemic sclerosis are less likely to get renal impairment or
pulmonary fibrosis than those with more extensive skin involvement.

B Anti-fibrillarin

Anti-fibrillarin is incorrect. Anti-fibrillarin antibodies are associated with cardiac and


pulmonary fibrosis, renal impairment, and myositis in patients with systemic sclerosis. This is
not suggested by the limited skin disease seen here.

C Anti-RNA polymerase 3
Anti-RNA polymerase 3 is incorrect. This auto-antibody is associated with diffuse systemic
sclerosis and is found in increased frequency where patients have scleroderma renal crisis. It
appears to be relatively protective against pulmonary fibrosis.

D Anti-topoisomerase 1

Anti-topoisomerase 1 is incorrect. Anti-topoisomerase 1 is strongly associated with pulmonary


fibrosis in systemic sclerosis, which is more frequently seen in patients with diffuse skin
disease rather than the limited picture seen here.

E Anti-U11 RNP

Anti-U11 RNP is incorrect. Anti-U11 RNP antibodies are found in both limited and diffuse
cutaneous disease related to systemic sclerosis, although they are associated with pulmonary
fibrosis, which doesn’t appear to be a feature of this patient’s disease.
51456

Rate this question:

Next Question

Previous Question Tag Question

Feedback End Review

Difficulty: Average

Peer Responses %

Q. Answered Flagged

Q1

Q2
Q. Answered Flagged

Q3

Q4

Q5

Q6

Q7

Q8
0:02:24/03:00:00

A 23-year-old woman presents to the Dermatology Clinic with an intensely itchy rash
characterised by small blisters and vesicles particularly affecting her elbows, her hairline and
her buttocks. The severity waxes and wanes from week to week. She has iron deficiency
anaemia treated by her GP with iron tablets but is otherwise well. You confirm the presence
of clusters of small vesicles and blisters, particularly over the extensor surfaces of the limbs,
scalp and buttocks.
Which of the following is the most likely diagnosis?

A Crohn’s disease

B Dermatitis herpetiformis

C Linear immunoglobulin A (IgA) bullous dermatosis

D Papular urticaria

E Psoriasis

Explanation 

B Dermatitis herpetiformis

The distribution of the rash, with small vesicles, is typical for dermatitis herpetiformis, and the
iron deficiency anaemia here raises the possibility of co-existent coeliac disease, where some
patients may not suffer obvious gastrointestinal upset. Skin biopsy to show granular IgA
deposits is used to confirm the diagnosis and avoidance of gluten in the diet is the mainstay
of therapy. Anti-tissue transglutaminase (TTG) antibodies should also be assayed.

A Crohn’s disease

Crohn’s disease is incorrect. Crohn’s disease may be associated with iron deficiency anaemia,
although it isn’t associated with the features of dermatitis herpetiformis seen here, and more
severe gastrointestinal (GI) upset would be expected.

C Linear immunoglobulin A (IgA) bullous dermatosis


Linear IgA bullous dermatosis is incorrect. Blisters associated with linear IgA bullous
dermatosis tend to be arranged like a ‘string of pearls’ and presents with lesions affecting the
trunk and limbs rather than the buttocks. It isn’t associated with iron deficiency anaemia, and
IgA deposition occurs along the basement membrane, and 50% of patients with the condition
have blisters around the mouth and lips.

D Papular urticaria

Papular urticaria is incorrect. Papular urticaria is much more transient, with lesions only
lasting a few hours, where a trigger can often be identified. Lesions do exhibit
dermatographism, where new wheals are formed when the skin is scratched.

E Psoriasis

Psoriasis is incorrect. Psoriasis exists as an erythematous scaling rash on extensor surfaces,


rather than the individual vesicles seen here. Only when the areas of dermatitis herpetiformis
rash become confluent is there usually diagnostic confusion versus psoriasis.
51409

Rate this question:

Next Question

Previous Question Tag Question

Feedback End Review

Difficulty: Average

Peer Responses %

Q. Answered Flagged

Q1
Q. Answered Flagged

Q2

Q3

Q4

Q5

Q6

Q7

Q8
0:02:24/03:00:00

A 53-year-old man comes to the Dermatology Clinic with discoloured thickened skin
affecting his armpits, groin and the back of his neck. When you examine him, you note areas
of dark, almost black pigmented skin with a velvety texture, particularly affecting the armpits
and the back of the neck. He has a history of indigestion lasting for six months or more,
smokes 20 cigarettes per day and has suffered from back pain for the two years.
Which of the following tumours is most likely to be associated with his underlying skin
condition?

A Adenocarcinoma of the bronchus

B Bronchial carcinoid

C Gastric adenocarcinoma

D Pancreatic adenocarcinoma

E Squamous cell carcinoma of the oesophagus

Explanation 

C Gastric adenocarcinoma

The skin rash described here is highly suggestive of a diagnosis of acanthosis nigricans.
Gastric adenocarcinoma is responsible for around 55% of cases of acanthosis and is the
commonest cause amongst solid tumours. Other cancers associated with acanthosis
nigricans include hepatocellular carcinoma and adenocarcinomas of the lung, ovary,
endometrium, kidneys, pancreas, bladder and breast.

A Adenocarcinoma of the bronchus

Adenocarcinoma of the bronchus is incorrect. Although acanthosis nigricans is recognised as


a presenting feature of bronchial carcinoma, it’s much rarer than with gastric cancer.
Although this patient has a history of smoking, it’s gastric cancer rather than bronchial
carcinoma that should be excluded first.

B Bronchial carcinoid
Bronchial carcinoid is incorrect. Bronchial carcinoid usually occurs in a younger age group
than this patient and isn’t associated with smoking. It is associated with pellagra rather than
the acanthosis nigricans-type rash seen here.

D Pancreatic adenocarcinoma

Pancreatic adenocarcinoma is incorrect. Acanthosis nigricans is seen in patients with


pancreatic adenocarcinoma but more rarely than in gastric cancer. Necrolytic migratory
erythema is seen in patients with glucagonoma.

E Squamous cell carcinoma of the oesophagus

Squamous cell carcinoma of the oesophagus is incorrect. Adenocarcinoma of the


oesophagus is associated with acanthosis nigricans, although more rarely than gastric cancer.
Squamous cell carcinoma is associated with tylosis, a hereditary hyperkeratotic skin
condition.
51381

Rate this question:

Next Question

Previous Question Tag Question

Feedback End Review

Difficulty: Average

Peer Responses %

Q. Answered Flagged

Q1

Q2
Q. Answered Flagged

Q3

Q4

Q5

Q6

Q7

Q8 
0:02:24/03:00:00

A 23-year-old man is evaluated in the Cardiology Clinic following a second collapse in the last
six months. On this occasion, he was seen by a passer-by to collapse whilst jogging and the
bystander reported that she couldn’t feel a pulse for a few seconds. His father died suddenly
at the age of 49. Examination reveals a blood pressure of 142/87 mmHg, pulse is 83 bpm and
regular; you note a double apex beat and a systolic murmur loudest at the left sternal edge.
Investigations:

Hb 135 g/l

WCC 6.9 × 10 9/l

PLT 217 × 10 9/l

Na + 142 mmol/l

K+ 4.5 mmol/l

Cr 109 µmol/l

Septal wall thickening and evidence of outflow tract obstruction


Echocardiogram
consistent with hypertrophic obstructive cardiomyopathy

24 h tape Multiple premature ventricular ectopics, couplets and triplets

Which of the following is the most appropriate intervention?

A Amiodarone

B Bisoprolol

C Implantable cardioverter defibrillator (ICD)

D Myomectomy

E Verapamil

Explanation 

C Implantable cardioverter defibrillator (ICD)


This patient would be viewed as high risk, with two episodes of collapse, the second of which
may well have been due to an episode of ventricular tachycardia. In this situation, an ICD is
the most appropriate approach to reduce the risk of sudden death.

A Amiodarone

Amiodarone is incorrect. Although amiodarone may suppress episodes of ventricular


tachycardia (VT), it is less effective with respect to sudden death versus ICD implantation,
and risks significant long-term sequelae associated with amiodarone use.

B Bisoprolol

Bisoprolol is incorrect. Beta-blockers reduce the left ventricular outflow tract gradient and
may positively impact on symptoms of angina, but they don’t affect risk of sudden death due
to ventricular arrhythmias.

D Myomectomy

Myomectomy is incorrect. Myomectomy is the intervention where left ventricular outflow


tract symptoms fail to respond to medical therapy alone. Both alcohol injection and surgical
techniques are recognised.

E Verapamil

Verapamil is incorrect. Verapamil is an alternative medical intervention to bisoprolol for


symptoms of elevated left ventricular outflow tract gradient. Disopyramide is another
potential alternative to beta-blockade.
51369

Rate this question:

Next Question

Previous Question Tag Question

Feedback End Review

Difficulty: Average

Peer Responses %
Q. Answered Flagged

Q1

Q2

Q3

Q4

Q5

Q6

Q7

Q8

Q9

0:02:24/03:00:00

A 28-year-old man is seen in the Genetics Clinic with his partner. He has suffered recurrent
gastrointestinal bleeding from gastrointestinal polyps and is known to suffer from Peutz-
Jeghers syndrome. He wants to start a family and would like to know about the chances of
his offspring inheriting the condition. His partner has no significant medical illnesses.
Which of the following is the percentage chance of his children inheriting Peutz-Jeghers
syndrome?

A 0%

B 25%

C 50%

D 75%

E 100%

Explanation 

C 50%

Peutz-Jeghers syndrome occurs due to mutations in the STK11 gene, a known tumour
suppressor gene. Mutation leads to increased risk of gastrointestinal hamartomas and later,
to the development of gastrointestinal malignant tumours. It follows an autosomal dominant
inheritance pattern. As such, this patient has a 50% chance of passing on the condition to
both male and female offspring.

A 0%

0% is incorrect. Patients with Peutz-Jeghers do not have subfertility, and the condition is
autosomal dominant, meaning that there is a 50% chance of passing on the gene to offspring.
0% would be the risk of inheritance if Peutz-Jegher’s was a mitochondrial condition, hence
it’s not a relevant answer here.

B 25%
25% is incorrect. The 25% risk of offspring inheriting a genetic condition applies to autosomal
recessive conditions where both parents are carriers.

D 75%

75% is incorrect. The 75% risk fits with inheriting an autosomal dominant condition where
both the parents are known to suffer symptoms of the disease. This is impossible as the
partner has no medical illnesses. In individuals who carry both dominant genes, for example,
those who are homozygous for familial hypercholesterolaemia, symptoms are very severe,
and the homozygous condition may indeed not be survivable, resulting in spontaneous
abortion.

E 100%

100% is incorrect. The 100% risk fits with inheriting an autosomal recessive condition where
both of the parents are known to have the disease.
51364

Rate this question:

Next Question

Previous Question Tag Question

Feedback End Review

Difficulty: Average

Peer Responses %

Q. Answered Flagged

Q1

Q2
Q. Answered Flagged

Q3

Q4

Q5

Q6

Q7

Q8 
0:02:24/03:00:00

A 43-year-old woman who is 10 weeks’ pregnant with her first child comes to the Emergency
Department with an anxiety attack. She is concerned that her child may have Down
syndrome and wants to know about risks and benefits of a screening test for Down
syndrome which she has found on the internet.
Results of a screening test are shown below:

Screening test result Affected fetus Unaffected fetus Total

Positive 9 351 360

Negative 1 4449 4450

Totals 10 4800 4810

Which of the following is the positive predictive value for this Down screening test?

A 2.5%

B 7.3%

C 10%

D 92.7%

E 99.9%

Explanation 

A 2.5%

This is the proportion of those who screen positive who have an affected fetus (true positive
/ (true positive + false positive)). In this case, 9/360 = 2.5%, meaning that only 1 in 40 women
who screen positive go on to have an affected child. The potential for significant unwarranted
stress is therefore significant.

B 7.3%
7.3% is incorrect. The false positive fraction is 7.3%, the proportion of unaffected fetuses who
screen positive (ie 351/4800).

C 10%

10% is incorrect. The false negative fraction is 10% (1/10), meaning that 1 in 10 patients could
be carrying a Down’s-affected fetus and falsely reassured.

D 92.7%

92.7% is incorrect. This is the specificity, the proportion of unaffected fetuses who screen
negative (4449/4800).

E 99.9%

99.9% is incorrect. 99.9% is the negative predictive value, the proportion of those who screen
negative who have an unaffected fetus (this is 4449/4450).
51404

Rate this question:

Next Question

Previous Question Tag Question

Feedback End Review

Difficulty: Average

Peer Responses %

Q. Answered Flagged

Q1
Q. Answered Flagged

Q2

Q3

Q4

Q5

Q6

Q7

Q8 
0:02:24/03:00:00

You are examining the effects of a new treatment for allergic rhinitis which interferes with the
T-helper cell type 2 (Th2) response.
Which of the following cytokines is mainly produced by Th2 cells?

A Fibroblast growth factor-21 (FGF-21)

B Interferon gamma

C Interleukin-2 (IL-2)

D Interleukin-4 (IL-4)

E Tumour necrosis factor (TNF)-beta

Explanation 

D Interleukin-4 (IL-4)

IL-4 drives differentiation of naïve T-helper cells to Th2 cells. This occurs in a positive-
feedback loop where the cells produce more IL-4 once differentiated. IL-4 promotes
macrophage differentiation into M2 macrophages, induces B-cell class switching to
immunoglobulin E (IgE) production, downregulates production of classical M1-type
macrophages and interferon gamma.

A Fibroblast growth factor-21 (FGF-21)

FGF-21 is incorrect. FGF-21 is secreted by the liver and is a protein which stimulates glucose
uptake and may be protective against obesity. When over-expressed in transgenic models, it
is associated with reduced both glucose and triglycerides.

B Interferon gamma

Interferon gamma is incorrect. Interferon gamma is produced by Th1 differentiated T cells and
by NK cells. It is an important activator of macrophages, and inducer of class II major
histocompatibility complex (MHC) expression. It forms a crucial part of the innate and
adaptive immune response.
C Interleukin-2 (IL-2)

IL-2 is incorrect. IL-2 promotes T-cell differentiation. Where there is an antigenic stimulus, IL-2
drives differentiation of T-cells into memory and effector cells. Where there is no antigenic
stimulus, it is recognised to stimulate differentiation of T-cells into regulatory T-cells.

E Tumour necrosis factor (TNF)-beta

TNF-beta is incorrect. TNF-beta, like IL-2 and interferon gamma, is involved in the Th1
response. It is a driver of inflammation, particularly in response to viral infection.
51396

Rate this question:

Next Question

Previous Question Tag Question

Feedback End Review

Difficulty: Average

Peer Responses %

Q. Answered Flagged

Q1

Q2

Q3

Q4

Q5
Q. Answered Flagged

Q6

Q7

Q8 
0:02:24/03:00:00

A 23-year-old woman with a history of depression comes to the Dermatology Clinic for
review. She also suffers from irritable bowel syndrome. Examination of her forearms reveals
multiple linear erythematous lesions, each 2–2.5 cm in length. Current medication is
amitriptyline taken at night.
Which of the following is the most likely diagnosis?

A Dermatitis artefacta

B Dermatitis herpetiformis

C Erythema ab igne

D Erythema multiforme

E Erythema nodosum

Explanation 

A Dermatitis artefacta

The uniformity of the lesions seen here, their linear nature and the fact they are found on the
forearms, which are easily accessible, fits well with a diagnosis of dermatitis artefacta. The
disease is more common in patients with depression, who will usually deny that the rash is
self-induced.

B Dermatitis herpetiformis

Dermatitis herpetiformis is incorrect. Dermatitis herpetiformis is characterised by crops of


small blisters which are intensely itchy. They occur against a background of gluten
hypersensitivity and respond to avoidance of gluten in the diet.

C Erythema ab igne

Erythema ab igne is incorrect. Erythema ab igne is characterised by hyperpigmentation, skin


scaling and telangiectasia, which develop in response to chronic heat exposure, for example,
because of placement of a hot water bottle.
D Erythema multiforme

Erythema multiforme is incorrect. Erythema multiforme is often related to acute viral


infection, it is characterised by multiple, rounded, ‘target’ lesions, rather than the appearance
seen here.

E Erythema nodosum

Erythema nodosum is incorrect. The rash of erythema nodosum is characterised by raised,


nodular, red/purple lesions on both shins. It is associated with sarcoidosis and with
mycobacterium tuberculosis infection.
51445

Rate this question:

Next Question

Previous Question Tag Question

Feedback End Review

Difficulty: Average

Peer Responses %

Q. Answered Flagged

Q1

Q2

Q3

Q4

Q5
Q. Answered Flagged

Q6

Q7

Q8

0:02:24/03:00:00

A 52-year-old woman presents to the Oncology Clinic to discuss her prognosis following
lumpectomy, chemotherapy and radiotherapy for left breast cancer. She’s interested in
whether blood tests can be used to monitor for recurrence of her tumour.
Which of the following markers would be a useful blood test for monitoring breast cancer
recurrence?

A CA 15-3

B CA 19-9

C CA 125

D CD 20

E Cytokeratin fragment 21-1

Explanation 

A CA 15-3

Both CA15-3 and CA 27.29 are assays which detect MUC1 in peripheral blood, a well-known
marker of carcinoma of the breast. The assay correlates well with the burden of metastatic
disease, although it doesn’t predict response to, or aid in, targeting of specific therapies.

B CA 19-9

CA 19-9 is incorrect. CA 19-9 is a surface antigen associated with pancreatic carcinoma, which
can be used to monitor for tumour recurrence. It should not be used for initial diagnosis, as it
is raised in a range of gastrointestinal tumours.

C CA 125

CA 125 is incorrect. CA 125 is associated with ovarian carcinoma. It is an antigen which


signifies the presence of mucin 16 in peripheral blood and can be used to monitor for tumour
burden/metastases associated with ovarian carcinoma.
D CD 20

CD 20 is incorrect. CD 20 is a lymphocyte cell-surface receptor. It is therefore used in the


management of lymphoma to determine if CD20 targeting therapies are appropriate.

E Cytokeratin fragment 21-1

Cytokeratin fragment 21-1 is incorrect. Cytokeratin fragment 21-1 is a fragment of cytokeratin


which is found in patients with non-small cell lung cancer and can therefore be used as a
peripheral blood marker to look for tumour recurrence.
51382

Rate this question:

Next Question

Previous Question Tag Question

Feedback End Review

Difficulty: Average

Peer Responses %

Q. Answered Flagged

Q1

Q2

Q3

Q4

Q5
Q. Answered Flagged

Q6

Q7

Q8 
0:02:24/03:00:00

A 52-year-old man presents to the Oncology Clinic for review. After presenting to his GP with
a change in bowel habit and weight loss, he has been diagnosed with a right-sided colonic
carcinoma and hepatic metastases.
Which of the following tumour markers is most likely to be elevated?

A Alpha-fetoprotein (AFP)

B Carcinoembryonic antigen (CEA)

C CA 125

D CA 19-9

E DR-70

Explanation 

B Carcinoembryonic antigen (CEA)

CEA is an oncofetal protein which is elevated in a variety of cancers. Because it lacks both
specificity and sensitivity, it can’t be used to make a diagnosis of colorectal cancer, although
it does correlate with disease burden and can be used to monitor for tumour recurrence.

A Alpha-fetoprotein (AFP)

AFP is incorrect. AFP is elevated in patients with hepatocellular carcinoma and in non-
seminiferous germ cell tumours. It isn’t of value here where the hepatic metastases are linked
to the colorectal carcinoma primary.

C CA 125

CA 125 is incorrect. CA 125 is a peripheral blood marker of mucin-16, which is elevated in


cases of ovarian carcinoma and is linked to the overall tumour burden. It can be used to
monitor for recurrence of disease.

D CA 19-9
CA 19-9 is incorrect. CA 19-9 is a cell-surface protein which is present in a range of
gastrointestinal cancers. Its primary use is in monitoring for recurrence of pancreatic
carcinoma. It may be used as a suite of markers with CEA and DR-70 to improve specificity
with respect to tumour detection, but is invariably not elevated in colon cancer.

E DR-70

DR-70 is incorrect. DR-70 is elevated in both bronchial and colorectal carcinoma. It is


elevated in carcinomas, not in adenomas, and can therefore indicate the transition point to
malignancy. Like CA 19-9, it’s used as part of a suite of tumour markers. CEA is, however, the
most likely tumour marker to be elevated.
51383

Rate this question:

Next Question

Previous Question Tag Question

Feedback End Review

Difficulty: Average

Peer Responses %

Q. Answered Flagged

Q1

Q2

Q3

Q4

Q5
Q. Answered Flagged

Q6

Q7

Q8

0:02:24/03:00:00

A 34-year-old man who has been diagnosed with asthma some three years ago by his GP
comes to the Respiratory Clinic for review. He is still short of breath despite regular inhaled
therapy using fluticasone and salmeterol in combination. He is also under investigation for
abnormal liver function by his GP. He smokes five cigarettes per day, and drinks ten units of
alcohol per week. Auscultation of the chest reveals bilateral quiet breath sounds with
scattered wheeze. Abdomen is soft and non-tender; his body mass index is 26 kg/m 2.
Investigations:

Hb 139 g/l

WCC 7.4 × 10 9/l

PLT 210 × 10 9/l

Na + 142 mmol/l

K+ 4.5 mmol/l

Cr 90 µmol/l

ALT 121 IU/l

Alk phos 132 IU/l

Bilirubin 10 µmol/l

hyper-expanded lung fields consistent with obstructive lung


Chest X-ray (CXR)
disease

Pulmonary function
obstructive picture with reduced transfer factor
tests

Which of the following is the most likely unifying diagnosis?

A Alpha-1-anti-trypsin deficiency

B Bronchial asthma

C Granulomatosis with polyangiitis

D Idiopathic pulmonary fibrosis


E Obesity hypoventilation syndrome

Explanation 

A Alpha-1-anti-trypsin deficiency

The abnormal liver function tests and very early presentation with obstructive lung disease,
coupled with reduced transfer factor, are suggestive of hepatic fibrosis and emphysema
related to a-1-anti-trypsin deficiency. Smoking is thought to accelerate the development of
emphysema, and he should be encouraged to stop. Alcohol consumption may also accelerate
the development of liver cirrhosis.

B Bronchial asthma

Bronchial asthma is incorrect. The reduced transfer factor is more in keeping with
emphysema, and the abnormal liver function tests suggest a-1-anti-trypsin deficiency as the
alternative diagnosis.

C Granulomatosis with polyangiitis

Granulomatosis with polyangiitis is incorrect. Granulomatosis, previously known as Wegener’s


granulomatosis, is associated with pulmonary fibrosis, with a restrictive pattern on pulmonary
function testing and pulmonary haemorrhage. There is also often evidence of inflammation
affecting the nasal sinuses.

D Idiopathic pulmonary fibrosis

Idiopathic pulmonary fibrosis is incorrect. Idiopathic pulmonary fibrosis is associated with a


restrictive lung picture on pulmonary function testing and reduced transfer factor. It is also
unlikely to present at the age of 34.

E Obesity hypoventilation syndrome

Obesity hypoventilation syndrome is incorrect. Given a body mass index of 26, obesity
hypoventilation syndrome resulting in an obstructive lung picture is very unlikely. In addition,
it would not lead to reduced transfer factor.
51449

Rate this question:

Next Question
Previous Question
Tag Question

Feedback End Review

Difficulty: Average

Peer Responses %

Q. Answered Flagged

Q1

Q2

Q3

Q4

Q5

Q6

Q7

Q8

Q9

0:02:24/03:00:00

A 67-year-old woman is admitted to the Emergency Department having suffered a collapse


whilst in church. She is currently taking clarithromycin for a respiratory tract infection and
has a history of ischaemic heart disease and chronic obstructive pulmonary disease (COPD).
On arrival in the Emergency Department, her blood pressure is 115/80 mmHg, pulse is 82
bpm and regular. There is poor air entry on auscultation of the chest consistent with COPD.
Whilst you are reviewing her, you note several short episodes of torsades de pointes
ventricular tachycardia (VT) on the cardiac monitor.
Investigations:

Hb 138 g/l

WCC 6.7 × 10 9/l

PLT 281 ×10 9/l

Na+ 143 mmol/l

K+ 4.8 mmol/l

Bicarbonate 28 mmol/l

Cr 105 µmol/l

Which of the following is the most appropriate intervention?

A Amiodarone

B Calcium

C Magnesium

D Metoprolol

E Verapamil

Explanation 

C Magnesium
This patient has torsades. As well as withdrawing the likely causative agent (in this case
clarithromycin), intravenous magnesium is the intervention of choice. A dose of 2 g is the
usual standard, which can be repeated. Other drugs apart from macrolides recognised as
common causes of torsades include the -azole antifungals, tricyclic antidepressants,
antipsychotics and methadone.

A Amiodarone

Amiodarone is incorrect. Amiodarone prolongs the action potential, has no positive effect
with respect to treating torsades and may actually increase the frequency of episodes.

B Calcium

Calcium is incorrect. Calcium gluconate IV boluses are used for cardioprotection in the
treatment of hyperkalaemia. In hyperkalaemia, boluses of IV calcium gluconate can be
repeated at 15 m intervals.

D Metoprolol

Metoprolol is incorrect. Metoprolol is used as a treatment for paroxysmal supraventricular


tachycardia. Like amiodarone, it leads to prolongation of the action potential and is not an
appropriate treatment for torsades de pointes VT.

E Verapamil

Verapamil is incorrect. Verapamil is used to treat paroxysmal supraventricular tachycardia


where patients have failed to respond to other interventions such as the postural modified
Valsalva manoeuvre or adenosine.
51370

Rate this question:

Next Question

Previous Question Tag Question

Feedback End Review

Difficulty: Average

Peer Responses %
Q. Answered Flagged

Q1

Q2

Q3

Q4

Q5

Q6

Q7

Q8

Q9

0:02:24/03:00:00

A 17-year-old is admitted to the Emergency Department with headaches over the past 48 h
with intermittent vomiting, diarrhoea and a deterioration in his renal function as measured by
his general practitioner. He has been working over the past two weeks at an organic farm as
part of his work experience. Two other helpers at the farm are also apparently ill with similar
symptoms. His blood pressure is 138/84 mmHg, pulse is 95 bpm and regular. Abdomen is
soft, although there is general tenderness.
Investigations:

Hb 97 g/l

WCC 10.5 × 10 9/l

PLT 58 × 10 9/l

Na + 144 mmol/l

K+ 6.1 mmo/l

Cr 231 µmol/l

Which of the following is the most likely cause of this patient’s presentation?

A Bacillus cereus

B Campylobacter jejuni

C Escherichia coli

D Staphylococcus aureus

E Salmonella typhi

Explanation 

C Escherichia coli

This patient has likely been exposed to E. coli 0157 via his work on the farm, which has
resulted in the clinical picture seen here with evidence of renal impairment and haemolytic
anaemia; haemolytic-uraemic syndrome. Treatment is supportive; there is no evidence for use
of antiplatelet agents, fresh frozen plasma (FFP) or anticoagulants. There is some evidence to
support use of eculizumab.

A Bacillus cereus

B. cereus is incorrect. B. cereus is found in rice and is associated with profuse vomiting or
diarrhoea, dependent on the type of toxin produced. It is associated in severe cases with pre-
renal impairment, but not with the haemolysis picture seen here

B Campylobacter jejuni

C. jejuni is incorrect. Campylobacter is found in dairy products or in contaminated meat; it


leads to diarrhoea and vomiting but not to the picture seen here with haemolysis and
significant renal impairment.

D Staphylococcus aureus

S. aureus is incorrect. S aureus food poisoning is caused by a toxin found in dairy foods which
have been stored at room temperature for a prolonged period of time. Its symptoms are
short-lived and are usually associated with profuse vomiting. It doesn’t cause the picture of
renal failure and haemolysis seen here.

E Salmonella typhi

S. typhi is incorrect. Salmonella infection is associated with diarrhoea and vomiting which
may persist for a number of days; it doesn’t usually lead to haemolysis, although it may lead
to pre-renal failure. There are rare reports of HUS occurring in association with typhoid fever.
(see External Links)
51407

Rate this question:

Next Question

Previous Question Tag Question

Feedback End Review

Difficulty: Average

Peer Responses %
Q. Answered Flagged

Q1

Q2

Q3

Q4

Q5

Q6

Q7

Q8

Q9

 External Links

Hemolytic uremic syndrome in association with typhoid fever.


ncbi.nlm.nih.gov/pubmed/12612998
(https://www.ncbi.nlm.nih.gov/pubmed/12612998)
0:02:24/03:00:00

A 65-year-old man presents to the Dermatology Clinic for review. He has begun to develop
an erythematous rash over his abdomen and chest that he describes as intensely itchy and
that it seems to be growing up to 1 cm per day. He smokes heavily, has a history of chronic
obstructive pulmonary disease (COPD) and has suffered from indigestion over the past 6
months. You can see an obvious erythematous scaling rash over the abdomen, with some
evidence of scratch marks, which has almost a wood-grain appearance. Routine blood testing
is unremarkable, apart from erythrocyte sedimentation rate which is raised at 72 mm/hr.
Urine testing is normal (no blood or protein is detected).

Which of the following is the most likely cause?

A Bronchial carcinoma

B Gastric cancer

C Prostate cancer

D Pancreatic cancer

E Transitional cell carcinoma of the bladder (TCC)

Explanation 

A Bronchial carcinoma

The rash here is consistent with erythema gyratum repens. It is most strongly associated with
bronchial carcinoma, which fits with the history of heavy smoking and COPD here. The rash
may appear up to 9 months before malignancy is actually diagnosed.

B Gastric cancer

Gastric cancer is incorrect. Although gastric cancer is associated with erythema gyratum
repens and the patient has symptoms of indigestion, this is less likely as a cause versus
bronchial carcinoma. Gastric cancer is also associated with the development of acanthosis
nigricans.

C Prostate cancer
Prostate cancer is incorrect. Prostate cancer is only rarely associated with erythema gyratum
repens, and there are no symptoms of prostatism reported here.

D Pancreatic cancer

Pancreatic cancer is incorrect. Although the indigestion may fit with a diagnosis of pancreatic
carcinoma, it isn’t commonly associated with erythema gyratum repens. Glucagonoma is of
course associated with necrolytic migratory erythema.

E Transitional cell carcinoma of the bladder (TCC)

TCC is incorrect. TCC is a rare cause of erythema gyratum repens; however, the lack of
urinary symptoms and the lack of haematuria here would count against TCC as the
underlying diagnosis, where at least microscopic haematuria would be expected.
51410

Rate this question:

Next Question

Previous Question Tag Question

Feedback End Review

Difficulty: Average

Peer Responses %

Q. Answered Flagged

Q1

Q2

Q3
Q. Answered Flagged

Q4

Q5

Q6

Q7

Q8 
0:02:24/03:00:00

A 46-year-old man with a history of chronic pancreatitis comes to the Gastroenterology


Clinic for review. He complains of chronic diarrhoea which is often difficult to flush away. He
has continued to suffer from intermittent epigastric pain over the past few months. His blood
pressure is 110/85 mmHg, pulse is 74 bpm and regular. His body mass index (BMI) is 21 kg/m 2.
Faecal elastase is 90 µg/g.
Which of the following is the most appropriate initial intervention for his diarrhoea?

A Cholestyramine

B Codeine phosphate

C Creon

D Loperamide

E Metformin

Explanation 

C Creon

The faecal elastase result seen here indicates severe exocrine pancreatic insufficiency. The
low BMI and diarrhoea are consistent with malabsorption. As such, pancreatic enzyme
replacement with creon capsules is the most appropriate intervention.

A Cholestyramine

Cholestyramine is incorrect. Cholestyramine is the most appropriate intervention in the


treatment of bile acid diarrhoea, which can be seen in malabsorption related to active
Crohn’s disease, or due to small bowel resection.

B Codeine phosphate

Codeine phosphate is incorrect. Codeine phosphate slows bowel transit and increases
absorption of water. It doesn’t affect malabsorption and therefore isn’t an appropriate
intervention here.
D Loperamide

Loperamide is incorrect. Loperamide is a partial opiate agonist and like codeine phosphate,
slows bowel transit and increases absorption of water from stool; reducing the volume of
diarrhoea but not affecting malabsorption.

E Metformin

Metformin is incorrect. Metformin is a cause of bile acid malabsorption and subsequent


diarrhoea, rather than being a therapeutic intervention for the condition. Gradual dose
titration of metformin is thought to reduce the risk of diarrhoea.
51358

Rate this question:

Next Question

Previous Question Tag Question

Feedback End Review

Difficulty: Average

Peer Responses %

Q. Answered Flagged

Q1

Q2

Q3

Q4

Q5
Q. Answered Flagged

Q6

Q7

Q8

0:02:24/03:00:00

A 37-year-old woman comes to the Rheumatology Clinic for review. She has an active flare of
her lupus, with multiple joint pains, worsening of her skin rash and an increase in her serum
creatinine by 40 µmol/l.
Which of the following changes is most likely to be found?

A Elevated C3

B Elevated C-reactive protein (CRP)

C Peripheral blood eosinophilia

D Reduced C3

E Elevated white blood cell count

Explanation 

D Reduced C3

Both C3 and C4 are decreased and C3d (a complement degradation product) is increased in
patients with active lupus, due to ongoing inflammation and consumption of complement
factors.

A Elevated C3

Elevated C3 is incorrect. C3 levels are decreased in patients with active inflammation related
to systemic lupus erythematosus (SLE) but C3d is increased.

B Elevated C-reactive protein (CRP)

Elevated CRP is incorrect. CRP is not usually elevated in patients with active lupus unless
there is concurrent infection or serositis.

C Peripheral blood eosinophilia


Peripheral blood eosinophilia is incorrect. Peripheral blood eosinophilia is seen in patients
with eosinophilic granulomatosis with polyangiitis, who often present with symptoms of
poorly controlled asthma.

E Elevated white blood cell count

Elevated white blood cell count is incorrect. A reduction in peripheral white cell count is often
seen in patients with active SLE.
51401

Rate this question:

Next Question

Previous Question Tag Question

Feedback End Review

Difficulty: Average

Peer Responses %

Q. Answered Flagged

Q1

Q2

Q3

Q4

Q5

Q6

Q7
Q. Answered Flagged

Q8

0:02:24/03:00:00

A 40-year-old woman comes to the Neurology Clinic with her father who has recently been
diagnosed with Alzheimer’s disease at the age of 71. Her uncle was also diagnosed with the
condition aged 69. She has a history of hypertension which is controlled with amlodipine
tablets and has mildly elevated triglycerides. Her blood pressure is 149/82 mmHg, her pulse is
70 bpm and regular and her body mass index is 31 kg/m 2. She wants to know about her risk
of developing Alzheimer’s disease.
Which of the following is her most important risk factor?

A Blood pressure of 149/82 mmHg

B Body mass index of 27 kg/m 2

C Elevated triglycerides

D Family history

E Treatment with amlodipine

Explanation 

D Family history

Family history is a significant risk factor for the development of Alzheimer’s. Patients with a
first-degree relative with dementia have up to a 30% increased risk of developing the
disorder. Where two or more siblings are affected, there is up to a threefold increase in their
risk of developing the disease compared with the general population.

A Blood pressure of 149/82 mmHg

Blood pressure of 149/82 mmHg is incorrect. Mid-life hypertension is consistently associated


with the development of dementia in cross-sectional and longitudinal cohort studies. Arterial
stiffness may be associated with the development of arterial inflammation and hence
deposition of amyloid plaques or atherosclerosis associated with multi-infarct dementia.

B Body mass index of 27 kg/m 2


Body mass index of 31 kg/m 2 is incorrect. Obesity is associated with an approximately 1.5-fold
increase in the risk of developing dementia; this is thought to be related to hyperinsulinaemia,
which is associated with increased risk of atherosclerotic vascular disease.

C Elevated triglycerides

Elevated triglycerides is incorrect. An association has been found between elevated levels of
total cholesterol and low-density lipoprotein (LDL), and risk of dementia, although no strong
evidence has been found to suggest hypertriglyceridaemia as a risk factor.

E Treatment with amlodipine

Treatment with amlodipine is incorrect. Calcium channel antagonists used for the treatment
of hypertension do not increase the risk of dementia. Beta blockers and thiazide diuretics
may have mild negative effects on insulin resistance, although these agents also have not
been shown to increase dementia risk.
51437

Rate this question:

Next Question

Previous Question Tag Question

Feedback End Review

Difficulty: Average

Peer Responses %

Q. Answered Flagged

Q1

Q2
Q. Answered Flagged

Q3

Q4

Q5

Q6

Q7

Q8 
0:02:24/03:00:00

An 18-year-old woman comes to the Endocrine Clinic for review. She is concerned, as she has
yet to begin her periods. Her only past medical history of note is bilateral inguinal hernia
repair while still a neonate. Her blood pressure is normal at 105/80 mmHg, her pulse is 65
bpm and regular, her height is 1.78 m, with a body mass index of 22 kg/m 2. She has normal
breasts and external genitalia, although you note an absence of pubic hair.
Which of the following is the most likely diagnosis?

A Androgen insensitivity syndrome (AIS)

B Classical congenital adrenal hyperplasia (CCAH)

C Klinefelter syndrome

D Non-classical adrenal hyperplasia (NCAH)

E Turner syndrome

Explanation 

A Androgen insensitivity syndrome (AIS)

AIS is caused by a congenital abnormality of the androgen receptor, rendering it insensitive


to androgens. This means that individuals with AIS have a male XY karyotype, although
external genitalia are female, with little or no pubic hair. Testosterone levels are actually
elevated. Mal-descent of testes means that patients often present with bilateral inguinal
hernias in the first weeks of life.

B Classical congenital adrenal hyperplasia (CCAH)

CCAH is incorrect. CCAH is associated with normal development of pubic and axillary hair
and may present with clitoromegaly/ambiguous genitalia, which isn’t the picture here. In
severe cases, infants may present with salt wasting.

C Klinefelter syndrome
Klinefelter syndrome is incorrect. Klinefelter syndrome is associated with XXY karyotype,
small testes and a male phenotype. Testosterone levels are low and the condition is
associated with relationship difficulties, erectile dysfunction and early-onset osteoporosis.

D Non-classical adrenal hyperplasia (NCAH)

NCAH is incorrect. NCAH may present with precocious puberty, although many women with
the condition present later in life with androgenisation and subfertility. It does not present
with salt wasting.

E Turner syndrome

Turner syndrome is incorrect. It is associated with an XO karyotype and presents with short
stature, hypertension and primary amenorrhoea. The normal blood pressure and height count
against Turner syndrome as the underlying diagnosis.
51357

Rate this question:

Next Question

Previous Question Tag Question

Feedback End Review

Difficulty: Average

Peer Responses %

Q. Answered Flagged

Q1

Q2
Q. Answered Flagged

Q3

Q4

Q5

Q6

Q7

Q8
0:02:24/03:00:00

A 53-year-old man comes to the Emergency Department with a severe episode of pharyngitis
and is found to have a neutrophil count of 0.1 × 10 9/l. He has a history of psoriatic arthritis,
who has had multiple intolerances to previous disease-modifying antirheumatic drugs is
currently taking azathioprine. He also has type 2 diabetes treated with metformin and
hypertension. Over the past two months, he has suffered two episodes of inflammation
affecting his right great toe which has been diagnosed as gout by his general practitioner.
Which of the following agents is most likely to have been responsible for his decreased
white blood cell count?

A Benzbromarone

B Canakinumab

C Colchicine

D Febuxostat

E Rasburicase

Explanation 

D Febuxostat

Like allopurinol, febuxostat is a xanthine oxidase inhibitor, which promotes toxic


accumulation of 6-mercaptopurine. This, in turn, can lead to significant bone marrow
suppression as is seen here. Xanthine oxidase inhibitors should therefore be avoided in
patients taking azathioprine.

A Benzbromarone

Benzbromarone is incorrect. Benzbromarone is an OAT-1 inhibitor which promotes increased


urate excretion. It is indicated for patients with gout who are resistant to treatment with
xanthine oxidase inhibitors alone. It doesn’t lead to accumulation of azathioprine.

B Canakinumab
Canakinumab is incorrect. Canakinumab is an interleukin-1 (IL-1) antagonist, used in the
treatment of acute gout flares; although it may cause a decrease in white cell count, it’s very
unlikely to have been prescribed by a GP as initial therapy.

C Colchicine

Colchicine is incorrect. Colchicine is an alternative for chronic treatment of gout in patients


who are unable to take xanthine oxidase inhibitors. It can cause diarrhoea and pre-renal
impairment and may depress white cell count in some patients, although it’s less likely to be
an initial therapy choice here.

E Rasburicase

Rasburicase is incorrect. Rasburicase is recombinant urate oxidase, which is used in the


treatment of patients undergoing induction chemotherapy for haematological malignancy, to
reduce the risk of acute urate nephropathy.
51457

Rate this question:

Next Question

Previous Question Tag Question

Feedback End Review

Difficulty: Average

Peer Responses %

Q. Answered Flagged

Q1

Q2
Q. Answered Flagged

Q3

Q4

Q5

Q6

Q7

Q8 
0:02:24/03:00:00

A 28-year-old woman of Japanese descent presents to the Emergency Department with jaw
pain whilst eating, worsening shortness of breath on minimal exertion and central chest pain
whilst running for the bus over the past three months. She has also suffered from night
sweats and has lost 5 kg in weight. Blood pressure is elevated at 165/90 mmHg in the right
arm, and 149/90 mmHg in the left arm. You note a right carotid bruit on auscultation. Blood
workup reveals a normochromic normocytic anaemia and an erythrocyte sedimentation rate
(ESR) of 62 mm/h.
Which of the following is this patient’s most likely human leukocyte antigen (HLA)-type?

A HLA-B52

B HLA-B51

C HLA-DR4

D HLA-DR2

E HLA-B27

Explanation 

A HLA-B52

HLA-B52 is associated with Takayasu’s arteritis, the most likely diagnosis here. The disease
presents with symptoms corresponding to the vessels where arteritis has occurred, with the
ascending aorta, aortic arch and its branches usually affected first. HLA-B52 positivity is also
associated with increased risk of ulcerative colitis.

B HLA-B51

HLA-B51 is incorrect. HLA-B51 is associated with increased risk of Bechet’s disease and
increased susceptibility to infection with rubella virus.

C HLA-DR4
HLA-DR4 is incorrect. HLA-DR4 is associated with increased risk of rheumatoid arthritis.
There is also an association between DR4 positivity and drug-induced systemic lupus
erythematosus (SLE), pemphigoid, immunoglobulin A (IgA) nephropathy and polymyalgia
rheumatica.

D HLA-DR2

HLA-DR2 is incorrect. DR2 serotypes are associated with Goodpasture syndrome, SLE,
multiple sclerosis and narcolepsy.

E HLA-B27

HLA-B27 is incorrect. HLA-B27 occurs with increased frequency in patients with inflammatory
bowel disease and seronegative arthritides, including ankylosing spondylitis and reactive
arthritis.
51458

Rate this question:

Next Question

Previous Question Tag Question

Feedback End Review

Difficulty: Average

Peer Responses %

Q. Answered Flagged

Q1

Q2
Q. Answered Flagged

Q3

Q4

Q5

Q6

Q7

Q8
0:02:24/03:00:00

A 26-year-old woman comes to the Neurology Clinic for review. She describes three to four
episodes over the past two months where she has developed acute-onset vertigo, slurred
speech and tinnitus, accompanied by double vision, lasting around an hour each time. These
episodes are followed by a throbbing, occipital headache which lasts for a few hours and isn’t
relieved by paracetamol alone. Her only medication is the combined oral contraceptive pill.
Blood pressure is 122/82 mmHg, pulse is 63 bpm and regular. Neurological examination is
entirely normal.
Which of the following is the most likely diagnosis?

A Basilar migraine

B Benign positional vertigo (BPV)

C Meniere’s disease

D Temporal lobe epilepsy (TLE)

E Transient ischaemic attacks (TIAs)

Explanation 

A Basilar migraine

The symptoms of posterior territory vascular disturbance, followed by a throbbing headache


lasting a number of hours, fit well with the aura and headache associated with vascular
migraine. She currently takes the combined oral contraceptive pill, which increase the
frequency and severity of migraines. For this reason, and because it increases the risk of
ischemic stroke, the COCP should be discontinued. The lack of vascular risk factors points to
migraine as the likely cause versus multiple TIAs.

B Benign positional vertigo (BPV)

BPV is incorrect. BPV is associated with vertigo and nystagmus, but not with the headaches
seen here. Episodes of vertigo are also more prolonged than those seen here.

C Meniere’s disease
Meniere’s disease is incorrect. Meniere’s disease is associated with dizziness, vertigo, and
tinnitus and hearing loss, but not with the throbbing headaches seen here. The duration of
episodes also doesn’t fit with Meniere’s.

D Temporal lobe epilepsy (TLE)

TLE is incorrect. TLE is more usually associated with a rising epigastric sensation,
automatisms, and altered consciousness lasting up to several minutes. Vertigo may be a
feature where seizures involve the posterior superior temporal gyrus. Experiential or psychic
phenomena (eg déjà vu) and depersonalisation may also be seen, as well as olfactory or
gustatory hallucination.

E Transient ischaemic attacks (TIAs)

TIAs is incorrect. Although TIAs may present with posterior neurological deficit, there are no
vascular risk factors to suggest risk of TIA here, and headache would not usually be expected
in association with a TIA.
51438

Rate this question:

Next Question

Previous Question Tag Question

Feedback End Review

Difficulty: Average

Peer Responses %

Q. Answered Flagged

Q1
Q. Answered Flagged

Q2

Q3

Q4

Q5

Q6

Q7

Q8
0:02:24/03:00:00

A 47-year-old woman with a history of bipolar disease treated with a combination of lithium
and olanzapine for the past nine months comes to the clinic for review. She complains of
weight gain of 6 kg over the past three months and feels like she is slowing down
significantly. Her blood pressure is 138/85 mmHg, pulse is 56 bpm and regular. Her body
mass index is 35 kg/m 2 and you note a smooth goitre on palpation of her neck.
Investigations:

Hb 130 g/l

WCC 7.9 × 10 9/l

PLT 241 × 10 9/l

Na + 143 mmol/l

K+ 4.5 mmol/l

Cr 110 µmol/l

Fasting Total Cholesterol 5.8 mmol/l

Fasting glucose 6.9 mmol/l

TSH 9 mIU/l

T4 7 pmol/l

Which of the following is the most appropriate next step in treating her thyroid disease?

A Start prednisolone

B Start thyroxine

C Stop lithium

D Start metformin

E Stop olanzapine

Explanation 
B Start thyroxine

Hypothyroidism is most commonly seen within the first two years of starting treatment with
lithium and can occur with or without the development of a goitre. It occurs around five
times more commonly than in age- and sex-matched controls not taking lithium. Lithium is
thought to inhibit thyroid hormone release and may inhibit proteolytic digestion of
thyroglobulin. The hypothyroidism generally responds to treatment with thyroxine, and
lithium treatment can often be continued.

A Start prednisolone

Start prednisolone is incorrect. Prednisolone is used as treatment for the destructive


thyroiditis associated with amiodarone therapy and has no value in treatment of
hypothyroidism here.

C Stop lithium

Stop lithium is incorrect. Although stopping lithium may result in improvement in symptoms
of hypothyroidism, it shouldn’t be considered until discussion with the patient’s psychiatrist
about potential alternatives has taken place.

D Start metformin

Start metformin is incorrect. Although this patient’s glucose is at the upper end of the normal
range, stopping the olanzapine may be a better way to treat her impaired fasting glucose
than starting treatment with metformin.

E Stop olanzapine

Stop olanzapine is incorrect. Olanzapine is associated with metabolic abnormalities including


weight gain, disordered lipid metabolism, impaired glucose tolerance and diabetes mellitus,
rather than hypothyroidism.
51359

Rate this question:

Next Question

Previous Question Tag Question

Feedback End Review

Difficulty: Average
Peer Responses %

Q. Answered Flagged

Q1

Q2

Q3

Q4

Q5

Q6

Q7

Q8

Q9

0:02:24/03:00:00

An 18-year-old man is referred to the Neurology Clinic with motor and vocal tics which have
been going on for the past 18 months. He has been excluded from school for use of bad
language and shouting out in class. When you review him in the clinic, you note involuntary
arm movements, shouting out with swearing, and mimicking/repeating of your words.
Physical examination is entirely normal.
Investigations:

Hb 141 g/l

WCC 6.4 × 10 9/l

PLT 201 × 10 9/l

Na + 142 mmol/l

K+ 4.5 mmol/l

Cr 90 µmol/l

ALT 22 IU/L

Alk phos 85 IU/l

Bilirubin 10 µmol/l

Which of the following is the most appropriate intervention?

A Haloperidol

B Penicillamine

C Risperidone

D Sertraline

E Zinc

Explanation 

C Risperidone
This patient has Tourette syndrome, with typical behaviours and motor tics that fit with the
diagnosis. The normal blood picture, including normal liver function tests, makes a metabolic
cause for neurological dysfunction much less likely. Risperidone at low dose (used off-label)
is the usual preferred first intervention; higher doses may lead to problematic tardive
dyskinesia.

A Haloperidol

Haloperidol is incorrect. Although haloperidol is licensed for control of tics in Tourette


syndrome, the greater burden of extra-pyramidal side effects versus atypical antipsychotics,
means that risperidone is preferred.

B Penicillamine

Penicillamine is incorrect. Penicillamine is a chelating agent used in the treatment of Wilson’s


disease. The normal liver function tests make Wilson’s much less likely as the cause of this
patient’s motor symptoms.

D Sertraline

Sertraline is incorrect. Although sertraline is effective in control of obsessive-compulsive


behaviours and anxiety associated with Tourette’s, it may worsen tics. As such, it isn’t an
appropriate initial option for treatment.

E Zinc

Zinc is incorrect. Zinc acetate is an option for the treatment of Wilson’s disease; it is
recognised to reduce the absorption of copper from the GI tract. The presence of a normal
blood picture makes Wilson’s less likely as the underlying diagnosis here.
51439

Rate this question:

Next Question

Previous Question Tag Question

Feedback End Review

Difficulty: Average

Peer Responses %
Q. Answered Flagged

Q1

Q2

Q3

Q4

Q5

Q6

Q7

Q8

Q9

0:02:24/03:00:00

A 67-year-old woman is admitted to the Emergency Department with mild confusion. She is
noted to have a low sodium at 127 mmol/l and symptoms of a urinary tract infection.
Medication includes chlorthalidone and ramipril for the treatment of hypertension.
Where is the primary site of action of chlorthalidone?

A Collecting duct

B Distal convoluted tubule

C Glomerulus

D Proximal convoluted tubule

E Thick ascending loop of Henle

Explanation 

B Distal convoluted tubule

Chlorthalidone is a thiazide-like diuretic. Thiazide and thiazide-like diuretics act primarily on


the distal renal tubule, inhibiting NaCl reabsorption by antagonising the NaCl cotransporter
and promoting calcium reabsorption (by an unknown mechanism). Enhanced delivery of Na
and water to the cortical collection tubule and/or the increased flow rate leads to increased
secretion and excretion of potassium and hydrogen ions.
(see External Links)

A Collecting duct

Collecting duct is incorrect. Antidiuretic hormone acts primarily on the collecting duct,
increasing expression of aquaporin 2 channels, increasing free water absorption.

C Glomerulus

Glomerulus is incorrect. Angiotensin-converting enzyme (ACE) inhibitors affect glomerular


pressure, reducing pressure in efferent glomerular arterioles. This leads to a reduction in
progression of renal disease in patients with chronic renal impairment and risk factors such
as hypertension and diabetes mellitus.

D Proximal convoluted tubule

Proximal convoluted tubule is incorrect. Carbonic anhydrase inhibitors act primarily in the
proximal convoluted tubule to increase sodium, bicarbonate and water loss into urine. They
are used in the management of acute glaucoma. Chlorthalidone has some carbonic
anhydrase inhibitory effects, although it’s primary mechanism of action (MoA) is as a
thiazide-like diuretic.

E Thick ascending loop of Henle

Thick ascending loop of Henle is incorrect. Loop diuretics inhibit the sodium-potassium-
chloride cotransporter in the thick ascending limb. This transporter normally reabsorbs about
25% of the total sodium load; inhibition leads to a significant increase in the distal tubular
concentration of sodium and less water reabsorption in the collecting duct. This altered
handling of sodium and water leads to both diuresis and natriuresis.
51393

Rate this question:

Next Question

Previous Question Tag Question

Feedback End Review

Difficulty: Average

Peer Responses %

Q. Answered Flagged

Q1
Q. Answered Flagged

Q2

Q3

Q4

Q5

Q6

Q7

Q8

 External Links

Hygroton Tablets 50mg


medicines.org.uk/emc/product/863/smpc
(https://www.medicines.org.uk/emc/product/863/smpc)
0:02:24/03:00:00

A 27-year-old man is admitted to the Emergency Department having tried to slit his throat
with a knife. He is successfully resuscitated and you review him later. He has had numerous
previous admissions with both alcohol and drug intoxication. When you ask him why he did it,
he tells you that he could hear people talking about him, providing a commentary that he was
planning to slit his throat and that the television is inserting thoughts into his brain.
Which of the following is the most likely diagnosis?

A Bipolar disease type 1

B Borderline personality disorder

C Delirium tremens

D Drug-induced psychosis

E Schizophrenia

Explanation 

E Schizophrenia

The commentary in the third person suggesting this patient would try to kill himself, coupled
with delusions of thought insertion, fits well with a diagnosis of schizophrenia. The
background of both alcohol and drug abuse over a significant time period may suggest self-
medication to deal with the symptoms of schizophrenia.

A Bipolar disease type 1

Bipolar disease type 1 is incorrect. Bipolar disease type 1 is associated with periods of mania
that have severe symptoms, usually necessitating hospital admission, interspersed with
periods of depression. The picture here of thought insertion is much more characteristic of
schizophrenia.

B Borderline personality disorder


Borderline personality disorder is incorrect. Borderline personality disorder is associated with
emotional lability, fear of abandonment, an inability to form stable relationships, and a
tendency to act impulsively.

C Delirium tremens

Delirium tremens is incorrect. Delirium tremens is associated with agitation, tremor, sweating
and tachycardia. Tactile and visual hallucinations are usual, rather than auditory persecutory
hallucinations reported here.

D Drug-induced psychosis

Drug-induced psychosis is incorrect. Psychotic episodes are reported in association with


cannabis use, although the florid nature of this patient’s hallucinations, including thought
insertion, is more consistent with schizophrenia.
51446

Rate this question:

Next Question

Previous Question Tag Question

Feedback End Review

Difficulty: Average

Peer Responses %

Q. Answered Flagged

Q1

Q2
Q. Answered Flagged

Q3

Q4

Q5

Q6

Q7

Q8
0:02:24/03:00:00

A 72-year-old man with a history of type 1 diabetes is admitted to the Emergency


Department with severe headache, neck stiffness, and confusion which has developed over
the past 3–4 days. He has recently returned from a cheese-tasting trip to northern France. His
BP is 110/80 mmHg, pulse is 88 bpm and regular, his temperature is 38.8°C. There are
obvious signs of meningism, he is unable to comply with fundoscopy because of
photophobia.
Investigations:

Hb 127 g/l

WCC 10.1 × 10 9/l

PLT 201 × 10 9/l

C-reactive protein (CRP) 187 mg/l

Na + 141 mmol/l

K+ 4.5 mmol/

Cr 122 µmol/l

Glucose 8.4 mmol/l

Computed tomography (CT) head Unremarkable

Lumbar puncture Cerebrospinal fluid (CSF) lymphocytosis

Which of the following is the most likely diagnosis?

A Cerebral lymphoma

B Herpes simplex encephalitis

C Listeria meningitis

D Meningococcal meningitis

E Tuberculous meningitis

Explanation 
C Listeria meningitis

This patient is elderly and has suffered a gradual deterioration in consciousness accompanied
by a raised CRP and CSF lymphocytosis. Given the history of eating French cheeses, listeria is
a possibility. Dual therapy with amoxicillin and gentamicin is the usual intervention of choice.

A Cerebral lymphoma

Cerebral lymphoma is incorrect. A more gradual deterioration in consciousness would be


expected in cerebral lymphoma, as would other signs of haematological malignancy, such as
peripheral lymphadenopathy.

B Herpes simplex encephalitis

Herpes simplex encephalitis is incorrect. Herpes simplex encephalitis is associated with


lymphocytosis on CSF analysis, symptoms of memory loss and possible temporal lobe
abnormalities.

D Meningococcal meningitis

Meningococcal meningitis is incorrect. Meningococcal meningitis is associated with more


acute onset of symptoms of meningism, with a CSF neutrophilia rather than the
lymphocytosis seen here. It may also be associated with a purpuric rash.

E Tuberculous meningitis

Tuberculosis meningitis is incorrect. There is no history of exposure to tuberculosis or


respiratory disease here. Taken together, these facts make tuberculous meningitis much less
likely than listeriosis.
51413

Rate this question:

Next Question

Previous Question Tag Question

Feedback End Review

Difficulty: Average

Peer Responses %
Q. Answered Flagged

Q1

Q2

Q3

Q4

Q5

Q6

Q7

Q8

Q9

0:02:24/03:00:00

A 41-year-old man who has suffered from psoriasis for the past 6 years presents to the
Rheumatology Clinic for review. His plaque psoriasis affecting his elbows and knees is slowly
worsening and over the past 6 months, he has developed stiffness and swelling affecting the
small joints of his hands, meaning that he can’t now continue his job as an electrician.
Examination reveals evidence of distal interphalangeal joint inflammatory arthritis, affecting
both hands.
Which of the following is the most appropriate long-term intervention?

A Etanercept

B Hydroxychloroquine

C Methotrexate

D Prednisolone

E Sulfasalazine

Explanation 

C Methotrexate

Methotrexate is a recommended disease-modifying anti-rheumatic drug (DMARD) for the


treatment of psoriatic arthritis, particularly when there is extensive plaque disease. It is given
weekly with folic acid supplementation. It should be accompanied by a non-steroidal anti-
inflammatory drug (NSAID) as further adjunctive therapy to reduce local inflammation.

A Etanercept

Etanercept is incorrect. An anti-tumour necrosis factor (anti-TNF) therapy could be


introduced in patients who have failed to gain control with at least one conventional DMARD,
such as methotrexate.

B Hydroxychloroquine
Hydroxychloroquine is incorrect. Anti-malarial DMARDs such as hydroxychloroquine are not
recommended because they can cause exfoliative dermatitis, leading to a worsening of skin
disease.

D Prednisolone

Prednisolone is incorrect. Prednisolone may be of value as a short-term therapy in patients


with severe joint inflammation, although the risk-benefit profile of corticosteroids doesn’t
support their use over the longer term.

E Sulfasalazine

Sulfasalazine is incorrect. Sulfasalazine is an alternative to agents such as methotrexate and


leflunomide when these are unsuitable or not tolerated.
51459

Rate this question:

Next Question

Previous Question Tag Question

Feedback End Review

Difficulty: Average

Peer Responses %

Q. Answered Flagged

Q1

Q2

Q3
Q. Answered Flagged

Q4

Q5

Q6

Q7

Q8 
0:02:24/03:00:00

A 34-year-old woman with a history of depression is found unconscious with an empty bottle
of tablets and an empty bottle of whisky at her side. She is brought to the Emergency
Department by ambulance. Her Glasgow Coma Score (GCS) is 10, blood pressure is 100/75
mmHg; she has a sinus tachycardia with a pulse of 110 bpm and also bilateral pupil dilatation.
Electrocardiogram (ECG) reveals QRS widening with a duration of 110 ms. She has
intravenous (IV) normal saline running. Her pH is 7.21.
Which of the following is the most appropriate next intervention?

A Amiodarone

B Atropine

C Intralipid®

D Magnesium

E Sodium bicarbonate

Explanation 

E Sodium bicarbonate

When QRS rises above 100 ms in the presence of tricyclic antidepressant overdose (the most
likely diagnosis here), there is significantly increased risk of both seizures (26% or more) and
ventricular tachycardia. In this situation sodium bicarbonate reverses sodium channel
blockade and can therefore narrow the QRS complex.

A Amiodarone

Amiodarone is incorrect. Amiodarone is potentially associated with widening of the QRS; in


this situation it’s ineffective and may even increase the risk of ventricular tachycardia
occurring.

B Atropine
Atropine is incorrect. Atropine is an anticholinergic used in the treatment of heart block, it
isn’t useful here in the context of tachycardia.

C Intralipid®

Intralipid® is incorrect. Lipid emulsion can be considered in the cardiac arrest situation, where
the underlying cause is an overdose of a highly lipophilic agent such as a tricyclic
antidepressant. It is given in a 20% solution at a dose of 1.0–1.5 ml/kg, over a minute. In this
situation, with relative cardiovascular stability, there is time to give sodium bicarbonate first.

D Magnesium

Magnesium is incorrect. Magnesium isn’t recommended as a first-line agent for the treatment
of tricyclic antidepressant overdose. It may however be used as a second-line therapy for
patients who do not respond adequately to sodium bicarbonate.

Overall Explanation

(see External Links)


51386

Rate this question:

Next Question

Previous Question Tag Question

Feedback End Review

Difficulty: Average

Peer Responses %

Q. Answered Flagged

Q1
Q. Answered Flagged

Q2

Q3

Q4

Q5

Q6

Q7

Q8

 External Links

Gudeline for the management of Tricyclic antidepressant overdose


rcem.ac.uk/docs/College%20Guidelines/5z32.%20Tricyclic%20Antidepressant%20Overdose%20-%20(Flowchart)%2…
(https://www.rcem.ac.uk/docs/College%20Guidelines/5z32.%20Tricyclic%20Antidepressant%
%20(Flowchart)%20(Dec%202009).pdf)
0:02:24/03:00:00

A 29-year-old health worker presents to the clinic for review. She has acute hepatitis B,
diagnosed some 4 weeks earlier after a needle-stick injury some 2 months previous. She has
significant lethargy with abnormal liver function tests, easy bruising, and has recently become
jaundiced. Examination reveals jaundiced sclerae, bruising affecting her arms and legs, and a
tender right upper quadrant on palpation of the abdomen.
Investigations:

Hb 121 g/l

WCC 10.5 × 10 9/l

PLT 98 × 10 9/l

Na + 140 mmol/l

K+ 4.5 mmol/l

Cr 112 µmol/l

ALT 1245 IU/

AST 1132 IU/l

Bilirubin 52 µmol/l

Alk phos 289 IU/l

International normalized ratio (INR) 1.7

Hepatitis B surface antigen (HBsAg) Positive

Anti-hepatitis B core immunoglobulin M (IgM) Positive

Which of the following is the most appropriate next step?

A Boceprevir

B Peginterferon alpha-2a

C Ribavirin

D Telaprevir
E Tenofovir

Explanation 

E Tenofovir

In this situation, where acute hepatitis B has failed to clear after a 4-week period, and there is
evidence of significant liver dysfunction, tenofovir is indicated. Tenofovir disoproxil is a
nucleotide analogue reverse-transcriptase inhibitor (NtRTI); it prevents transcription of viral
ribonucleic acid (RNA) to deoxyribonucleic acid (DNA) and can be used in the treatment of
both Hepatitis B and human immunodeficiency virus 1 (HIV1).

A Boceprevir

Boceprevir is incorrect. Boceprevir is an inhibitor of NS3/4A serine protease; it is a modern


agent used in the treatment of hepatitis C and has significant efficacy with respect to viral
clearance.

B Peginterferon alpha-2a

Peginterferon alpha-2a is incorrect. Weekly peginterferon alpha-2a and daily oral ribavirin
was the standard regimen for treatment of hepatitis C in patients before more modern serine
protease inhibitors were launched.

C Ribavirin

Ribavirin is incorrect. Ribavirin is an oral agent used in the treatment of hepatitis C; it has
largely been superseded now by more modern protease inhibitors.

D Telaprevir

Telaprevir is incorrect. Like boceprevir, telaprevir is an inhibitor of NS3/4A serine protease. It


is used in the treatment of hepatitis C; it doesn’t have a role in the treatment of hepatitis B.
51415

Rate this question:

Next Question

Previous Question Tag Question

Feedback End Review


Difficulty: Average

Peer Responses %

Q. Answered Flagged

Q1

Q2

Q3

Q4

Q5

Q6

Q7

Q8

Q9

 External Links

Viread 245 mg film-coated tablets


medicines.org.uk/emc/product/1615/smpc
(https://www.medicines.org.uk/emc/product/1615/smpc)
0:02:24/03:00:00

A 27-year-old teacher who has been on assignment in Northern Thailand comes to the
Infectious Diseases clinic for review. He has been feeling unwell with a dry cough, epigastric
pain and gradual weight loss over the past few months. He also has an urticarial rash
affecting his buttocks and around his waist. He has a body mass index of 22 kg/m 2 and tells
you he has lost around 6 kg over the past 4 months.
Investigations:

Hb 127 g/l

WCC 10.9 × 10 9/l

Eosinophils 3.4 × 10 9/l

PLT 203 × 10 9/l

Na + 140 mmol/l

K+ 4.5 mmol/l

Cr 107 µmol/l

CRP 67 mg/l

Which of the following is the most useful intervention?

A Albendazole

B Ciprofloxacin

C Omalizumab

D Prednisolone

E Tinidazole

Explanation 

A Albendazole
This patient’s symptoms of gradual weight loss, abdominal pain, respiratory symptoms and
an urticarial rash affecting the buttock area are consistent with a diagnosis of strongyloides
infection. The infection is acquired via walking barefoot in an area where it is endemic. The
elevated C-reactive protein (CRP) and eosinophilia are consistent with strongyloides as the
underlying diagnosis. Both ivermectin and albendazole are potential interventions.

B Ciprofloxacin

Ciprofloxacin is incorrect. Ciprofloxacin is an option for gastrointestinal bacterial infections


including salmonella and campylobacter, where symptoms fail to improve or there is
evidence of chronic carriage of salmonella.

C Omalizumab

Omalizumab is incorrect. Omalizumab inhibits the binding of immunoglobulin E (IgE) to mast


cells and basophils; it is used in the treatment of asthma where IgE is markedly elevated.
Given the other symptoms, including gastrointestinal (GI) disturbance and skin rash,
strongyloides is much more likely to be the underlying cause.

D Prednisolone

Prednisolone is incorrect. Corticosteroids have no role in the treatment of this patient; where
there is chronic eosinophilia due to inflammation rather than infection, they are the first-line
intervention.

E Tinidazole

Tinidazole is incorrect. Tinidazole given as a one-off dose is the intervention of choice for the
treatment of giardia infection. A dose of 2 g is usual. A more prolonged course of
metronidazole may also be used.
51422

Rate this question:

Next Question

Previous Question Tag Question

Feedback End Review

Difficulty: Average

Peer Responses %
Q. Answered Flagged

Q1

Q2

Q3

Q4

Q5

Q6

Q7

Q8

Q9

0:02:24/03:00:00

A 49-year-old man with idiopathic membranous nephropathy comes to the clinic for review.
Despite optimal blood pressure control, his creatinine has increased from 157 to 188 µmol/l in
the past 6 months and proteinuria has increased from 4.9 to 6.5 g/24 h on latest
measurement. His blood pressure is 132/82 mmHg, and there is obvious pitting oedema
affecting both lower limbs.
Which of the following is the most appropriate immunosuppressive regimen for him?

A Mycophenolate mofetil

B Prednisolone and chlorambucil

C Prednisolone and golimumab

D Prednisolone and cyclophosphamide

E Rituximab

Explanation 

D Prednisolone and cyclophosphamide

In this situation, with a rapidly worsening serum creatinine, and increasing proteinuria,
intervention is indicated. Corticosteroids alone do not impact significantly on progression of
membranous nephropathy, combination treatment with an agent such as cyclophosphamide
is therefore required. Although cyclophosphamide and chlorambucil both reduce proteinuria
in combination, the side-effect profile of cyclophosphamide is more favourable, meaning this
is the preferred agent.

A Mycophenolate mofetil

Mycophenolate mofetil is incorrect. Mycophenolate mofetil is a mainstay of therapy for lupus


nephritis. Limited head-to-head evidence exists comparing mycophenolate with a
cyclophosphamide-based regimen, although referenced versus a historical control, it appears
inferior to cyclophosphamide, with less tolerability.

B Prednisolone and chlorambucil


Prednisolone and chlorambucil is incorrect. Chlorambucil is not preferred for the treatment of
membranous nephropathy because of its side-effect profile, including significant risk of bone
marrow suppression and haematological malignancy.

C Prednisolone and golimumab

Prednisolone and golimumab is incorrect. Golimumab is an anti-tumour necrosis factor


(https://en.wikipedia.org/wiki/Tumor_necrosis_factor) (TNF) agent which is used to treat
psoriasis and psoriatic arthritis; however, it is not of value in the treatment of membranous
nephropathy. Case reports suggest that anti-TNF agents may actually be associated with
increased risk of membranous nephropathy.

E Rituximab

Rituximab is incorrect. Rituximab is used where membranous nephropathy is unresponsive to


prednisolone and cyclophosphamide in combination. It is usually used in combination with
steroids rather than as monotherapy. Small studies also suggest a benefit in treatment-naïve
patients with membranous nephropathy at moderate risk of progression, which is maximal
some 18–24 months after treatment initiation.
51426

Rate this question:

Next Question

Previous Question Tag Question

Feedback End Review

Difficulty: Average

Peer Responses %

Q. Answered Flagged
Q. Answered Flagged

Q1

Q2

Q3

Q4

Q5

Q6

Q7

Q8 
0:02:24/03:00:00

A 41-year-old woman with worsening thyroid eye disease comes to the clinic for review. She
is treated with a block-replace regimen for her Graves’ disease and has a stable thyroid-
stimulating hormone (TSH) level. Her main complaint is of worsening diplopia and most
recently, impairment of colour vision. Her GP has commenced high-dose oral prednisolone,
although the patient feels that things are not improving. You confirm bilateral proptosis
consistent with Graves’ and decreased visual acuity.
Which of the following is the most appropriate next step?

A Alemtuzumab

B Bendroflumethiazide

C Furosemide

D Octreotide

E Rituximab

Explanation 

E Rituximab

Rituximab is an anti-CD20 monoclonal antibody, which is B-cell depleting. It reduces


antibody production and B-cell/T-cell crosstalk, leading to a reduction in symptoms of
thyroid eye disease. The greatest impact of rituximab is when it is used early, in patients with
severe symptoms. An alternative to rituximab in this situation is cyclosporine.

A Alemtuzumab

Alemtuzumab is incorrect. Alemtuzumab is an anti-CD52 monoclonal antibody. The CD52


cell-surface receptor is found on both T and B lymphocytes and binding to the receptor leads
to antibody-dependent cell lysis, and complement-mediated cell lysis.

B Bendroflumethiazide
Bendroflumethiazide is incorrect. Diuretics have no value in the treatment of Graves’ disease.
Use of thiazides has fallen in the treatment of hypertension, where they have been largely
replaced by thiazide-like agents such as indapamide.

C Furosemide

Furosemide is incorrect. Like thiazides, loop diuretics have no role in the treatment of Graves’
disease; they don’t impact significantly on symptoms or prognosis.

D Octreotide

Octreotide is incorrect. Trials of somatostatin analogues suggest they are ineffective in the
treatment of Graves. A meta-analysis has determined that there is no impact on diplopia,
proptosis or lid aperture.

Overall Explanation

(see External Links)


51414

Rate this question:

Next Question

Previous Question Tag Question

Feedback End Review

Difficulty: Average

Peer Responses %

Q. Answered Flagged

Q1
Q. Answered Flagged

Q2

Q3

Q4

Q5

Q6

Q7

Q8

 External Links

Treatment modalities for Graves' ophthalmopathy: systematic review and metaanalysis.


ncbi.nlm.nih.gov/pubmed?term=19491222
(https://www.ncbi.nlm.nih.gov/pubmed?term=19491222)

The 2016 European Thyroid Association/European Group on Graves' Orbitopathy Guidelines for the Mana…
karger.com/Article/FullText/443828
(https://www.karger.com/Article/FullText/443828)
0:02:24/03:00:00

A 63-year-old man who has type 2 diabetes, diagnosed 8 years earlier, comes to the clinic for
review. Current medication includes metformin 1 g twice daily (bd) and gliclazide 80 mg bd.
His glucose is well controlled with a glycated haemoglobin (HbA1c) of 54 mmol/mol. His main
complaint is of pins and needles and pain affecting both feet, particularly at night.
Examination reveals sensory loss to the ankles on both lower limbs.
Which of the following is the most appropriate initial intervention?

A Capsaicin

B Codeine phosphate

C Lamotrigine

D Pregabalin

E Tramadol

Explanation 

D Pregabalin

A number of first-line systemic agents are recommended by the National Institute for Health
and Care Excellence (NICE) guidelines for the treatment of diabetic peripheral neuropathy.
Amitriptyline, duloxetine, gabapentin or pregabalin can all be considered. Pregabalin is a
gamma-aminobutyric acid (GABA) analogue which increases the density of GABA
transporter proteins and increases the rate of functional GABA transport.

(see External Links)

A Capsaicin

Capsaicin is incorrect. Capsaicin cream applied topically to areas of neuropathic pain is


effective, although it is usually only employed when oral options are unsuitable or not
tolerated. When initially applied, it is associated with a burning sensation.

B Codeine phosphate
Codeine phosphate is incorrect. Opiates are less effective than other agents in controlling
neuropathic pain and have addictive potential. They should therefore be avoided here.

C Lamotrigine

Lamotrigine is incorrect. Other anti-epileptic drugs, apart from pregabalin, such as sodium
valproate, topiramate and lamotrigine, are not recommended first-line agents for neuropathic
pain. They may be used in combination or in a specialist pain clinic setting.

E Tramadol

Tramadol is incorrect. Although tramadol is less associated with gastrointestinal side effects
of opiate therapy such as constipation, it is not recommended here because of lack of
efficacy vs other options and potential for addiction.
51360

Rate this question:

Next Question

Previous Question Tag Question

Feedback End Review

Difficulty: Average

Peer Responses %

Q. Answered Flagged

Q1

Q2

Q3
Q. Answered Flagged

Q4

Q5

Q6

Q7

Q8 

 External Links

Neuropathic pain in adults: pharmacological management in non-specialist settings


nice.org.uk/guidance/CG173/chapter/1-Recommendations#treatment
(https://www.nice.org.uk/guidance/CG173/chapter/1-Recommendations#treatment)
0:02:24/03:00:00

A 28-year-old woman who is 14 weeks’ pregnant, presents to the Sexually Transmitted


Diseases Clinic with a painless vulval ulcer. She is found to have syphilis and is given a first
dose of penicillin. She presents 2–3 h later to the Emergency Department with a headache,
muscle pains and a tachycardia. She is found to be pyrexial with a fever of 38.1°C. She is not
known to be allergic to penicillin.
Which of the following is the most appropriate intervention?

A Intramuscular (IM) adrenaline

B Intravenous (IV) chlorpheniramine

C IV hydrocortisone

D Per oral (PO) azithromycin

E Reassurance

Explanation 

E Reassurance

This is the Jarisch–Herxheimer (JH) reaction after antibiotic treatment of syphilis. It occurs
because of release of bacterial proteins during cell lysis, which drives an immune response.
No treatment is required, apart from antipyretics where symptoms are severe, and it resolves
spontaneously. In women who are pregnant, it may stimulate uterine contractions.

A Intramuscular (IM) adrenaline

IM adrenaline is incorrect. IM adrenaline is an important step in the treatment of anaphylaxis


due to penicillin, the absence of other symptoms of anaphylaxis such as hypotension
erythema or bronchospasm counts against this as the underlying diagnosis.

B Intravenous (IV) chlorpheniramine


IV chlorpheniramine is incorrect. IV chlorpheniramine is used in the management of
anaphylaxis; it doesn’t have a role in treatment of the Jarisch-Herxheimer (JH) reaction. It is
an antihistamine and complementary to adrenaline and hydrocortisone in anaphylaxis
situations.

C IV hydrocortisone

IV hydrocortisone is incorrect. IV hydrocortisone is used in the management of anaphylaxis,


specifically; it has not been shown to improve symptoms related to the JH reaction.

D Per oral (PO) azithromycin

PO azithromycin is incorrect. A 2 g dose of azithromycin may be considered for the


treatment of syphilis in patients who are penicillin allergic. Macrolide resistance is, however,
increasing, so sensitivity testing is of key importance. Azithromycin doesn’t have a role in the
treatment of the JH reaction.
51423

Rate this question:

Next Question

Previous Question Tag Question

Feedback End Review

Difficulty: Average

Peer Responses %

Q. Answered Flagged

Q1

Q2
Q. Answered Flagged

Q3

Q4

Q5

Q6

Q7

Q8 
0:02:24/03:00:00

You are asked to evaluate the benefit of sacubitril–valsartan versus angiotensin-converting


enzyme inhibition for use in the heart failure clinic of your local hospital. The results of the
study showed that death from cardiovascular causes or hospitalisation for heart failure (the
primary endpoint) occurred in 914 patients (21.8%) in the sacubitril–valsartan group and 1117
patients (26.5%) in the enalapril group over 27 months of follow up.
Which of the following approximates best to the number needed to treat to prevent one
event over 27 months?

A 4.7

B 21

C 26.5

D 0.82

E 73

Explanation 

B 21

The number needed to treat for one event of death from cardiovascular causes or
hospitalisation for heart failure is 100/(26.5-21.8) = 100/4.7 or 21. In this population of patients
with low ejection fraction and high cardiovascular risk, sacubitril–valsartan would be
considered an effective option.

A 4.7

4.7 is incorrect. This is 4.7%, which is the absolute reduction in risk of a primary event
occurring between sacubitril–valsartan and enalapril in this study.

C 26.5

26.5 is incorrect. The absolute risk of a primary event occurring in patients prescribed
enalapril is 26.5.
D 0.82

0.82 is incorrect. This is the relative risk reduction for a primary event occurring in the
sacubitril-valsartan group versus the enalapril group, (21.8/26.5).

E 73

73 is incorrect. The percentage of patients who didn’t suffer an event in the patients treated
with enalapril over the course of 27 months is 73.
51405

Rate this question:

Next Question

Previous Question Tag Question

Feedback End Review

Difficulty: Average

Peer Responses %

Q. Answered Flagged

Q1

Q2

Q3

Q4

Q5

Q6
Q. Answered Flagged

Q7

Q8

 External Links

Angiotensin–Neprilysin Inhibition versus Enalapril in Heart Failure


nejm.org/doi/pdf/10.1056/NEJMoa1409077
(https://www.nejm.org/doi/pdf/10.1056/NEJMoa1409077)
0:02:24/03:00:00

A 46-year-old woman who has undergone lumpectomy for right-sided carcinoma of the
breast comes to the clinic for planning of her chemotherapy. She has an oestrogen- and
progesterone-receptor-positive tumour. A plan is made to treat her with trastuzumab.
Which of the following is the target for trastuzumab?

A CD3

B CD20

C CTLA-4

D HER2

E PD-1

Explanation 

D HER2

HER2 is a member of the epidermal growth factor receptor family, and over expression of
HER2 signifies a more aggressive breast carcinoma. Trastuzumab is a HER2 antagonist.
Normally, oestrogen and selective oestrogen receptor modulators such as tamoxifen
downregulate HER2 expression, although tumours may become resistant to tamoxifen, which
is then associated with HER2 over-expression.
(see External Links)

A CD3

CD3 is incorrect. CD3 is a T-cell surface receptor. Binding to CD3 leads to downregulation of
T-cell activity. CD3 is therefore a target in T-cell-mediated autoimmune diseases and used to
reduce risk of transplant rejection.

B CD20
CD20 is incorrect. CD20 is a B-lymphocyte surface receptor. Agents which bind to CD20,
such as rituximab, are used in the treatment of haematological malignancies and autoimmune
disease.

C CTLA-4

CTLA-4 is incorrect. CTLA-4 is a T-cell receptor which functions as an immune checkpoint.


Binding of CTLA-4 immunoglobulin downregulates immune activity and can be used as a
treatment for autoimmune disease. CTLA-4 antagonists are utilised in the treatment of solid
tumours.

E PD-1

PD-1 is incorrect. PD-1 is an immune-checkpoint cell-surface marker which is expressed on


several solid tumour types. PD-1 downregulates immune system attack, and PD-1 inhibitors
are therefore used to enhance recognition of tumour cells by T lymphocytes.
51384

Rate this question:

Next Question

Previous Question Tag Question

Feedback End Review

Difficulty: Average

Peer Responses %

Q. Answered Flagged

Q1

Q2
Q. Answered Flagged

Q3

Q4

Q5

Q6

Q7

Q8 

 External Links

Herceptin 150mg Powder for concentrate for solution for infusion


medicines.org.uk/emc/product/3856/smpc
(https://www.medicines.org.uk/emc/product/3856/smpc)
0:02:24/03:00:00

You are out jogging in the countryside when you come across an elderly male collapsed on
the path. You clear his airway and check for a pulse, you are unable to find one.
Which of the following is the most appropriate next step?

A Call for help

B Precordial thump

C Ten chest compressions

D Thirty chest compressions

E Two breaths

Explanation 

A Call for help

It is well known for out-of-hospital cardiac arrest that it is crucial for survival to minimise the
time between cardiac arrest and defibrillation as much as possible. For this reason, the
modern resuscitation guidelines recommend calling for help as the most important first step.

(see External Links)

B Precordial thump

Precordial thump is incorrect. Precordial thump is no longer recommended as part of


resuscitation guidelines. The amount of energy delivered in joules is unlikely to significantly
impact outcomes.

C Ten chest compressions

Ten chest compressions is incorrect. Maintaining uninterrupted circulation is most important


in successful resuscitation. As such, thirty chest compressions are recommended before
pausing to deliver a rescue breath.
D Thirty chest compressions

Thirty chest compressions is incorrect. Thirty chest compressions should be delivered only
once you have called for help, preferably using a mobile to call 999, being careful to give your
location.

E Two breaths

Two breaths is incorrect. Two breaths should be delivered only once you have completed
thirty chest compressions. When a defibrillator has arrived, it is recommended to attach it as
soon as is practically possible.
51371

Rate this question:

Next Question

Previous Question Tag Question

Feedback End Review

Difficulty: Average

Peer Responses %

Q. Answered Flagged

Q1

Q2

Q3

Q4

Q5
Q. Answered Flagged

Q6

Q7

Q8

 External Links

Adult BLS Algorithm


emedcert.com/blog/acls-algorithms-review-adult-bls-algorithm
(https://emedcert.com/blog/acls-algorithms-review-adult-bls-algorithm)
0:02:24/03:00:00

A 52-year-old man comes to the Gastroenterology Clinic complaining of early satiety,


bloating, and intermittent diarrhoea. He also reports a significant increase in flatulence over
the past two months. He has type 2 diabetes for which he takes metformin 1 g twice a day
and has not lost any weight. His body mass index is 32 kg/m 2 and his abdomen is soft and
non-tender with no palpable masses.
Investigations:

Hb 100 g/l (MCV 101)

WCC 7.2 × 10 9/l

PLT 187 × 10 9/l

Na + 137 mmol/l

K+ 4.5 mmol/l

Cr 95 µmol/l

Albumin 38 g/l

International normalized ratio (INR) 1.3

Lactulose hydrogen breath test Abnormal

Which of the following is the most appropriate intervention?

A Cholestyramine

B Codeine phosphate

C Creon

D Rifamixin

E Stop metformin

Explanation 

D Rifamixin
This patient has symptoms consistent with bacterial overgrowth, which is known to occur
with increased frequency in patients suffering from diabetes mellitus. The abnormal lactulose
hydrogen breath test further supports the diagnosis of bacterial overgrowth. The macrocytic
anaemia seen here is potentially due to vitamin B 12 deficiency occurring as a result of
bacterial metabolism of available B 12 in the small bowel. Lactulose hydrogen breath testing is
a non-invasive way to confirm the diagnosis. Rifamixin is a usual initial intervention of choice,
metronidazole and tetracyclines are potential alternatives.

A Cholestyramine

Cholestyramine is incorrect. Cholestyramine is used as a treatment for patients who have


diarrhoea as a result of short bowel syndrome due to resection, because of small bowel
inflammation or ischaemia. It binds to bile acids, reducing the diarrhoea which they cause.

B Codeine phosphate

Codeine phosphate is incorrect. Codeine acts to slow gastrointestinal transit time, although it
may reduce diarrhoea by decreasing stool water content, it won’t act to relieve the
underlying cause of this patient’s symptoms.

C Creon

Creon is incorrect. Creon is used as a treatment for pancreatic exocrine insufficiency.


However, there is no evidence with respect to this patient’s symptoms that chronic
pancreatitis and exocrine pancreatic failure is the reason for his symptoms.

E Stop metformin

Stop metformin is incorrect. Metformin is an effective therapy for type 2 diabetes and has
been shown to reduce cardiovascular risk. There is no evidence that the diarrhoea seen here
is due to the typical picture of bile acid malabsorption that is seen with metformin treatment.
51373

Rate this question:

Next Question

Previous Question Tag Question

Feedback End Review

Difficulty: Average

Peer Responses %
Q. Answered Flagged

Q1

Q2

Q3

Q4

Q5

Q6

Q7

Q8

Q9

0:02:24/03:00:00

An 18-year-old man is admitted to the Emergency Department having suffered a seizure. He


is from a travelling family, who admit to having ‘covered up’ a number of previous seizures
over the past few years. They tell you he suffered a dislocated shoulder after one previous
seizure. You note a number of small reddish spots around his nose and cheeks, a number of
skin tags, and some smooth fleshy growths adjacent to his fingernails. He has patches of skin
depigmentation over his trunk. Fundoscopy reveals a number of translucent rounded lesions
in the periphery of the retina.
Which of the following is the most likely diagnosis?

A Autoimmune polyglandular syndrome type 1

B Hypermobile Ehlers–Danlos syndrome

C Neurofibromatosis type 1

D Neurofibromatosis type 2

E Tuberous sclerosis

Explanation 

E Tuberous sclerosis

This patient has a number of features consistent with tuberous sclerosis including ash leaf
macules, adenoma sebaceum, ungual fibromas and retinal hamartomas. The epilepsy occurs
due to tuber formation within the brain. A brain magnetic resonance imaging (MRI) is the
obvious next step to further investigate the cause of this patient’s epilepsy.

A Autoimmune polyglandular syndrome type 1

Autoimmune polyglandular syndrome type 1 is incorrect. Although vitiligo is associated with


autoimmune polyglandular syndrome type 1, the other features here, including retinal
hamartomas and subungual fibromas and the absence of other features of endocrine
dysfunction, make this diagnosis unlikely.

B Hypermobile Ehlers–Danlos syndrome


Hypermobile Ehlers–Danlos syndrome is incorrect. This is a cause of recurrent shoulder
dislocation and may result in central nervous system symptoms when there is atlanto-axial
subluxation and an Arnold Chiari malformation; this isn’t suggested by the clinical features
seen here.

C Neurofibromatosis type 1

Neurofibromatosis type 1 is incorrect. Neurofibromatosis type 1 is associated with Lisch


nodules rather than retinal hamartomas. Patients also suffer from cutaneous neurofibromas
and multiple café au lait spots. Larger neurofibromas may result in spinal deformities,
including severe scoliosis in some patients.

D Neurofibromatosis type 2

Neurofibromatosis type 2 is incorrect. The main manifestation of neurofibromatosis type 2 is


the development of vestibular schwannomas; the tumours are bilateral and are diagnostic of
neurofibromatosis type 2.
51440

Rate this question:

Next Question

Previous Question Tag Question

Feedback End Review

Difficulty: Average

Peer Responses %

Q. Answered Flagged

Q1
Q. Answered Flagged

Q2

Q3

Q4

Q5

Q6

Q7

Q8
0:02:24/03:00:00

You are conducting an audit of adverse events associated with transfusion of blood products
in your hospital’s trust.
Which of the following do you think is the commonest cause of transfusion-associated
morbidity in the trust?

A ABO incompatibility

B Rhesus incompatibility

C Transfusion-associated circulatory overload (TACO)

D Transfusion-related lung injury (TRALI)

E Transfusion-related septicaemia

Explanation 

C Transfusion-associated circulatory overload (TACO)

TACO is the commonest cause of transfusion-associated morbidity in the UK. Serious


Hazards of Transfusion (SHOT), the organisation which monitors adverse events associated
with blood products, rates TACO as the most significant concern. Where transfusion is
considered, it’s very important to consider fluid balance, with appropriate use of diuretics
where required. Fourteen cases of definite, probable and possible TACO deaths were
reported across the UK in 2016.

(see External Links)

A ABO incompatibility

ABO incompatibility is incorrect. ABO incompatibility is thankfully very rare, particularly with
use of positive identification and introduction of computerised procedures for appropriately
labelling both patient samples and donor blood products.

B Rhesus incompatibility
Rhesus incompatibility is incorrect. Like ABO incompatibility, rhesus incompatibility is rarely
seen because of positive patient identification and computerised labelling of samples. Only
one (probable) death due to haemolytic transfusion reaction was reported across the whole
of the UK in 2016.

D Transfusion-related lung injury (TRALI)

TRALI is incorrect. TRALI is rare; it has an incidence of between 1 in 1500 and 1 in 5000
transfusions of plasma-related products. Where it occurs, it has a mortality rate of between
6% and 9%. It is potentially the next leading cause of mortality after TACO.

E Transfusion-related septicaemia

Transfusion-related septicaemia is incorrect. Bacterial contamination of packed cells is rare,


although visual inspection of the unit for transfusion is recommended to exclude the
presence of gas, or darkening of the blood, consistent with infection.
51385

Rate this question:

Next Question

Previous Question Tag Question

Feedback End Review

Difficulty: Average

Peer Responses %

Q. Answered Flagged

Q1

Q2
Q. Answered Flagged

Q3

Q4

Q5

Q6

Q7

Q8 

 External Links

ANNUAL SHOT REPORT 2016


shotuk.org/wp-content/uploads/myimages/SHOT-Report-2016_web_11th-July.pdf
(https://www.shotuk.org/wp-content/uploads/myimages/SHOT-Report-
2016_web_11th-July.pdf)
0:02:24/03:00:00

A 49-year-old man comes to the Endocrine Clinic for review. He has increased in weight by 7
kg over the past 3 months and his blood pressure has become increasingly difficult to
control, despite triple anti-hypertensive therapy. He also has impaired fasting glucose and has
been trying desperately hard to control his weight without success. His blood pressure is
155/90 mmHg, pulse is 67 bpm and regular. He has truncal obesity with striae and his body
mass index is 35 kg/m 2.
Investigations:

Hb 135 g/l

WCC 7.8 × 10 9/l

PLT 273 × 10 9/l

Na + 144 mmol/l

K+ 3.5 mmol/l

HCO 3 31 mmol/l

Cr 118 µmol/l

Fasting glucose 6.4 mmol/l

Which of the following is the most appropriate next investigation?

A 24 h urinary free cortisol

B Midnight cortisol

C Magnetic resonance imaging (MRI) pituitary

D Random cortisol

E Short Synacthen™ test

Explanation 

A 24 h urinary free cortisol


The obesity with impaired fasting glucose and metabolic alkalosis raises the possibility of
Cushing syndrome. Cushing syndrome is excess production of corticosteroid hormones by
the adrenal glands, it may be due to Cushing’s disease, where an ACTH producing pituitary
adenoma is identified. At this stage, an outpatient investigation such as urine collection for
cortisol is the least invasive next step. It is ideally repeated three times. Other investigations
can include midnight cortisol testing or dexamethasone suppression testing, but all of these
are more difficult to execute than collecting urine as a next investigation.

B Midnight cortisol

Midnight cortisol is incorrect. Midnight cortisol can be measured in blood or as a salivary test,
in patients with Cushing’s, diurnal variation may be lost, with elevated cortisol levels at night
(the sample is usually taken between 2300 and 0100 h. This is obviously more difficult to
arrange in the outpatient setting as a next investigation versus giving the patient a urine
container for collection.

C Magnetic resonance imaging (MRI) pituitary

MRI pituitary is incorrect. MRI pituitary without biochemical confirmation of Cushing’s runs
the risk of identifying an incidentaloma. MRI should therefore take place only after Cushing’s
disease is confirmed with elevated cortisol levels and an elevated adrenocorticotrophin
(ACTH).

D Random cortisol

Random cortisol, is incorrect. Cortisol release is pulsatile, even in the context of Cushing’s
syndrome. As such, a random cortisol is not useful in confirming an underlying diagnosis of
Cushing’s. A 24 h collection and/or midnight level is preferred.

E Short Synacthen™ test

Short Synacthen™ test is incorrect. A short Synacthen™ test is used to stimulate production
of cortisol and is an investigation used for possible adrenal insufficiency. Dexamethasone
suppression testing is used for diagnosis of Cushing’s, a low dose test for diagnosing the
condition, and a high dose test for differentiating between ectopic ACTH production and
production of ACTH by a pituitary adenoma.
51361

Rate this question:

Next Question

Previous Question Tag Question


Feedback End Review

Difficulty: Average

Peer Responses %

Q. Answered Flagged

Q1

Q2

Q3

Q4

Q5

Q6

Q7

Q8

Q9

0:02:24/03:00:00

A 45-year-old man who is a known alcoholic, comes to the Emergency Department having
suffered a 300 ml upper gastrointestinal (GI) haemorrhage of fresh blood. He presented with
a haemorrhage some six months earlier and was found to have large oesophageal varices,
but had defaulted from further follow-up. His blood pressure is 95/60 mmHg, pulse is 95
bpm and regular. He is tender in the epigastrium and you note dried blood around his mouth.
He has been cannulated and intravenous (IV) fluid resuscitation commenced.
Investigations:

Hb 88 g/l

WCC 11.3 × 10 9/l

PLT 85 × 10 9/l

Na + 135 mmol/l

K+ 4.9 mmol/l

Creatinine 88 µmol/l

Urea 12.2 mmol/l

International normalized ratio (INR) 2.1

Which of the following is the most important next intervention?

A Ciprofloxacin

B Omeprazole

C Propranolol

D Terlipressin

E Tranexamic acid

Explanation 

D Terlipressin
Terlipressin is an analogue of vasopressin and is recommended by National Institute for
Clinical Excellence (NICE) guidelines to be started as soon as possible in a patient presenting,
as here, with suspected variceal haemorrhage. It has been shown to reduce transfusion
requirements, achieve more rapid control of bleeding, and impact on mortality versus
standard of care in patients with variceal haemorrhage.

(see External Links)

A Ciprofloxacin

Ciprofloxacin is incorrect. Symptomatic bacterial infection occurs in up to 50% of patients


presenting with variceal haemorrhage. Endotoxins produced by the bacteria may impair
haemostasis and promote further haemorrhage. For this reason, antibiotics should be started
early but are not the most important next intervention.

B Omeprazole

Omeprazole is incorrect. There is no evidence that early administration of proton-pump


inhibitors diminishes the risk of variceal haemorrhage. They may reduce the risk of re-
bleeding in patients presenting with GI haemorrhage related to peptic ulcer disease.

C Propranolol

Propranolol is incorrect. Propranolol is the intervention of choice for long-term prophylaxis


against variceal haemorrhage because it acts to reduce portal pressure.

E Tranexamic acid

Tranexamic acid is incorrect. Agents such as tranexamic acid do not have a role in the
treatment of variceal haemorrhage. Where there is active bleeding and the INR is raised,
fresh frozen plasma (FFP) is the preferred intervention.
51374

Rate this question:

Next Question

Previous Question Tag Question

Feedback End Review

Difficulty: Average

Peer Responses %
Q. Answered Flagged

Q1

Q2

Q3

Q4

Q5

Q6

Q7

Q8

Q9

 External Links

Acute upper gastrointestinal bleeding in over 16s: management


nice.org.uk/guidance/cg141/chapter/introduction
(https://www.nice.org.uk/guidance/cg141/chapter/introduction)
0:02:24/03:00:00

A 22-year-old man who has been trekking in Nepal comes to the Emergency Department
with a lesion on his left ankle, fevers, muscle pains, vomiting, a dry cough and a severe
headache that has worsened significantly over the last 12 h. He is pyrexial at 38.3°C, pulse is
95 bpm and his blood pressure is 105/72 mmHg. He has a blackened, ulcerated lesion near his
left medial malleolus, and left inguinal lymphadenopathy. There is bilateral patchy infiltration
seen on the chest X-ray, and his white blood cell count is mildly elevated.
Which of the following is the most likely cause of his symptoms?

A Borrelia burgdorferi

B Orientia tsutsugamushi

C Rickettsia parkeri

D Rickettsia prowazekii

E Rickettsia typhi

Explanation 

B Orientia tsutsugamushi

O. tsutsugamushi is the cause of scrub typhus, which is endemic in Eastern and Southern
Asia, Northern Australia and the mountains of Tibet and Nepal. The presence of the eschar
coupled with symptoms of systemic infection, including pulmonary infiltration, fits well with
this as the cause. Doxycycline is the intervention of choice, with azithromycin an option
where tetracyclines are contraindicated.

A Borrelia burgdorferi

B. burgdorferi is incorrect. Lyme disease is caused by B. burgdorferi and is associated with a


tick bite. Although the eschar seen here may be consistent with tick exposure, pulmonary
infiltration in Lyme disease is not a usual finding.

C Rickettsia parkeri
R. parkeri is incorrect. R. parkeri is a cause of Rocky Mountain spotted fever, rather than tick
typhus, and wouldn’t fit with this patient’s trip to Nepal.

D Rickettsia prowazekii

R. prowazekii is incorrect. R. prowazekii is a cause of epidemic typhus. It is spread mainly by


lice, although it is also a cause of sylvatic typhus which is spread by fleas from the flying
squirrel.

E Rickettsia typhi

R. typhi is incorrect. This is a cause of endemic or murine typhus transmitted by fleas.


Therefore, it doesn’t fit with the picture of an eschar and systemic infection seen here.
51424

Rate this question:

Next Question

Previous Question Tag Question

Feedback End Review

Difficulty: Average

Peer Responses %

Q. Answered Flagged

Q1

Q2

Q3
Q. Answered Flagged

Q4

Q5

Q6

Q7

Q8
0:02:24/03:00:00

A 65-year-old man who has suffered from chronic back pain over the last six months comes
to the Emergency Department with weakness of his left leg. He smokes 20 cigarettes per day
and has been diagnosed with chronic obstructive pulmonary disease (COPD) some ten years
earlier. He admits to a worsening cough with occasional haemoptysis over recent months.
Examination confirms weakness of the left leg, there is loss of vibration and position sense
over the lower left portion of the abdomen and leg and loss of pain and temperature
sensation on the right-hand side.
Which of the following is the most likely cause of his neurological symptoms?

A Anterior spinal artery occlusion

B Central nervous system (CNS) lymphoma

C Disc herniation

D Lumbar spine radiculopathy

E Spinal cord hemisection

Explanation 

E Spinal cord hemisection

The picture of ipsilateral muscle weakness, vibration and position sense loss, coupled with
contra-lateral loss of pain and temperature sensation fits with a diagnosis of spinal cord
hemi-section, also known as Brown-Séquard syndrome; in this case, most likely due to a
metastasis from a bronchial carcinoma. A chest X-ray and spinal cord magnetic resonance
imaging (MRI) would be obvious next steps in investigation.

A Anterior spinal artery occlusion

Anterior spinal artery occlusion is incorrect. Anterior spinal artery occlusion is characterised
by infarction of the anterior two-thirds of the spinal cord, leading to bilateral loss of motor
function, pain and temperature sensation, although vibration and position sense are
preserved.
B Central nervous system (CNS) lymphoma

CNS lymphoma is incorrect. In CNS lymphoma, a gradual deterioration in cognitive function,


with signs of raised intracranial pressure and meningism, is more likely than the features of
spinal cord hemisection seen here.

C Disc herniation

Disc herniation is incorrect. Disc herniation is more likely to be associated with sudden-onset
back pain against a background of degenerative disease of the lumbar spine. Compression of
spinal nerves is usually more marked on one side than the other, leading to limited straight
leg raise, and pain in a nerve distribution corresponding to the position of the disc herniation.

D Lumbar spine radiculopathy

Lumbar spine radiculopathy is incorrect. This leads to chronic pain along the distribution of
the affected nerve, with loss of both motor and sensory function corresponding to a
particular nerve distribution.
51441

Rate this question:

Next Question

Previous Question Tag Question

Feedback End Review

Difficulty: Average

Peer Responses %

Q. Answered Flagged
Q. Answered Flagged

Q1

Q2

Q3

Q4

Q5

Q6

Q7

Q8
0:02:24/03:00:00

A 27-year-old woman comes to the clinic for review. She has decreased energy,
amenorrhoea, decreased libido and problems with milk leakage from her breasts on minimal
stimulation. You can reproduce the milk leakage on examination. Physical examination apart
from this, including visual field testing, is normal. Prolactin is measured at 5400 mU/l.
Which of the following is the most appropriate treatment for this patient?

A Cabergoline

B L-dopa

C Metoclopramide

D Octreotide

E Pegvisomant

Explanation 

A Cabergoline

Cabergoline is a potent ergo-derived D2 receptor dopamine agonist. It is thought to be more


effective than bromocriptine in suppressing prolactin levels in patients with a
macroprolactinoma, (the most likely diagnosis here). Patients who take dopamine agonists
should be warned that they are associated with restoration of fertility.
(see External Links)

B L-dopa

L-dopa is incorrect. Although L-dopa is an effective first-line therapy for Parkinson’s disease,
it isn’t recommended here because of the superior tolerability and efficacy profile of
dopamine agonists in the treatment of prolactinoma.

C Metoclopramide
Metoclopramide is incorrect. Metoclopramide is a dopamine antagonist and actually leads to
an increase rather than reduction in prolactin levels. It is, in fact, a cause of galactorrhoea.
Use of metoclopramide over the longer-term is now discouraged because of the risk of
neurological sequelae, including tardive dyskinesia.

D Octreotide

Octreotide is incorrect. Octreotide is a somatostatin analogue, it is preferred in the treatment


of acromegaly and carcinoid syndrome, rather than for the treatment of prolactinoma.

E Pegvisomant

Pegvisomant is incorrect. Pegvisomant is a growth hormone antagonist; as such, like


somatostatin analogues, it is primarily used for the treatment of acromegaly.
51378

Rate this question:

Next Question

Previous Question Tag Question

Feedback End Review

Difficulty: Average

Peer Responses %

Q. Answered Flagged

Q1

Q2

Q3
Q. Answered Flagged

Q4

Q5

Q6

Q7

Q8 

 External Links

Cabaser 1 mg Tablets
medicines.org.uk/emc/product/1074/smpc
(https://www.medicines.org.uk/emc/product/1074/smpc)
0:02:24/03:00:00

A 19-year-old student is admitted to the Emergency Department following a collapse at a


local nightclub. His friends tell you that he has taken three (https://medical-
dictionary.thefreedictionary.com/third) 4-methylenedioxymethamphetamine (MDMA) tablets
and alcohol during the evening at the club. On arrival in the Emergency Department, his BP is
155/90 mmHg, pulse is 110 bpm and regular. His temperature is 40ºC. He appears agitated,
with a Glasgow Coma Scale (GCS) of 13 and complains of generalised pain on palpation of
the abdomen. You note brisk reflexes and increased muscle tone.
Investigations:

Hb 140 g/l

WCC 14.2 × 10 9/l

PLT 322 × 10 9/l

Na + 149 mmol/l

K+ 5.9 mmol/l

Cr 152 µmol/l

Creatine kinase (CK) 1150 U/l

He is treated with intravenous (IV) benzodiazepines and IV fluids.

Which of the following is the most appropriate next step?

A Dantrolene

B Glycopyrronium

C Metoprolol

D Mechanical cooling measures

E Phenytoin

Explanation 

D Mechanical cooling measures


This patient’s temperature has reached a dangerously high level, with signs of dehydration
and rhabdomyolysis (elevated CK). Although benzodiazepines are helpful in this situation,
more aggressive measures to reduce pyrexia are also required. Mechanical cooling measures
such as a cooling blanket or even an ice bath may be of value here.

A Dantrolene

Dantrolene is incorrect. Dantrolene was used as monotherapy in the treatment of malignant


hyperthermia, although now it is used as a second-line agent, in some cases, on top of
anticholinergic therapy. Used as monotherapy, it may, in fact, be associated with worse
outcomes.

B Glycopyrronium

Glycopyrronium is incorrect. Anticholinergics are usually introduced in the event that


temperature fails to fall despite mechanical cooling and benzodiazepines.

C Metoprolol

Metoprolol is incorrect. Metoprolol is not recommended because it can lead to unopposed


alpha-adrenergic stimulation, thus exacerbating coronary artery vasospasm in patients with
MDMA overdose. In the event that blood pressure control is required, labetalol (a mixed
alpha- and beta-antagonist) could be used, although it isn’t a usual first-line option.

E Phenytoin

Phenytoin is incorrect. This patient isn’t suffering from seizures, and phenytoin isn’t
recommended for seizures related to MDMA overdose. Benzodiazepines are the usual
intervention.
51387

Rate this question:

Next Question

Previous Question Tag Question

Feedback End Review

Difficulty: Average

Peer Responses %
Q. Answered Flagged

Q1

Q2

Q3

Q4

Q5

Q6

Q7

Q8

Q9

0:02:24/03:00:00

A 67-year-old man with a history of lethargy, night sweats and chronic back pain over the last
18 months comes to the Emergency Department with nausea, vomiting, diarrhoea and
dehydration which have developed over the past four days. He has been investigated for
anaemia by his GP and has been prescribed ferrous sulphate tablets. His blood pressure is
95/60 mmHg, pulse is 92 bpm and regular. He looks pale.
Investigations:

Hb 92 g/l (MCV 89 fl)

WCC 10.5 × 10 9/l

PLT 110 × 10 9/l

NA + 144 mmol/l

K+ 5.9 mmol/l

Cr 239 µmol/l

Ca 2 2.95 mmol/l

Urine protein +++, blood -

Which of the following is the most likely cause of this patient’s renal impairment?

A Granulomatosis with polyangiitis (GP)

B Idiopathic membranous glomerulonephritis (IMG)

C Light chain deposition disease (LCDD)

D Post-renal failure (PoRF)

E Pre-renal failure (PRF)

Explanation 

C Light chain deposition disease (LCDD)


The normochromic, normocytic anaemia coupled with back pain, renal impairment and
hypercalcaemia raises the possibility of LCDD as a result of multiple myeloma. It’s likely the
patient has become dehydrated due to a gastrointestinal illness, precipitating the acute
presentation seen here. Chemotherapy for myeloma reduces light chain production and
decreases the risk of acute-on-chronic renal impairment.

A Granulomatosis with polyangiitis (GP)

GP is incorrect. GP is associated with evidence of renal vasculitis, evidenced by both


proteinuria and haematuria, rather than just marked proteinuria. Although hypercalcaemia
may be seen due to dehydration, the marked elevation in calcium seen here is more
consistent with multiple myeloma.

B Idiopathic membranous glomerulonephritis (IMG)

IMG is incorrect. IMG is associated with proteinuria in the absence of hypercalcaemia. Back
pain, in this case related to myeloma, would be unexpected.

D Post-renal failure (PoRF)

PoRF is incorrect. PoRF in a male is likely to be related to prostatic disease, either benign
prostatic hypertrophy or prostatic carcinoma. There are no symptoms to suggest either as
the underlying cause of this patient’s renal impairment.

E Pre-renal failure (PRF)

PRF is incorrect. PRF can occur as a result of a diarrhoeal illness, although the marked
proteinuria and hypercalcaemia seen here would be unexpected. The history of back pain and
anaemia also suggest a chronic condition such as myeloma underlying this event.
51427

Rate this question:

Next Question

Previous Question Tag Question

Feedback End Review

Difficulty: Average

Peer Responses %
Q. Answered Flagged

Q1

Q2

Q3

Q4

Q5

Q6

Q7

Q8

Q9

0:02:24/03:00:00

You are examining the results of a cardiovascular outcome trial where you are testing for
statistical significance with respect to cardiovascular death, non-fatal myocardial infarction or
stroke. A survival curve for the primary endpoint has been generated.
Which of the following is the most appropriate statistical test to determine if the difference
between the intervention and conventional therapy is significant?

A Cox regression analysis

B Chi-squared test

C Independent t-test

D Pearson correlation coefficient

E Spearman correlation coefficient

Explanation 

A Cox regression analysis

The difference between therapies with respect to cardiovascular risk is usually estimated
using a Cox regression model. The model generates a hazard ratio when the survival curves
for cardiovascular events are compared between therapies.

B Chi-squared test

Chi-squared test is incorrect. Chi-squared testing is most appropriate for comparing binary
outcomes. An example would be testing of a new cardiovascular stent and measuring re-
stenosis yes/no at a particular point in time.

C Independent t-test

Independent t-test is incorrect. A t-test tests the hypothesis that there is no difference
between the means of two different samples. An example of where a t-test would be useful is
in comparing mean bowel transit times in two groups of patients who are given different
bowel preparations.
D Pearson correlation coefficient

Pearson correlation coefficient is incorrect. The Pearson correlation coefficient looks for an
association between two continuous variables. An example where the Pearson test might be
of value would in looking for a correlation between waist circumference and blood pressure.

E Spearman correlation coefficient

Spearman correlation coefficient is incorrect. The Spearman coefficient tests for the strength
of the association between two ordinal variables; it does not rely on the assumption of
normally distributed data.
51408

Rate this question:

Next Question

Previous Question Tag Question

Feedback End Review

Difficulty: Average

Peer Responses %

Q. Answered Flagged

Q1

Q2

Q3

Q4

Q5
Q. Answered Flagged

Q6

Q7

Q8

0:02:24/03:00:00

A 32-year-old woman comes to the Early Pregnancy Clinic for review. She is 10 weeks’
pregnant with her first child. Current medication includes infliximab for rheumatoid arthritis
(she stopped methotrexate therapy after pre-pregnancy counselling). Her disease is currently
well controlled with no evidence of active inflammation, on musculoskeletal examination.
Which of the following is the best way to manage her rheumatoid disease in this pregnancy?

A Add azathioprine at week 16

B Continue infliximab throughout the pregnancy

C Switch infliximab for low-dose corticosteroids at week 16

D Stop infliximab at week 16

E Switch infliximab to etanercept at week 16

Explanation 

D Stop infliximab at week 16

Anti-tumour necrosis factor (anti-TNF) agents are generally considered safe in early
pregnancy, although they are either discontinued at the end of the second trimester, or in the
case of infliximab, at week 16. The risks of stopping the anti-TNF agent earlier are greater
with respect to the impact of worsening rheumatoid disease on the pregnancy than any risks
of continuing the therapy.

(see External Links)

A Add azathioprine at week 16

Add azathioprine at week 16 is incorrect. Although azathioprine is considered a safe option in


pregnancy, given this patient’s disease is well controlled, there is no indication to add it at
this stage.

B Continue infliximab throughout the pregnancy


Continue infliximab throughout the pregnancy is incorrect. Anti-TNF agents should not be
used in later pregnancy; in the case of infliximab, it should be discontinued at week 16.

C Switch infliximab for low-dose corticosteroids at week 16

Switch infliximab for low-dose corticosteroids at week 16 is incorrect. The benefit-risk for
continued low-dose corticosteroid therapy is not positive in pregnancy because of the
impact on weight gain and glucose tolerance.

E Switch infliximab to etanercept at week 16

Switch infliximab to etanercept at week 16 is incorrect. The other members of the anti-TNF
class apart from infliximab are contraindicated in the third trimester of pregnancy.
51460

Rate this question:

Next Question

Previous Question Tag Question

Feedback End Review

Difficulty: Average

Peer Responses %

Q. Answered Flagged

Q1

Q2

Q3
Q. Answered Flagged

Q4

Q5

Q6

Q7

Q8

 External Links

BSR and BHPR guideline on prescribing drugs in pregnancy and breastfeeding—Part I: standard and biolo…
academic.oup.com/rheumatology/article/55/9/1693/1744535
(https://academic.oup.com/rheumatology/article/55/9/1693/1744535)
0:02:24/03:00:00

A 26-year-old man with a history of human immunodeficiency virus (HIV) comes to the
Emergency Department with a heavy cough, a fever and left-sided pleuritic chest pain of 48
h duration. He is compliant with highly active antiretroviral therapy (HAART) and is currently
well, holding down a job as a junior solicitor. His temperature is 38.6°C, blood pressure is
100/60 mmHg, pulse is 98 bpm and regular. There are signs of right-sided consolidation on
auscultation of the chest.
Investigations:

Hb 119 g/l

WCC 11.9 × 10 9/l (neutrophilia)

CD4 count 398 cells/mm 3

PLT 219 × 10 9/l

Na + 141 mmol/l

K+ 4.9 mmol/l

Cr 99 µmol/l

Chest X-ray (CXR) right lower lobe consolidation

Which of the following is the most likely cause of this patient’s pneumonia?

A Klebsiella pneumoniae

B Mycoplasma pneumoniae

C Pneumocystis jirovecii

D Staphylococcus aureus

E Streptococcus pneumoniae

Explanation 

E Streptococcus pneumoniae
S. pneumoniae is the commonest cause of community-acquired pneumonia. Given the normal
CD4 count and the lobar consolidation coupled with neutrophilia, S. pneumoniae is the most
likely cause of this patient’s presentation. Co-amoxiclav and clarithromycin are the
intervention of choice.

A Klebsiella pneumoniae

K. pneumoniae is incorrect. K. pneumoniae is more common in patients with a history of


alcoholism and leads to cavitating pneumonia, rather than the clinical picture seen here with
acute lobar pneumonia without cavitation, against a background of HIV infection.

B Mycoplasma pneumoniae

M. pneumoniae is incorrect. M. pneumoniae is a less likely cause of community-acquired


pneumonia; it is associated with slowly progressive respiratory symptoms, with chest pain
and wheeze over a number of days. It is associated with erythema multiforme, and outbreaks
tend to occur in institutions, such as an army barracks.

C Pneumocystis jirovecii

P. jirovecii (PCP) is incorrect. PCP is seen most frequently in patients with a low CD4 count. It
is associated with patchy areas of consolidation and desaturation on walk testing.

D Staphylococcus aureus

S. aureus is incorrect. S. aureus is associated with cavitating pneumonia post influenza; it


doesn’t, therefore, fit with the picture seen here of community-acquired pneumonia without a
preceding viral illness.
51450

Rate this question:

Next Question

Previous Question Tag Question

Feedback End Review

Difficulty: Average

Peer Responses %
Q. Answered Flagged

Q1

Q2

Q3

Q4

Q5

Q6

Q7

Q8

Q9

0:02:24/03:00:00

A 19-year-old student comes to the clinic for review. He tells you that every year he has
terrible hay fever in springtime, and over the past two years has noticed itching and tingling
in his mouth when he eats uncooked apples. Over two successive years, he has required a
combination inhaler and oral steroids from his GP for extrinsic allergic asthma. Examination in
the clinic is unremarkable.
Which of the following is the most appropriate main intervention?

A Intramuscular (IM) triamcinolone

B Intermittent oral prednisolone

C Oral cetirizine

D Oral montelukast

E Oral roflumilast

Explanation 

C Oral cetirizine

This patient most likely has birch fruit syndrome, with seasonal asthma and allergic rhinitis
corresponding to the season of birch pollen, and a mild allergic reaction to proteins in
uncooked apple. An oral antihistamine is an appropriate initial intervention to reduce
seasonal symptoms.

A Intramuscular (IM) triamcinolone

IM triamcinolone is incorrect. Although potent corticosteroids are effective in abating


symptoms of seasonal asthma, they wouldn’t be advised here because of the long-term
impact of corticosteroids with respect to bone mineral density and glucose tolerance.

B Intermittent oral prednisolone


Intermittent oral prednisolone is incorrect. Courses of oral prednisolone are not
recommended unless patients fail to respond to oral antihistamines combined with inhaled
therapy.

D Oral montelukast

Oral montelukast is incorrect. Montelukast is recommended for patients with asthma who fail
to respond to low-dose inhaled corticosteroids alone. Those who fail a trial of montelukast
should be treated with inhaled corticosteroids in combination with a long-acting beta agonist
as a next step.

E Oral roflumilast

Oral roflumilast is incorrect. Oral roflumilast is a PDE4 inhibitor which is used for the
treatment of chronic obstructive pulmonary disease in patients who fail to respond to inhaled
therapies alone. It is known to cause gastrointestinal upset.
51451

Rate this question:

Next Question

Previous Question Tag Question

Feedback End Review

Difficulty: Average

Peer Responses %

Q. Answered Flagged

Q1

Q2
Q. Answered Flagged

Q3

Q4

Q5

Q6

Q7

Q8 
0:02:24/03:00:00

A 64-year-old man who has type 2 diabetes has been helping his daughter move house,
lifting a number of heavy boxes. Over the past two days, he has suffered progressively
increasing pain over his left shoulder and global limitation of movement. He is unable to sleep
on the left-hand side and still has pain despite maximal doses of paracetamol and ibuprofen.
Examination reveals global restriction of all movements, passive and active, of the left
shoulder.
Which of the following is the most likely diagnosis?

A Acromioclavicular (AC) joint disruption

B Adhesive capsulitis

C Brachial neuritis

D Cervical radiculopathy

E Supraspinatus tear

Explanation 

B Adhesive capsulitis

Adhesive capsulitis, known as frozen shoulder, is seen more commonly in patients with type 2
diabetes and fits with the global restriction of shoulder movements and pain after a period of
unaccustomed exercise. Non-steroidal anti-inflammatory drugs (NSAIDs) and physiotherapy
are the intervention of choice.

A Acromioclavicular (AC) joint disruption

AC joint disruption is incorrect. AC joint disruption is caused by a fall onto the outstretched
hand or trauma directly to the shoulder. It results in pain and instability of the shoulder,
particularly with overhead movements.

C Brachial neuritis
Brachial neuritis is incorrect. Acute brachial neuritis is characterised by acute severe pain in
the nerves of the brachial plexus, followed by weakness and eventual loss of sensation. Pain
can affect the shoulder, arm and the hand.

D Cervical radiculopathy

Cervical radiculopathy is incorrect. Cervical radiculopathy is characterised by neck pain and


nerve pain and loss of function along the nerve root where compression has occurred.

E Supraspinatus tear

Supraspinatus tear is incorrect. A supraspinatus tear leads to weakness and pain affecting
shoulder abduction, rather than the global restriction of movement and pain seen here.
51461

Rate this question:

Next Question

Previous Question Tag Question

Feedback End Review

Difficulty: Average

Peer Responses %

Q. Answered Flagged

Q1

Q2

Q3
Q. Answered Flagged

Q4

Q5

Q6

Q7

Q8
0:02:24/03:00:00

A 68-year-old woman is admitted with an exacerbation of chronic obstructive pulmonary


disease (COPD). Despite 28% O 2, intravenous (IV) hydrocortisone and back-to-back
nebulisers, her condition has deteriorated. Her blood pressure is 100/70 mmHg, her pulse is
85 bpm (atrial fibrillation). Her chest is hyper-expanded with bilateral poor air entry and quiet
wheeze on auscultation. She is alert and orientated, but becoming tired.
Investigations:

Hb 140 g/l

WCC 11.9 × 10 9/l

PLT 239 × 10 9/l

Na + 142 mmol/l

K+ 4.5 mmol/l

Bicarbonate 29 mmol/l

pO 2 8.9 kPa

pCO 2 8.3 kPa

pH 7.29

Chest X-ray (CXR) hyper-expanded lung fields; no evidence of focal infection

Which of the following is the most appropriate intervention?

A Increase inspired O2 to 35%

B Intubation and ventilation

C IV aminophylline

D IV doxapram

E Nasal intermittent positive pressure ventilation (NIPPV)

Explanation 
E Nasal intermittent positive pressure ventilation (NIPPV)

In this situation with respiratory acidosis and a deteriorating condition despite optimal
management of her COPD exacerbation, NIPPV is the recommended next step. When
consciousness has deteriorated further, or there is significant confusion, intubation and
ventilation is the only practical option.

A Increase inspired O2 to 35%

Increase inspired O 2 to 35% is incorrect. This won’t correct this patient’s respiratory acidosis
and risks further deterioration because of the vicious circle of worsening pH and subsequent
further fatigue.

B Intubation and ventilation

Intubation and ventilation is incorrect. Intubation and ventilation are an option for
deteriorating consciousness in a patient with a previous good functional status, where
recovery is thought possible. In this case, there is a window of opportunity for intervening
with NIPPV first.

C IV aminophylline

IV aminophylline is incorrect. IV aminophylline is used for acute bronchospasm in patients


where other options have been exhausted, although its use is now third or fourth line
because of the risk of atrial fibrillation associated with its use.

D IV doxapram

IV doxapram is incorrect. IV doxapram was previously used as a respiratory stimulant in


patients with COPD where there was no other option; however, outcomes are very poor with
doxapram, and it’s more important to decide between NIPPV, ventilation and palliation in this
situation.
51452

Rate this question:

Next Question

Previous Question Tag Question

Feedback End Review

Difficulty: Average
Peer Responses %

Q. Answered Flagged

Q1

Q2

Q3

Q4

Q5

Q6

Q7

Q8

Q9

0:02:24/03:00:00

A 45-year-old man is admitted to the Emergency Department with his partner following a
meal of tuna, mussels and white wine. They both have similar symptoms with shortness of
breath, stridor, hypotension and an erythematous skin rash. The male has a blood pressure of
95/60 mmHg, pulse is 98 bpm and regular. There is obvious stridor and facial flushing. He has
diarrhoea and abdominal cramps.
Which of the following is the most likely cause of their presentation?

A Anaphylaxis to fish proteins

B Bacillus cereus contamination

C Scombroid toxin

D Staphylococcus aureus toxin

E Tetrodotoxin

Explanation 

C Scombroid toxin

Scombroid toxin poisoning occurs because of accumulation of histamine in inappropriately


stored fish. Fish commonly implicated include tuna, mackerel, sardine, anchovy, herring,
bluefish and marlin. Antihistamines are the mainstay of therapy; intramuscular (IM) adrenalin
may be required in severe cases.

A Anaphylaxis to fish proteins

Anaphylaxis to fish proteins is incorrect. Although allergy to fish proteins may present with
similar symptoms, it would be highly unusual for two unrelated patients to present with
symptoms at the same time.

B Bacillus cereus contamination


Bacillus cereus contamination is incorrect. B. cereus contamination occurs in rice and leads to
either profuse vomiting or diarrhoea, depending on the subtype of the bacteria. It doesn’t
present with the symptoms of histamine excess seen here.

D Staphylococcus aureus toxin

Staphylococcus aureus toxin is incorrect. S. aureus toxin is found in dairy products that are
kept standing at room temperature. It leads to vomiting a few hours after ingestion of the
food stuff.

E Tetrodotoxin

Tetrodotoxin is incorrect. Tetrodotoxin is a nerve toxin which is found in incorrectly prepared


puffer fish. It results in a small number of deaths in the Far East each year.
51388

Rate this question:

Next Question

Previous Question Tag Question

Feedback End Review

Difficulty: Average

Peer Responses %

Q. Answered Flagged

Q1

Q2

Q3
Q. Answered Flagged

Q4

Q5

Q6

Q7

Q8 
0:02:24/03:00:00

A 32-year-old woman is referred to the Cardiology Clinic. She presented some three months
earlier with atrial fibrillation and gradually worsening shortness of breath. An echocardiogram
has revealed severe dilated cardiomyopathy. On further questioning, it transpires that her
father died of the same condition at 45 years of age. She has a sister who is 36 years of age
who has no symptoms of cardiomyopathy.
Which of the following is the likely mode of inheritance for her cardiomyopathy?

A Autosomal dominant

B Autosomal recessive

C X-linked dominant

D X-linked recessive

E Uniparental disomy

Explanation 

A Autosomal dominant

Around 80–90% of cases of familial dilated cardiomyopathy follow an autosomal dominant


inheritance pattern. The condition does exhibit variable penetrance, which may explain why
her sister does not suffer symptoms of heart failure. Autosomal recessive and X-linked forms
of the condition are also described.

B Autosomal recessive

Autosomal recessive is incorrect. Only around 10% of cases of dilated cardiomyopathy are
inherited in autosomal recessive fashion; the fact that this patient and her father are affected
makes autosomal recessive inheritance less likely here.

C X-linked dominant
X-linked dominant is incorrect. Only a few conditions follow an X-linked dominant inheritance
pattern. One example is Alport syndrome, where both males and females are affected by the
condition, although males appear to suffer deafness and renal impairment with greater
severity.

D X-linked recessive

X-linked recessive is incorrect. In X-linked recessive conditions, the majority of females do not
have clinical symptoms or signs. Examples include red–green colour blindness, haemophilia A
& B, Duchenne’s and Becker’s muscular dystrophies.

E Uniparental disomy

Uniparental disomy is incorrect. Uniparental disomy occurs when only one of a pair of
chromosomes (the maternal or paternal one) is active with respect to gene expression. An
example of a disease where uniparental disomy is exhibited is Prader–Willi syndrome.
51372

Rate this question:

Next Question

Previous Question Tag Question

Feedback End Review

Difficulty: Average

Peer Responses %

Q. Answered Flagged

Q1

Q2
Q. Answered Flagged

Q3

Q4

Q5

Q6

Q7

Q8 
0:02:24/03:00:00

A 45-year-old man is reviewed on the ward prior to discharge, a few days after suffering a
large variceal haemorrhage, for which he received banding. He will abstain from alcohol and
attend follow-up endoscopy appointments for prophylactic banding therapy.
Which of the following is the most important long-term intervention to reduce the risk of
further bleeding?

A Diltiazem

B Furosemide

C Propranolol

D Spironolactone

E Terlipressin

Explanation 

C Propranolol

Non-selective beta-blockers such as propranolol are recognised to reduce portal pressure. By


doing this, they reduce the risk of new varices forming and further upper gastrointestinal
haemorrhage occurring.

(see External Links)

A Diltiazem

Diltiazem is incorrect. Diltiazem is a cardioselective calcium channel antagonist; its primary


role is in the treatment of angina in patients where other agents are unsuitable or are poorly
tolerated.

B Furosemide

Furosemide is incorrect. Many patients with significant hepatic impairment are hypokalaemic
and suffer from disordered aldosterone metabolism. For this reason, aldosterone antagonists
such as spironolactone are preferred to loop diuretics.
D Spironolactone

Spironolactone is incorrect. Spironolactone is useful to reduce fluid accumulation associated


with hepatic impairment, although it doesn’t significantly reduce portal pressure compared
with propranolol.

E Terlipressin

Terlipressin is incorrect. Terlipressin is a vasopressin analogue used in the treatment of acute


variceal haemorrhage. It doesn’t have a role in chronic treatment of oesophageal varices.
51375

Rate this question:

Next Question

Previous Question Tag Question

Feedback End Review

Difficulty: Average

Peer Responses %

Q. Answered Flagged

Q1

Q2

Q3

Q4

Q5
Q. Answered Flagged

Q6

Q7

Q8 

 External Links

Resuscitation Council (UK): Resuscitation guidelines


resus.org.uk/resuscitation-guidelines/
(https://www.resus.org.uk/resuscitation-guidelines/)
0:02:24/03:00:00

A 42-year-old man comes to the clinic for review. He has noted problems with blurring of his
vision, particularly when he enters a brightly lit room. He has also noticed when he looks in a
mirror that his right pupil is slightly larger than the left. On examination, the right pupil reacts
only very slowly to light but has a normal reaction to accommodation. The left pupil and all
eye movements appear normal.
Where is the most likely site of the lesion causing this patient’s symptoms?

A Lateral lemniscus

B Medial lemniscus

C Medial longitudinal fasciculus (MLF)

D Parasympathetic ciliary ganglion

E Superior cerebellar peduncle

Explanation 

D Parasympathetic ciliary ganglion

This patient has a right Holmes Adie pupil, which occurs because of damage to the
parasympathetic ciliary ganglion. The parasympathetic innervation to both the iris and ciliary
body travels with the third cranial nerve and synapses at the ciliary ganglion. After damage
to the ganglion, re-innervation and upregulation of the postsynaptic receptors occurs; this is
known as denervation supersensitivity. This results in the aberrant pupillary reaction to light.
It is most commonly found in younger women.

A Lateral lemniscus

Lateral lemniscus is incorrect. The lateral lemniscus carries sound information from the
cochlear nucleus to the inferior colliculus, medial geniculate body, mid-brain reticular
formation and the superior olivary complex.

B Medial lemniscus
Medial lemniscus is incorrect. The medial lemniscus carries information from the posterior
columns. Interruption of the medial lemniscus leads to impairment of vibration and fine touch
sensation.

C Medial longitudinal fasciculus (MLF)

MLF is incorrect. Lesions of the MLF lead to slowed or absent adduction of the ipsilateral eye,
this is usually associated with nystagmus of the abducting eye, the clinical features are
known as internuclear ophthalmoplegia.

E Superior cerebellar peduncle

Superior cerebellar peduncle is incorrect. It consists mainly of efferent fibres which run from
a cerebellar hemisphere to the contralateral thalamus and the cerebellorubral tract that runs
from cerebellar hemisphere to the red nucleus.
51442

Rate this question:

Next Question

Previous Question Tag Question

Feedback End Review

Difficulty: Average

Peer Responses %

Q. Answered Flagged

Q1

Q2
Q. Answered Flagged

Q3

Q4

Q5

Q6

Q7

Q8
0:02:24/03:00:00

A 55-year-old woman comes to the Dermatology Clinic for review of extensive eczema. She
has a history of chronic obstructive pulmonary disease (COPD) and maintains that she has
stopped smoking, although her husband tells you she still smokes four to five cigarettes per
day. She is prescribed a paraffin-based emollient and a low-potency topical steroid cream.
Which of the following is the most important advice to give her?

A Apply the emollient after bathing

B Apply the steroid cream sparingly

C Not to smoke whilst using the emollient

D She gently warm the emollient to make it easier to apply

E Wash hands after applying the steroid cream

Explanation 

C Not to smoke whilst using the emollient

Several case reports exist of patients catching fire/suffering significant burns when using
paraffin-based skin emollients and continuing to smoke. This has led to specific advice from
National Health Service (NHS) improvement to reinforce the need to avoid smoking in all
circumstances when using emollient products.

A Apply the emollient after bathing

Apply the emollient after bathing is incorrect. Although it is most useful to apply the
emollient after bathing, the advice about not smoking is most important for this patient.

B Apply the steroid cream sparingly

Apply the steroid cream sparingly is incorrect. Low-potency corticosteroids represent much
less of a risk of systemic absorption and adrenal suppression than higher-potency products.
Advise patient to apply the cream only to lesions is important but again, it’s less important
than avoiding smoking.
D She gently warm the emollient to make it easier to apply

She should gently warm the emollient to make it easier to apply is incorrect. This again runs
the risk of introducing paraffin vapour into the air and possible ignition.

E Wash hands after applying the steroid cream

Wash hands after applying the steroid cream is incorrect. This patient may well have eczema
lesions on her hands; in this case, it’s probably appropriate not to wash hands after applying
the cream to other areas of the body. In any case, the risks from systemic absorption are low
for a low-potency cream and less important than risks of smoking-related ignition of the
emollient.
51411

Rate this question:

Next Question

Previous Question Tag Question

Feedback End Review

Difficulty: Average

Peer Responses %

Q. Answered Flagged

Q1

Q2

Q3

Q4
Q. Answered Flagged

Q5

Q6

Q7

Q8 
0:02:24/03:00:00

A 35-year-old man who is known to have autosomal-dominant polycystic kidney disease


(ADPKD), presents to the Emergency Department with rapidly worsening left flank pain and
haematuria. He says a dull ache came on all of a sudden and hasn’t been relieved by
paracetamol and codeine. His temperature is 37.6°C, blood pressure is 155/89 mmHg, pulse is
92 bpm and regular. He is obviously in pain. There is marked left flank tenderness on
palpation and obvious bilateral cystic renal masses.
Investigations:

Hb 98 g/l

WCC 10.0 × 10 9/l

PLT 203 × 10 9/l

Na + 142 mmol/l

K+ 4.5 mmol/l

Cr 249 µmol/l

Urine blood +++

Which of the following is the most likely cause of this man’s acute pain?

A Cyst haemorrhage

B Renal artery embolism

C Renal vasculitis

D Renal vein thrombosis

E Urinary tract infection

Explanation 

A Cyst haemorrhage
Up to 70% of patients with polycystic kidney disease suffer haemorrhage into a renal cyst.
Pain results from distension of the renal capsule and haematuria results from a cyst into the
collecting system. Management is conservative, with intravenous (IV) hydration and analgesia
avoiding the use of non-steroidal anti-inflammatory drugs (NSAIDs).

B Renal artery embolism

Renal artery embolism is incorrect. Renal artery embolism can result in haematuria, and is
associated with flank pain, although there are no risk factors for arterial embolism in this
patient’s history and examination, and given the history of polycystic kidney disease, a cyst
haemorrhage is much more likely.

C Renal vasculitis

Renal vasculitis is incorrect. Both haematuria and proteinuria would be expected in


association with renal vasculitis. Other features associated with connective tissue disease
would also be expected which are absent here.

D Renal vein thrombosis

Renal vein thrombosis is incorrect. Renal vein thrombosis is associated with flank pain, but is
usually associated with a marked increase in proteinuria (not seen here), and is most often
seen in patients with risk factors for venous thrombosis.

E Urinary tract infection

Urinary tract infection is incorrect. Haematuria alone would be unexpected in urinary tract
infection, and flank pain in the absence of supra-pubic pain, as in indication of ascending
urinary tract infection, would be very unusual.
51428

Rate this question:

Next Question

Previous Question Tag Question

Feedback End Review

Difficulty: Average

Peer Responses %
Q. Answered Flagged

Q1

Q2

Q3

Q4

Q5

Q6

Q7

Q8

Q9

0:02:24/03:00:00

An 18-year-old man comes to the Emergency Department complaining of facial puffiness and
frothy urine over the past two weeks. He says that this has happened to him twice in the past
and that on each occasion, his symptoms responded well to high-dose oral prednisolone.
Examination reveals a blood pressure of 122/80 mmHg; pulse is 85 bpm and regular. There is
evidence of peri-orbital oedema. Serum creatinine is in the normal range, and there is 3+
protein on urine dipstick.
Which of the following is the most appropriate next intervention?

A Chlorambucil

B Cyclophosphamide

C High-dose prednisolone

D Mycophenolate mofetil

E Rituximab

Explanation 

C High-dose prednisolone

This patient is most likely to have minimal change disease, which presents with recurrent
proteinuria, usually against a background of normal blood pressure and serum creatinine.
Approximately 90% of children respond within two weeks of the introduction of
prednisolone, although a longer response time is usually seen in adults. Where there is
incomplete resolution of proteinuria, a range of second-line agents can be used.

A Chlorambucil

Chlorambucil is incorrect. Chlorambucil is a commonly used second-line agent for


membranous nephropathy. It carries significant risk of bone marrow suppression, meaning
that it isn’t normally used in the treatment of minimal change disease.

B Cyclophosphamide
Cyclophosphamide is incorrect. Cyclophosphamide is a conventional second-line agent of
choice; when given at a dose of 2 mg/kg/day for 8–12 weeks, it improves remission rates in
adults with minimal change disease.

D Mycophenolate mofetil

Mycophenolate mofetil is incorrect. Mycophenolate is an alternative second-line agent to


cyclophosphamide in adults who fail to respond to corticosteroids alone. It is a first-line
agent for the treatment of lupus nephritis.

E Rituximab

Rituximab is incorrect. Rituximab is the biological agent having the most evidence for
preventing relapse in patients with frequent episodes of minimal change disease, which can
be defined as four episodes within any period of 12 months. It acts via the CD20 receptor,
depleting B cells.

(see External Links)


51429

Rate this question:

Next Question

Previous Question Tag Question

Feedback End Review

Difficulty: Average

Peer Responses %

Q. Answered Flagged

Q1
Q. Answered Flagged

Q2

Q3

Q4

Q5

Q6

Q7

Q8 

 External Links

Minimal change disease and focal segmental glomerulosclerosis in adults: rituximab


nice.org.uk/advice/es1/resources/minimal-change-disease-and-focal-segmental-glomerulosclerosis-in-adults-rituxima…
(https://www.nice.org.uk/advice/es1/resources/minimal-change-disease-and-focal-
segmental-glomerulosclerosis-in-adults-rituximab-pdf-32169639877)
0:02:24/03:00:00

An 18-year-old man from a travelling family presents to the Emergency Department with
severe headaches. He tells you that these have been increasing significantly over the past
three months. Examination reveals a blood pressure of 155/95 mmHg, pulse is 75 bpm and
regular. His weight is normal with a body mass index of 23 kg/m 2.
Investigations:

Hb 121 g/l

WCC 7.5 × 10 9/l

PLT 195 × 10 9/l

Na + 145 mmol/l

K+ 3.2 mmol/l

Bicarbonate 33 mmol/l

Cr 112 µmol/l

Which of the following is the most likely diagnosis?

A Bartter syndrome

B Chronic reflux nephropathy

C Cushing syndrome

D Gitelman syndrome

E Liddle syndrome

Explanation 

E Liddle syndrome

Liddle syndrome is a congenital form of hypertension with hypokalaemic metabolic alkalosis.


It is autosomal dominant and results from mutations in genes on chromosome 16 that encode
the beta and gamma subunits of the collecting tubule sodium channel. Amiloride or
triamterene are interventions of choice, spironolactone is ineffective in this case.
A Bartter syndrome

Bartter syndrome is incorrect. Bartter’s is typically diagnosed by school age. It results in


hypokalaemic metabolic alkalosis, although it is associated with a normal blood pressure.
Bartter’s type 1 results from a mutation in the SLC12A1 gene, which codes for the sodium,
potassium and chloride cotransporter.

B Chronic reflux nephropathy

Chronic reflux nephropathy is incorrect. Chronic reflux nephropathy is associated with


hypertension and chronic renal impairment and occurs against a background of recurrent
urinary tract infection, usually in childhood.

C Cushing syndrome

Cushing syndrome is incorrect. Cushing syndrome results in hypertension with hypokalaemic


metabolic alkalosis, although other features of Cushing’s, including obesity and impaired
glucose tolerance, are absent.

D Gitelman syndrome

Gitelman syndrome is incorrect. Like Bartter syndrome, Gitelman’s is associated with


hypokalaemic metabolic alkalosis. Gitelman’s occurs most commonly from mutations in the
SLC12A3 gene, which codes for the sodium chloride cotransporter.
51430

Rate this question:

Next Question

Previous Question Tag Question

Feedback End Review

Difficulty: Average

Peer Responses %
Q. Answered Flagged

Q1

Q2

Q3

Q4

Q5

Q6

Q7

Q8

Q9

0:02:24/03:00:00

A 19-year-old man from a student hall of residence is admitted to the Emergency Department
with left-sided pleuritic chest pain and shortness of breath, accompanied by a fever, which
has steadily increased over the past 48 h. He has felt progressively more unwell over the past
seven days. Apparently, three others from the hall of residence have similar symptoms. His
temperature is 38°C, blood pressure is 105/70 mmHg, and pulse is 89 bpm and regular. There
is wheeze on auscultation, more marked on the left than the right. You notice target lesions
on the upper and lower limbs.
Investigations:

Hb 135 g/l

WCC 11.2 × 10 9/l

PLT 203 × 10 9/l

CRP 182 mg/l

Na + 140 mmol/l

K+ 4.5 mmol/l

Cr 112 µmol/l

Chest X-ray (CXR) left lower lobe consolidation

Which of the following is the most likely cause of his pneumonia?

A Chlamydia pneumoniae

B Klebsiella pneumoniae

C Mycoplasma pneumoniae

D Staphylococcus aureus

E Streptococcus pneumoniae

Explanation 

C Mycoplasma pneumoniae
M. pneumonia occurs in outbreaks, which fits well with the increase in cases from a hall of
residence reported here. Symptoms and CXR changes are often more severe than signs
heard on auscultation, and patients may become progressively more unwell over a number of
days. Erythema multiforme (the cause of target lesions) is commonly associated with
mycoplasma infection. Macrolide antibiotics such as clarithromycin are the intervention of
choice.

A Chlamydia pneumoniae

C. pneumoniae is incorrect. Although this presents with a gradual build-up of symptoms


similar to mycoplasma, features of an upper respiratory tract infection, including coryzal
symptoms, are common early in the condition. Both macrolides and tetracyclines are
potential interventions.

B Klebsiella pneumoniae

K. pneumoniae is incorrect. Klebsiella occurs more commonly in patients with


immunocompromise and those with a history of alcoholism. It leads to cavitation and
potentially empyema formation.

D Staphylococcus aureus

S. aureus is incorrect. S. aureus occurs most commonly post influenza infection. Like
klebsiella, it is a cause of cavitating pneumonia. Flucloxacillin is the intervention of choice.

E Streptococcus pneumoniae

S. pneumoniae is incorrect. Streptococcal infection is the commonest cause of community-


acquired pneumonia. It is associated with herpes simplex activation and with the
development of lobar pneumonia. Symptoms are usually more acute than the gradual build-
up reported here.
51453

Rate this question:

Next Question

Previous Question Tag Question

Feedback End Review

Difficulty: Average
Peer Responses %

Q. Answered Flagged

Q1

Q2

Q3

Q4

Q5

Q6

Q7

Q8

Q9

0:02:24/03:00:00

A 19-year-old man from a travelling family comes to the Emergency Department with
symptoms of a severe sore throat, neck stiffness, headache, and pyrexia. He says these
symptoms have significantly worsened over the past 36 h. He did not receive routine
vaccinations as a child. He is pyrexial at 38.9°C, there is evidence of pharyngitis, bilateral
parotitis and meningism. A lumbar puncture reveals a lymphocytosis, and routine blood tests
reveal evidence of hepatitis.
Which of the following is the most likely diagnosis?

A Cytomegalovirus (CMV) infection

B Epstein–Barr virus infection

C Herpes simplex infection

D Herpes zoster infection

E Paramyxovirus infection

Explanation 

E Paramyxovirus infection

This patient has symptoms of mumps, caused by paramyxovirus infection. It is likely that he
didn’t receive the measles, mumps and rubella (MMR) vaccination as a child, accounting for
his presentation. Parotitis (seen here) may be absent in up to 50% of patients with central
nervous system (CNS) symptoms of infection. Treatment is supportive.

A Cytomegalovirus (CMV) infection

CMV infection is incorrect. Although CMV infection may be associated with CNS infection
with a lymphocytic picture, it wouldn’t fit with the features of parotitis seen here. The
pharyngitis and hepatitis would, however, fit.

B Epstein–Barr virus infection


Epstein–Barr virus infection is incorrect. Epstein–Barr virus infection is associated with
symptoms of glandular fever, including pharyngitis, hepatitis and splenic pain, but not with
the parotitis seen here.

C Herpes simplex infection

Herpes simplex infection is incorrect. Herpes simplex encephalitis isn’t associated with
parotitis. It may, however, lead to a lymphocytosis on cerebrospinal fluid (CSF) examination.
Gradually decreasing consciousness and impairment of short-term memory are usual
features.

D Herpes zoster infection

Herpes zoster infection is incorrect. Primary zoster infection leading to chicken pox is
associated with encephalitis, particularly when it occurs in older individuals. It isn’t associated
with parotitis.
51425

Rate this question:

Next Question

Previous Question Tag Question

Feedback End Review

Difficulty: Average

Peer Responses %

Q. Answered Flagged

Q1

Q2
Q. Answered Flagged

Q3

Q4

Q5

Q6

Q7

Q8 
0:02:24/03:00:00

A 45-year-old woman with a history of bipolar disease is admitted to the Emergency


Department with nausea, vomiting and decreased consciousness. She is treated with regular
lithium therapy and has recently been started on a thiazide diuretic by her GP. Her blood
pressure is 115/80 mmHg, pulse is 95 bpm and regular. Her Glasgow Coma Scale (GCS) is 11.
She has irregular coarse tremors and you note the presence of myoclonic jerks.
Investigations:

Hb 134 g/l

WCC 10.9 × 10 9/l

PLT 302 × 10 9/l

Na + 145 mmol/l

K+ 5.2 mmol/l

Cr 182 µmol/l

Lithium 4.9 mmol/l

Which of the following is the most appropriate intervention?

A Diphenhydramine

B Haemodialysis

C Oral activated charcoal

D Polyethylene glycol (PEG) bowel irrigation

E Sodium polystyrene sulphonate

Explanation 

B Haemodialysis

This patient has severe lithium toxicity. Where lithium concentration is greater than 4 mmol/l
with abnormal renal function, or > 5 mmol/l per se, haemodialysis to remove lithium is
indicated. Where hypotension prevents haemodialysis, fluid resuscitation is indicated as
dehydration often co-exists with lithium toxicity.

A Diphenhydramine

Diphenhydramine is incorrect. Diphenhydramine is a sedating antihistamine; it actually


reduces the seizure threshold in patients treated with lithium and has no role in the treatment
of lithium toxicity.

C Oral activated charcoal

Oral activated charcoal is incorrect. Activated charcoal does not bind lithium and has no role
in the treatment of lithium overdose.

D Polyethylene glycol (PEG) bowel irrigation

PEG bowel irrigation is incorrect. PEG irrigation is useful in patients with large ingestions of
lithium or acute lithium overdose. It is most effective given within 2–4 h after overdose and is
administered via nasogastric tube.

E Sodium polystyrene sulphonate

Sodium polystyrene sulphonate is incorrect. Sodium polystyrene sulphonate can be used as


an adjunct to supportive care in lithium overdose, although it needs to be given in repeated
doses and may result in symptomatic hypokalaemia.
51389

Rate this question:

Next Question

Previous Question Tag Question

Feedback End Review

Difficulty: Average

Peer Responses %
Q. Answered Flagged

Q1

Q2

Q3

Q4

Q5

Q6

Q7

Q8

Q9

0:02:24/03:00:00

A 71-year-old man is admitted to the Emergency Department having been found on the floor
after a fall. He is found to have a fractured left neck of femur, is hypotensive and dehydrated
with extensive bruising and pressure areas affecting his left side. On examination, his blood
pressure is 95/60 mmHg, his pulse is 98 bpm and regular. Urine is 3+ positive for blood. An
initial serum creatinine is 185 µmol/l and his initial creatine kinase (CK) is 24 195 IU.
Which of the following is the most important intervention?

A Intravenous (IV) furosemide

B IV mannitol

C IV normal saline

D IV sodium bicarbonate

E Oral allopurinol

Explanation 

C IV normal saline

The most important intervention in rhabdomyolysis is adequate IV hydration. The goals of


volume expansion are to maintain or enhance renal blood flow, minimizing ischemic injury,
and increasing urine flow rate. This will limit intratubular cast formation by diluting the
concentration of haem pigment within tubular fluid, wash out partially obstructing
intratubular casts, and increase urinary potassium excretion.

A Intravenous (IV) furosemide

IV furosemide is incorrect. Loop diuretics may drive volume depletion, and worsen
hypocalcaemia, which is recognised as occurring in association with rhabdomyolysis. As such,
they should be avoided in rhabdomyolysis unless there is significant volume overload.

B IV mannitol
IV mannitol is incorrect. Although experimental studies in animals suggest a possible benefit
of mannitol in driving a diuresis and minimising intratubular haem deposition, there is no
evidence in humans that it ameliorates proximal tubular necrosis.

D IV sodium bicarbonate

IV sodium bicarbonate is incorrect. Bicarbonate may be of value in patients with a rising CK


but it should only be considered in the absence of hypocalcaemia, and where the pH is less
than 7.5 and the serum bicarbonate is less than 30 mmol/l.

E Oral allopurinol

Oral allopurinol is incorrect. Where hyperuricaemia is seen in conjunction with


rhabdomyolysis, oral allopurinol is an intervention of choice with IV fluid replacement,
although it isn’t the primary therapy for rhabdomyolysis.
51431

Rate this question:

Next Question

Previous Question Tag Question

Feedback End Review

Difficulty: Average

Peer Responses %

Q. Answered Flagged

Q1

Q2
Q. Answered Flagged

Q3

Q4

Q5

Q6

Q7

Q8
0:02:24/03:00:00

A 74-year-old woman with advanced chronic obstructive pulmonary disease (COPD) and two
previous myocardial infarctions comes to the clinic for review. She is suffering from recurrent
uterine prolapse which is leading to significant discomfort, made worse by chronic coughing.
She is able to walk only 50 metres without stopping on the flat and requires home oxygen.
Which of the following is the most appropriate intervention?

A Hormone replacement therapy (HRT)

B Mesh repair

C Oestrogen cream

D Ring pessary

E Tolterodine

Explanation 

D Ring pessary

A ring pessary is a local solution to prevent further prolapse in patients like this one who are
unfit to undergo surgery. As large a pessary as is comfortable is usually inserted. The pessary
fits well if a finger can be swept between the pessary and the walls of the vagina.

A Hormone replacement therapy (HRT)

HRT is incorrect. HRT, both locally and systemically delivered, has less impact on symptoms
of prolapse than mechanical or surgical solutions. Systemic HRT also carries significant risks
in a 74-year old.

B Mesh repair

Mesh repair is incorrect. Mesh repair is now much less popular as an option for genitourinary
prolapse due to women suffering a chronic inflammatory response to the mesh and in some
cases, migration through body structures, leading to chronic pain.
C Oestrogen cream

Oestrogen cream is incorrect. Oestrogen cream has been shown to have limited impact on
symptoms of genitourinary prolapse; as such, it is best avoided here.

E Tolterodine

Tolterodine is incorrect. Tolterodine is an anti-muscarinic, it is used for symptoms of


overactive bladder, it is therefore of no value in the treatment of genitourinary prolapse.
51447

Rate this question:

Next Question

Previous Question Tag Question

Feedback End Review

Difficulty: Average

Peer Responses %

Q. Answered Flagged

Q1

Q2

Q3

Q4

Q5

Q6
Q. Answered Flagged

Q7

Q8

0:02:24/03:00:00

A 38-year-old woman presents to the Emergency Department with a history of intermittent


right-sided pleuritic chest pain which has steadily increased over the past month. She had a
left lower limb deep vein thrombosis one year ago and has now been diagnosed with
systemic lupus erythematosus (SLE). Examination reveals a blood pressure of 135/85 mmHg,
pulse is 95 bpm and regular. Her chest is clear. Abdomen is soft and non-tender, body mass
index (BMI) is 29 kg/m 2.
Investigations:

Hb 119 g/l

WCC 10.8 × 10 9/l

PLT 203 × 10 9/l

Na + 144 mmol/l

K+ 4.5 mmol/l

Cr 102 µmol/l

Chest X-ray (CXR) lung fields clear

PaO 2 7.8 kPa

PaCO 2 3.8 kPa

pH 7.40

Forced vital capacity (FVC) 3.6 l/min

Forced expiratory volume at 1 second (FEV 1) 3.2 l/min

Which of the following is the most likely cause of her symptoms?

A Asthma

B Chronic obstructive pulmonary disease (COPD)

C Obesity hypoventilation syndrome

D Pulmonary embolism (PE)

E Pulmonary fibrosis
Explanation 

D Pulmonary embolism (PE)

There is a significant increase in the A–a gradient, with tachycardia and pleuritic chest pain, in
the absence of any focal chest signs. This fits with a diagnosis of PE. Indeed, this patient is at
increased risk of PE given her overweight and history of SLE. A computed tomography (CT)
pulmonary angiography is the most appropriate next step.

A Asthma

Asthma is incorrect. The FVC and FEV 1 are in the normal range for this patient’s age and sex,
and there is no history of cough or wheeze, making asthma extremely unlikely as the cause of
this patient’s symptoms.

B Chronic obstructive pulmonary disease (COPD)

COPD is incorrect. Both the FVC and FEV 1 are in the normal range and this patient is young
with no history of smoking, making a diagnosis of COPD extremely unlikely.

C Obesity hypoventilation syndrome

Obesity hypoventilation syndrome is incorrect. BMI is just below the obesity range (29) and
spirometry is in the normal range; this makes obesity hypoventilation syndrome very unlikely
as the cause of this patient’s symptoms.

E Pulmonary fibrosis

Pulmonary fibrosis is incorrect. Although SLE is associated with pulmonary fibrosis, the FEV 1
and FVC are in the normal range and there are no findings on the CXR to suggest pulmonary
fibrosis. PE is therefore much more likely to be the cause of this patient’s symptoms.
51454

Rate this question:

Next Question

Previous Question Tag Question

Feedback End Review


Difficulty: Average

Peer Responses %

Q. Answered Flagged

Q1

Q2

Q3

Q4

Q5

Q6

Q7

Q8

Q9

0:02:24/03:00:00

You are asked to see a 54-year-old man on the Orthopaedic Ward, who is recovering two
days after surgery for a complex fracture of his right wrist. He apparently fell whilst walking
up steps at his home. He has suffered a number of fractured ribs in the past and was
admitted on one previous occasion for alcohol intoxication. He is anxious and sweating and
has been agitated and angry when asked by nursing staff to quieten down and rest. His blood
pressure is 145/85 mmHg, pulse is 95 bpm and regular. He claims that something is crawling
on his lower legs and he can see insects on the ward.
Which of the following is the most appropriate intervention?

A Carbamazepine

B Chlordiazepoxide

C Risperidone

D Ropinirole

E Thiamine

Explanation 

B Chlordiazepoxide

This patient is almost certainly suffering from acute alcohol withdrawal for which a reducing
course of chlordiazepoxide is the most appropriate intervention. Most hospitals now use a
Clinical Institute Withdrawal Assessment (CIWA) protocol to determine the frequency of
dosing. In the event that chlordiazepoxide is unavailable, other benzodiazepines can be
substituted. A dose of up to 20 mg four times a day (qds) is usually required over the first 24
h of admission. The admission with intoxication and multiple fractures are strong pointers
towards habitual alcohol use.

A Carbamazepine

Carbamazepine is incorrect. Although carbamazepine may be used in the event that alcohol
withdrawal results in seizures, it is not the first-line intervention for management of alcohol
withdrawal symptoms.
C Risperidone

Risperidone is incorrect. Both traditional antipsychotic agents and atypical drugs like
risperidone are inappropriate in the management of alcohol withdrawal symptoms.

D Ropinirole

Ropinirole is incorrect. The tactile hallucinations seen here are a feature of alcohol
withdrawal. Ropinirole is used in the management of restless legs syndrome but doesn’t have
a role in the treatment of symptoms here.

E Thiamine

Thiamine is incorrect. Although thiamine is required in the treatment of patients with chronic
alcoholism, to prevent the development of Wernicke’s encephalopathy, it won’t impact on his
current symptoms.
51390

Rate this question:

Next Question

Previous Question Tag Question

Feedback End Review

Difficulty: Average

Peer Responses %

Q. Answered Flagged

Q1

Q2
Q. Answered Flagged

Q3

Q4

Q5

Q6

Q7

Q8

0:02:24/03:00:00

A 39-year-old man comes to the Renal Clinic for review. He is hypertensive despite treatment
with both ramipril 10 mg daily and indapamide 2.5 mg daily. A computed tomography (CT) of
his abdomen has revealed a left adrenal adenoma. His blood pressure is 152/90 mmHg, pulse
is 70 bpm and regular. His body mass index is 22 kg/m 2.
Investigations:

Hb 139 g/l

WCC 5.9 × 10 9/l

PLT 191 × 10 9/l

Na + 144 mmol/l

K+ 3.1 mmol/l

Cr 102 µmol/l

Bicarbonate 32 mmol/l

Which of the following is the most appropriate intervention for blood pressure control?

A Amiloride

B Amlodipine

C Bisoprolol

D Spironolactone

E Valsartan

Explanation 

D Spironolactone

This patient has primary hyperaldosteronism due to an adrenal adenoma, as evidenced by the
hypokalaemic metabolic alkalosis against the background of a normal body mass index. Prior
to surgery, improved blood pressure control is crucial, and in this regard, a direct aldosterone
antagonist such as spironolactone is the intervention of choice. In patients with symptomatic
gynaecomastia, this can be switched to eplerenone.

A Amiloride

Amiloride is incorrect. Although amiloride is a potassium-sparing diuretic, it lacks


mineralocorticoid inhibition and is a weak anti-hypertensive. For this reason, spironolactone
or eplerenone are preferred.

B Amlodipine

Amlodipine is incorrect. Calcium antagonists in this situation will aid control of hypertension,
but won’t impact significantly on the hyperkalaemia.

C Bisoprolol

Bisoprolol is incorrect. Beta-blockade is most useful in hypertension against a background of


ischaemic heart disease. However, in this situation, bisoprolol isn’t the most useful option,
given that it isn’t an aldosterone antagonist.

E Valsartan

Valsartan is incorrect. Although valsartan will increase potassium when used in combination
with ramipril, a direct aldosterone antagonist such as spironolactone will have greater impact
with respect to achieving blood pressure control.
51432

Rate this question:

Next Question

Previous Question Tag Question

Feedback End Review

Difficulty: Average

Peer Responses %
Q. Answered Flagged

Q1

Q2

Q3

Q4

Q5

Q6

Q7

Q8

Q9

0:02:24/03:00:00

A 62-year-old man working as a taxi driver has a 7-year history of type 2 diabetes and comes
to the clinic for review. Current medication includes 1 g of metformin twice daily (bd), ramipril
10 mg daily, indapamide 2.5 mg daily, aspirin 75 mg daily, bisoprolol 10 mg daily and
atorvastatin 10 mg daily. He has recently been admitted with an episode of cardiac failure. His
blood pressure is 148/85 mmHg, pulse is 65 bpm and regular. There are minor crackles at
both bases, on auscultation; no pitting oedema of the ankles is present. Body mass index is
34 kg/m 2.
Investigations:

Hb 132 g/l

WCC 7.1 × 10 9/l

PLT 201 × 10 9/l

Na + 139 mmol/l

K+ 4.4 mmol/l

Cr 110 µmol/l

Glycated haemoglobin (HbA1c) 61 mmol/l

Which of the following is the most appropriate intervention with respect to glycaemic
control?

A Empagliflozin

B Gliclazide

C Insulin glargine

D Pioglitazone

E Sitagliptin

Explanation 

A Empagliflozin
Empagliflozin has been shown to reduce the incidence of cardiac failure in patients with type
2 diabetes and reduces mortality from cardiovascular disease by around one third. It controls
blood glucose by increasing urinary glucose output and has modest positive effects on
weight control, without increasing the risk of hypoglycaemia in combination with metformin.
It is, therefore, the most appropriate intervention here.

(see External Links)

B Gliclazide

Gliclazide is incorrect. Gliclazide is not preferred here. Sulphonylureas lead to weight gain
and increase the risk of hypoglycaemia. Given that this patient is obese and works as a taxi
driver, he is likely to find it difficult to control his weight, and hypoglycaemia could put his
driving licence at significant risk.

C Insulin glargine

Insulin glargine is incorrect. Insulin promotes weight gain of up to 6 kg in the first year of
therapy and increases the risk of hypoglycaemia significantly. This is not desirable, given the
patient is obese and works as a taxi driver. Insulin may also increase the risk of cardiac failure
because it drives fluid retention and may increase sympathetic tone.

D Pioglitazone

Pioglitazone is incorrect. Pioglitazone is effective in controlling blood glucose, although it


promotes fluid retention and should not be used in patients who suffer from cardiac failure. It
promotes substantial weight gain, which equates to improvement in HbA1c. Given that this
patient is obese with a previous episode of cardiac failure, it is therefore not the best option.

E Sitagliptin

Sitagliptin is incorrect. Although dipeptidyl peptidase-IV (DPP-IV) inhibitors such as


sitagliptin may be an appropriate option here, they are less effective with respect to weight
control vs a sodium–glucose co-transporter-2 (SGLT-2) inhibitor and don’t have a positive
impact on cardiovascular outcomes. As such, empagliflozin is the preferred choice.
51363

Rate this question:

Next Question

Previous Question Tag Question

Feedback End Review


Difficulty: Average

Peer Responses %

Q. Answered Flagged

Q1

Q2

Q3

Q4

Q5

Q6

Q7

Q8

Q9

 External Links

Empagliflozin, Cardiovascular Outcomes, and Mortality in Type 2 Diabetes


nejm.org/doi/full/10.1056/nejmoa1504720
(https://www.nejm.org/doi/full/10.1056/nejmoa1504720)
0:02:24/03:00:00

A 24-year-old man with a history of ulcerative colitis (UC) is admitted with fulminant colitis,
under joint care with the gastrointestinal (GI) surgeons. He is opening his bowels up to ten
times a day with blood, mucus and diarrhoea. He has been taking oral prednisolone at home,
which has now been converted to intravenous (IV) methylprednisolone. He is also treated
with long-term oral mesalazine. His blood pressure is 100/70 mmHg, pulse is 92 bpm and
regular. He is pyrexial 38.4°C. Abdomen is generally tender with mild distension and active
bowel sounds.
Investigations:

Hb 102 g/l

WCC 13.7 × 10 9/l

PLT 389 × 10 9/l

Na + 142 mmol/l

K+ 4.9 mmol/l

Cr 152 µmol/l

CRP 189 mg/l

Abdominal X-ray shows colonic oedema with 6 cm of dilation.

Which of the following is the most appropriate next intervention?

A Azathioprine

B Colectomy

C Cyclophosphamide

D Cyclosporine

E Infliximab

Explanation 

D Cyclosporine
In this situation with severe/fulminant UC, cyclosporine used over the short-term can avoid
progression to colectomy in some patients. It is recommended when there is no response to
steroid therapy within 72 h or where symptoms worsen despite corticosteroid therapy.
Patients should be monitored and in the event that colonic dilatation or other symptoms
worsen then colectomy is the next step.

A Azathioprine

Azathioprine is incorrect. Both azathioprine and 6-mercaptopurine are appropriate to induce


long-term remission in UC; onset of action is too slow for use in acute therapy. For this
reason, cyclosporine is started first and used over the short-term as a bridge to remission.

B Colectomy

Colectomy is incorrect. Colectomy is the most appropriate next step in patients who fail to
respond to immune interventions. In this situation, with 6 cm of colonic dilation, there is a
small window of time during which medical remission can still be attempted.

C Cyclophosphamide

Cyclophosphamide is incorrect. Cyclophosphamide is used in combination with


corticosteroids to induce remission in renal and pulmonary vasculitis rather than in UC.

E Infliximab

Infliximab is incorrect. Infliximab can induce rapid remission in patients with UC. It is,
however, usually reserved for those who are intolerant of or unable to be treated with
cyclosporine therapy.
51376

Rate this question:

Next Question

Previous Question Tag Question

Feedback End Review

Difficulty: Average

Peer Responses %
Q. Answered Flagged

Q1

Q2

Q3

Q4

Q5

Q6

Q7

Q8

Q9

0:02:24/03:00:00

An 18-year-old woman is referred to the Renal Clinic with haematuria. Her two brothers were
also noted to have haematuria from a young age and they developed clinically significant
chronic renal impairment and hearing loss by the age of 20. Her blood pressure is 139/85
mmHg, pulse is 67 bpm and regular. Abdomen is soft and non-tender with no palpable
masses, and she has a body mass index of 22.5 kg/m 2.
Investigations:

Hb 121 g/l

WCC 6.5 × 10 9/l

PLT 201 × 10 9/l

Na + 144 mmol/l

K+ 5.2 mmol/l

Cr 141 µmol/l

Urine blood +, protein +

Which of the following is the most likely diagnosis?

A Alport syndrome

B Autoimmune polyglandular syndrome

C Membranoproliferative glomerulonephritis type 1 (MPGN1)

D Maturity-onset diabetes of the young (MODY) type 5

E Polycystic kidney disease

Explanation 

A Alport syndrome

This patient has an X-linked dominant form of progressive renal impairment and hearing loss,
known as Alport syndrome. It is caused by defects in type 4 collagen and leads to early-onset
haematuria, proteinuria and progressive sensorineural deafness which usually pre-dates renal
impairment. Symptoms are more severe in males than in females. Tight blood pressure
control with angiotensin-converting enzyme (ACE) inhibitors at full dose with spironolactone,
if required, is the intervention of choice.

B Autoimmune polyglandular syndrome

Autoimmune polyglandular syndrome is incorrect. This results in the development of


autoimmune-attack-related endocrine dysfunction rather than progressive renal impairment.

C Membranoproliferative glomerulonephritis type 1 (MPGN1)

MPGN1 is incorrect. MPGN is an acquired condition associated with nephritis, which does fit
with the blood results and proteinuria seen here. The strong X-linked heritability is out of
keeping with MPGN as the underlying diagnosis.

D Maturity-onset diabetes of the young (MODY) type 5

MODY type 5 is incorrect. MODY type 5 is associated with the development of diabetes
mellitus, along with hepatic and renal cysts.

E Polycystic kidney disease

Polycystic kidney disease is incorrect. This follows an autosomal-dominant inheritance


pattern, where males and females are equally affected and given that the creatinine in this
female patient is already abnormal, some renal cyst disease would be expected.
51433

Rate this question:

Next Question

Previous Question Tag Question

Feedback End Review

Difficulty: Average

Peer Responses %
Q. Answered Flagged

Q1

Q2

Q3

Q4

Q5

Q6

Q7

Q8

Q9

0:02:24/03:00:00

A 47-year-old woman with abnormal liver function testing is referred to the Gastroenterology
Clinic for liver biopsy. She has developed elevated transaminases and alkaline phosphatase,
with clinical symptoms of itching and easy bruising. Examination reveals blood pressure of
110/70 mmHg, pulse is 70 bpm and regular. There are signs of chronic liver disease, including
ascites.
Which of the following is a contraindication to transcutaneous liver biopsy?

A Alanine aminotransferase (ALT) 1215 IU/l

B Ascites

C Body mass index (BMI) 32 kg/m 2

D International normalised ratio (INR) 1.4

E Platelet count 72 × 10 9/l

Explanation 

B Ascites

Ascites is a contraindication for transcutaneous liver biopsy because it can make it


technically difficult to obtain an adequate sample or control bleeding, should this occur. For
this reason, transjugular liver biopsy is preferred to transcutaneous liver biopsy when there is
significant ascites.

A Alanine aminotransferase (ALT) 1215 IU/l

ALT 1215 IU/l is incorrect. There is no reason that elevated transaminases prevent liver biopsy
taking place. Evidence of hepatocellular dysfunction may not correlate well to hepatic
synthetic function, which predicts risk of bleeding in conjunction with liver biopsy.

C Body mass index (BMI) 32 kg/m 2


BMI 32 kg/m 2 is incorrect. Although obesity makes transcutaneous liver biopsy technically
more difficult, it doesn’t preclude it. Ultrasound scanning is, however, less useful in patients
who are morbidly obese.

D International normalised ratio (INR) 1.4

INR 1.4 is incorrect. INR > 1.6 is a contraindication to liver biopsy because of the risk of
bleeding. In this case, if INR is elevated and liver biopsy is required, it can be done under
fresh frozen plasma (FFP) cover.

E Platelet count 72 × 10 9/l

Platelet count 72 × 10 9/l is incorrect. Platelet count < 60 × 10 9/l is associated with increased
risk of bleeding, and this cut off is commonly used to rule out routine liver biopsy.
51377

Rate this question:

Next Question

Previous Question Tag Question

Feedback End Review

Difficulty: Average

Peer Responses %

Q. Answered Flagged

Q1

Q2

Q3
Q. Answered Flagged

Q4

Q5

Q6

Q7

Q8 
0:02:24/03:00:00

A 78-year-old man is admitted having suffered a transient ischaemic attack (TIA). He has
diet-controlled type 2 diabetes, erectile dysfunction and has suffered from frequent, severe
headaches over the past few months. He is under investigation by his GP for an increased
haemoglobin count. On examination, his blood pressure is 167/92 mmHg, pulse is 76 bpm and
regular. He has a ruddy complexion with some petechial bruising over his arms and legs. You
note a right-sided facial weakness which his relatives tell you has begun to resolve. He also
has a swollen knee with erythema, an effusion and limited flexion.
Investigations:

Hb 190 g/l

WCC 11.2 × 10 9/l

PLT 695 × 10 9/l

Na + 141 mmol/l

K+ 5.2 mmol/l

Cr 132 µmol/l

Which of the following is the most likely cause of this patient’s knee pain?

A Gout

B Osteoarthritis

C Pseudogout

D Psoriatic arthritis

E Septic arthritis

Explanation 

A Gout
This patient almost certainly has primary polycythaemia, associated with increased serum
urate and episodes of acute gout. In this situation, knee aspiration to confirm both white
blood cell count and to send for crystal examination and serum uric acid are obvious next
steps.

B Osteoarthritis

Osteoarthritis is incorrect. Although osteoarthritis can present with acute single joint pain, it’s
less likely here to be the cause versus inflammatory or infective arthritis.

C Pseudogout

Pseudogout is incorrect. Gout, rather than pseudogout is associated with polycythaemia,


although the presentation is similar between the two conditions. Joint aspiration and crystal
examination can differentiate between the two.

D Psoriatic arthritis

Psoriatic arthritis is incorrect. Psoriatic arthritis in the absence of nail pitting or typical
extensor surface psoriatic rash is extremely unlikely. A deforming arthritis of the hands is also
often seen in patients with psoriatic arthritis.

E Septic arthritis

Septic arthritis is incorrect. This is the main differential; knee aspiration for white cell count,
microscopy and crystal examination are clearly very important. Given the history of
polycythaemia, acute gout is, however, the more likely diagnosis.
51448

Rate this question:

Next Question

Previous Question Tag Question

Feedback End Review

Difficulty: Average

Peer Responses %
Q. Answered Flagged

Q1

Q2

Q3

Q4

Q5

Q6

Q7

Q8

Q9

0:02:24/03:00:00

You are asked to review a new potential treatment for a genetic disorder which occurs
because of intracellular accumulation of a signalling protein. The treatment induces
premature ribonucleic acid (RNA) splicing.
Where in the cell does RNA splicing occur?

A Cell membrane

B Cytoplasm

C Endoplasmic reticulum

D Golgi body

E Nucleus

Explanation 

E Nucleus

Splicing is the editing of pre-mRNA into a mature messenger RNA (mRNA). After splicing,
introns are removed and exons are joined together. For nuclear-encoded genes, splicing
takes place within the nucleus either during or immediately after transcription.

A Cell membrane

Cell membrane is incorrect. Membrane transport proteins exist which facilitate movement of
other proteins, ions or small molecules across the cell membrane, but splicing of RNA does
not occur at the level of the cell membrane itself.

B Cytoplasm

Cytoplasm is incorrect. Ribosomes which are involved in protein assembly are found in the
cytoplasm; splicing of RNA to produce mRNA for protein assembly occurs earlier, however,
within the nucleus.

C Endoplasmic reticulum
Endoplasmic reticulum is incorrect. The cytosolic face of the rough endoplasmic reticulum is
involved in protein synthesis and is studied with ribosomes, although it is not the site of RNA
splicing. The smooth endoplasmic reticulum lacks ribosomes and functions in lipid and
steroid hormone manufacture.

D Golgi body

Golgi body is incorrect. The function of the Golgi body is to sort and process peptides
produced by the rough endoplasmic reticulum; it is not responsible for RNA splicing. It
consists of a number of cisternae which fulfil different functions with respect to peptide
processing.
51397

Rate this question:

Next Question

Previous Question Tag Question

Feedback End Review

Difficulty: Average

Peer Responses %

Q. Answered Flagged

Q1

Q2

Q3

Q4

Q5
Q. Answered Flagged

Q6

Q7

Q8

0:02:24/03:00:00

A 62-year-old woman who has type 2 diabetes, hypertension, and smokes ten cigarettes per
day, comes to the Dermatology Department with an ulcer over her left medial malleolus. Her
blood pressure is 145/82 mmHg, pulse is 73 bpm and regular. Her ankle/brachial pressure
index (ABPI) is 0.7. There is an ulcer over the medial malleolus, with erythema, evidence of
eczema and sloughy material. There is minor loss of sensation over the soles of both feet, and
both dorsalis pedis and anterior tibial pulses are easily palpable.
Which of the following is the most likely cause of the ulcer?

A Arterial insufficiency

B Diabetic peripheral neuropathy

C Erythema nodosum

D Necrobiosis lipoidica diabeticorum (NLD)

E Venous insufficiency

Explanation 

E Venous insufficiency

Ulcers due to venous insufficiency are more likely to occur over the medial malleolus and are
associated with significant varicose eczema and surrounding erythema. The ABPI of 0.7
counts against an arterial ulcer as an alternative diagnosis. De-sloughing and compression
bandaging, with offloading where possible, are the interventions of choice.

A Arterial insufficiency

Arterial insufficiency is incorrect. The ABPI counts against a diagnosis of significant arterial
insufficiency, and medial malleolus ulceration would be unusual in this context. Absent
peripheral pulses and evidence of hair loss, blue/white discolouration and delayed venous
return in the toes would be expected.

B Diabetic peripheral neuropathy


Diabetic peripheral neuropathy is incorrect. More extensive sensory loss over the feet would
be expected, and neuropathic ulcers are usually found over the soles of the feet rather than
over the medial malleolus.

C Erythema nodosum

Erythema nodosum is incorrect. Erythema nodosum is associated with blue/purple


discoloured lesions over the shins rather than the picture seen here.

D Necrobiosis lipoidica diabeticorum (NLD)

NLD is incorrect. NLD is associated with enlarging red–brown lesions over the shins which
become atrophic with a yellowing centre over time. It doesn’t present with the lesion seen
here, which is much more characteristic of ulcerated varicose eczema.
51412

Rate this question:

Next Question

Previous Question Tag Question

Feedback End Review

Difficulty: Average

Peer Responses %

Q. Answered Flagged

Q1

Q2

Q3
Q. Answered Flagged

Q4

Q5

Q6

Q7

Q8 
0:02:24/03:00:00

A 19-year-old woman comes to the Endocrine Clinic for review. She suffers from
pseudohypoparathyroidism which is known to be due to a G-protein-coupled receptor
(GPCR).
Where are G-protein-coupled receptors found?

A In the cell membrane

B In the nucleolus

C In the nucleus

D On endoplasmic reticulum

E On ribosomes

Explanation 

A In the cell membrane

GPCRs consist of seven transmembrane domains or helices. They sit across the cell
membrane, and activation of GPCRs leads to a conformational change, where alpha and
beta/gamma subunits can then interact further with intracellular signalling mechanisms.
Examples of GPCR include the receptors for both thyroid and parathyroid hormones.

B In the nucleolus

In the nucleolus is incorrect. The nucleolus is primarily the site of ribosome synthesis; it also
captures and immobilises a number of intracellular proteins, a process known as nucleolar
dentition.

C In the nucleus

In the nucleus is incorrect. A number of nuclear receptors exist; the peroxisome proliferator-
activated receptors are one example. They regulate gene transcription and therefore can
have wide effects across a range of metabolic processes.
D On endoplasmic reticulum

On endoplasmic reticulum is incorrect. The endoplasmic reticulum is involved in protein


transport and synthesis. It contains receptors for ribosomes and is responsible for re-
processing of some GPCR components.

E On ribosomes

On ribosomes is incorrect. Ribosomes are the key players in protein synthesis, rather than
having a role in cell signalling. They contain receptors which are used to bind with
endoplasmic reticulum.
51365

Rate this question:

End Session

Previous Question Tag Question

Feedback End Review

Difficulty: Average

Peer Responses %

Q. Answered Flagged

Q1

Q2

Q3

Q4

Q5
Q. Answered Flagged

Q6

Q7

Q8

You might also like